Eureka 2001

Page 1

"! # $% & % ' ( ) ' $% * ( % *+( ' , . /103234 5 687 9 5*9:034 ; <:=35 9 > 7@?A/CB 6*53B /C7*D 7 9:5 "! # $% & % ' ( ) ' $% * ( % *+( ' , . /103234 5 687 9 5*9:034 ; <:=35 9 > 7"9:5 F3;3G >37*D 739H5 "! # $% & % ' ( ) ' $% * ( % *+( ' , . /103234 5 687 9 5*9:034 ; <:=35 9 > 78I 5 /CJ:53B /C7*D 739:5 "! # $% & % ' ( ) ' $% * ( % *+( ' , L 5 9:;34 I 73>30 9

E3 3E -3K

M N O%PCQ R S T U 8 V % ' , VT' ) X 5 /C6*73G 08Y /CB > ( *! # $' % Z ' [$' $ \*] T%$ (] ^'_V `$ `! V&' ( * % `! V! 3 V ! & b * % ! % ! V 3 d T%$ (# 8! ), , $% T% '$ % d T '

-3W K W a a c e f e E


Sociedade Brasileira de MatemĂĄtica

g hjik l m h

Realizamos durante o ano 2000 a XXII Olimpíada Brasileira de Matemåtica, atingindo na realização da Primeira Fase cerca de 80.000 alunos. Este ano a Olimpíada Brasileira de Matemåtica terå importantes modificaçþes em relação à aplicação das provas da segunda e terceira fases (para maiores esclarecimentos leia com atenção o nosso novo regulamento publicado no site: http://www.obm.org.br/). AlÊm disso a competição contarå pela primeira vez com a participação de alunos de ensino superior, para os quais foi criado o nível Universitårio. Assim, a partir deste ano, a OBM passa a ser realizada em 4 níveis de competição. O calendårio para este ano Ê o seguinte:

nVoqp%r%sqtvu(w x y z

{%|~}€  y }€|~‚ ƒ ‚(„ y SĂĄbado, 09 de junho %t y(Â… † ‡ ˆ ‚ ƒ ‚(„ y – –SĂĄbado, 01 de setembro ‰ y(|‹Š y }€|~‚ ƒ ‚(„ y – SĂĄbado, 20 de outubro (nĂ­veis 1, 2 e 3) Domingo, 21 de outubro (NĂ­veis 2 e 3 – Segundo dia de prova).

nVoqp%r%Œ �VnVsqp%r%ŽVt%sq‰ ��ŽVsq‘

t%y(… † ‡ ˆ[‚ ƒ ‚(„ y

{%|‹}q  y(}q|‹‚ ƒ ‚(„ y

– Såbado, 01 de setembro – Såbado, 20 de outubro e Domingo, 21 de outubro

GostarĂ­amos de registrar a realização da IV Semana OlĂ­mpica. Neste ano o evento teve lugar no ColĂŠgio Militar de Salvador (Salvador – BA) entre os dias 19 a 26 de janeiro. Aproveitamos a oportunidade para expressar o nosso agradecimento pela calorosa acolhida. Durante a IV Semana OlĂ­mpica, reunimos os alunos premiados na XXII OBM nos trĂŞs nĂ­veis de competição. Um arquivo com as aulas ministradas durante o evento pode ser consultado no seguinte endereço eletrĂ´nico: http://www.obm.org.br/semana.htm Por fim, queremos agradecer aos alunos que tĂŞm nos ajudado com a revisĂŁo da revista Â’b“•” Â’b–%—*˜

‘k„ y(ˆ }q™›š |‹y(„ Âœ EUREKA! N°10, 2001

2


Sociedade Brasileira de MatemĂĄtica

Â? Â? lžl Â&#x;iklž ¢¥ ÂŁ g g¼¤ m g h lžik lžm g j g  ZÂŚ l§ g ¨Â?ŠÂžÂŞÂŹÂŤ ­ ÂŽÂŹÂŻ ° ¹²Ž ¹³ª ­ ´ ¾¡œ Ž¹²¸° š ŠÂžÂş ÂŻÂ&#x;ÂŽ[Âş Šq° Âť ° ¹³Ž½Ÿ ž Âż1Ă€ ÂŽ ­³à 1. Observe as multiplicaçþes a seguir: 12 345 679 Ă— 18 = 222 222 222 12 345 679 Ă— 27 = 333 333 333 12 345 679 Ă— 54 = 666 666 666

Para obter 999 999 999 devemos multiplicar 12 345 679 por: A) 29 B) 99 C) 72 D) 41

E) 81

2. Outro dia ganhei 250 reais, incluindo o pagamento de horas extras. O salĂĄrio (sem horas extras) excede em 200 reais o que recebi pelas horas extras. Qual ĂŠ o meu salĂĄrio sem horas extras? A) 200 reais B) 150 reais C) 225 reais D) 175 reais E) 180 reais 3. Num relĂłgio digital, que marca de 0:00 atĂŠ 23:59, quantas vezes por dia o mostrador apresenta todos os algarismos iguais?

A) 10

B) 8

C) 6

D) 7

E) 9

4. A prefeitura de uma certa cidade fez uma campanha que permite trocar 4 garrafas de 1 litro vazias por uma garrafa de 1 litro cheia de leite. AtÊ quantos litros de leite pode obter uma pessoa que possua 43 dessas garrafas vazias? A) 11 B) 12 C) 13 D) 14 E) 15 5. Numa caixa havia vårias bolas, sendo 5 azuis, 4 amarelas, 3 vermelhas, 2 brancas e 1 preta. Renato retirou 3 bolas da caixa. Sabendo que nenhuma delas era azul, nem amarela, nem preta, podemos afirmar a respeito dessas 3 bolas que: A) são da mesma cor. B) são vermelhas. C) uma Ê vermelha e duas são brancas. D) uma Ê branca e duas são vermelhas. E) pelo menos uma Ê vermelha. 6. Se a årea do retângulo dado Ê 12, qual Ê a årea da figura sombreada?

A) 3

B) 4

C) 5

D) 6

EUREKA! N°10, 2001

3

E) 8


Sociedade Brasileira de Matemática

7. O número 10 pode ser escrito de duas formas como soma de dois números primos: 10 = 5 + 5 e 10 = 7 + 3. De quantas maneiras podemos expressar o número 25 como uma soma de dois números primos? A) 4 B) 1 C) 2 D) 3 E) nenhuma 8. 1 litro de álcool custa R$0,75. O carro de Henrique percorre 25 km com 3 litros de álcool. Quantos reais serão gastos em álcool para percorrer 600 km? A) 54 B) 72 C) 50 D) 52 E) 45 9. Um certo número N de dois algarismos é o quadrado de um número natural. Invertendo-se a ordem dos algarismos desse número, obtém-se um número ímpar. A diferença entre os dois números é o cubo de um número natural. Podemos afirmar que a soma dos algarismos de N é: A) 7 B) 10 C) 13 D) 9 E) 11 10. Juliano colou uma bandeirinha cinza em cada engrenagem, como mostra a figura abaixo:

As engrenagens são iguais e quando a engrenagem da esquerda girou um pouco, a sua bandeirinha ficou na posição indicada com a bandeirinha branca pontilhada. Nesta condição, podemos afirmar que a posição da bandeirinha na engrenagem da direita é: a) A)

b)

c)

B)

C)

d) D)

e) E)

11. Uma fábrica embala 8 latas de palmito em caixas de papelão cúbicas de 20 cm de lado. Para que possam ser melhor transportadas, essas caixas são colocadas, da melhor maneira possível, em caixotes de madeira de 80 cm de largura por 120 cm de comprimento por 60 cm de altura. O número de latas de palmito em cada caixote é A) 576 B) 4.608 C) 2.304 D) 720 E) 144 12. Há 18 anos Hélio tinha precisamente três vezes a idade de seu filho. Agora tem o dobro da idade desse filho. Quantos anos têm Hélio e seu filho? EUREKA! N°10, 2001

4


Sociedade Brasileira de Matemática

A) 72 anos e 36 anos. D) 50 anos e 25 anos.

B) 36 anos e 18 anos. E) 38 anos e 19 anos.

C) 40 anos e 20 anos.

13. Se os números naturais são colocados em colunas, como se mostra abaixo, debaixo de que letra aparecerá o número 2000? A 1

B

C 2

9 10

D

E 3

F

8 11

18

G 4

7

H 6

12

13

17

I 5

16

14 15

19

20

21

...

A) F

B) B

C) C

D) I

...

E) A

14. O emir Abdel Azir ficou famoso por vários motivos. Ele teve mais de 39 filhos, incluindo muitos gêmeos. De fato, o historiador Ahmed Aab afirma num dos seus escritos que todos os filhos do emir eram gêmeos duplos, exceto 39; todos eram gêmeos triplos, exceto 39; todos eram gêmeos quádruplos, exceto 39. O numero de filhos do emir é: A) 111 B) 48 C) 51 D) 78 E) 75 15. Quatro amigos vão visitar um museu e um deles resolve entrar sem pagar. Aparece um fiscal que quer saber qual deles entrou sem pagar. – –

Eu não fui, diz o Benjamim. Foi o Pedro, diz o Carlos.

– Foi o Carlos, diz o Mário. – O Mário não tem razão, diz o Pedro.

Só um deles mentiu. Quem não pagou a entrada do museu? A) Mário B) Pedro C) Benjamim E) não é possível saber, pois faltam dados

D) Carlos

16. Em um jogo de duas pessoas, os jogadores tiram, alternadamente, 1, 2, 3, 4 ou 5 palitos de uma pilha que inicialmente tem 1000 palitos. Ganha o jogador que tirar o último palito da pilha. Quantos palitos o jogador que começa deve tirar na sua jogada inicial de modo a assegurar sua vitória? A) 1 B) 2 C) 3 D) 4 E) 5 17. Quantos números inteiros e positivos menores do que 1.000.000 existem cujos cubos terminam em 1? A) 1.000 B) 10.000 C) 50.000 D) 100.000 E) 500.000 EUREKA! N°10, 2001

5


Sociedade Brasileira de MatemĂĄtica

18. Os 61 aprovados em um concurso, cujas notas foram todas distintas, foram distribuĂ­dos em duas turmas, de acordo com a nota obtida no concurso: os 31 primeiros foram colocados na turma A e os 30 seguintes na turma B. As mĂŠdias das duas turmas no concurso foram calculadas. Depois, no entanto, decidiu-se passar o Ăşltimo colocado da turma A para a turma B. Com isso: A) A mĂŠdia da turma A melhorou, mas a da B piorou. B) A mĂŠdia da turma A piorou, mas a da B melhorou. C) As mĂŠdias de ambas as turmas melhoraram. D) As mĂŠdias de ambas as turmas pioraram. E) As mĂŠdias das turmas podem melhorar ou piorar, dependendo das notas dos candidatos. 19. Escrevem-se, em ordem crescente, os nĂşmeros inteiros e positivos que sejam mĂşltiplos de 7 ou de 8 (ou de ambos), obtendo-se 7, 8, 14, 16, ... . O 100o nĂşmero escrito ĂŠ: A) 406 B) 376 C) 392 D) 384 E) 400 20. A figura abaixo foi desenhada em cartolina e dobrada de modo a formar um cubo.

Qual das alternativas mostra o cubo assim formado?

A)

C)

B)

D)

E)

Ă‚Ă„ĂƒĂ†Ă…\ĂƒĂ†Ă‡Ă‰ĂˆĂ‹ĂŠ ĂŒ T # Ă? ½ĂŽ 1) E 2) C 3) B 4) D 5) E

6) D 7) B 8) A 9) D 10) A

11) A 12) A 13) C 14) C 15) B

EUREKA! N°10, 2001

6

16) D 17) D 18) C 19) E 20) B


Sociedade Brasileira de MatemĂĄtica

Â? Â? lžl Â&#x;iklž ¢¥ ÂŁ g g¼¤ m g h lžik lžm g j g  ZÂŚ l§ g ¨Â?ŠÂžÂŞÂŹÂŤ ­ ÂŽÂŹÂŻ ° ¹²Ž ¹³ª ­ ´ ¾¡œ Ž¹²¸° š ŠÂžÂş ÂŻÂ&#x;ÂŽ[Âş Šq° Âť ° ¹³Ž½Ÿ ž Âż1Ă€ ÂŽ ­:Ă? 1. 2. 3. 4. 5.

Veja o problema 17 do nĂ­vel 1. Veja o problema 11 do nĂ­vel 1. Veja o problema 10 do nĂ­vel 1. Veja o problema 15 do nĂ­vel 1. Veja o problema 18 do nĂ­vel 1.

6. No triângulo ABC representado ao lado, a Medida do ângulo Cˆ ĂŠ 60° e a bissetriz do ângulo Bˆ forma 70° com a altura relativa ao vĂŠrtice A. A medida do ângulo Aˆ ĂŠ: A) 50°

B) 30°

C) 40°

D) 80°

E) 70°

B

A

C

7. Veja o problema 6 do nĂ­vel 1. 8. Alberto, Beatriz e Carlos correm numa pista circular. Todos saem ao mesmo tempo e do mesmo lugar, cada um desenvolvendo velocidade constante. Alberto e Beatriz correm no mesmo sentido. Correndo no sentido oposto, Carlos encontra Alberto, pela primeira vez, exatamente 90 segundos apĂłs o inĂ­cio da corrida e encontra Beatriz exatamente 15 segundos depois. Quantos segundos sĂŁo necessĂĄrios para que Alberto ultrapasse Beatriz pela primeira vez? A) 105 B) 630 C) 900 D) 1.050 E) nĂŁo pode ser determinado 9. DEFG ĂŠ um quadrado no exterior do pentĂĄgono regular ABCDE. Quanto mede o ângulo EĂ‚F? A) 9o B) 12o C) 15o D) 18o E) 21o 10. Quantos sĂŁo os nĂşmeros inteiros de 2 algarismos que sĂŁo iguais ao dobro do produto de seus algarismos? A) 0 B) 1 C) 2 D) 3 E) 4 11. Veja o problema 19 do nĂ­vel 1. 12. Uma caixa contĂŠm 900 cartĂľes, numerados de 100 a 999. Retiram-se EUREKA! N°10, 2001

7


Sociedade Brasileira de Matemática

ao acaso (sem reposição) cartões da caixa e anotamos a soma dos seus algarismos. Qual é a menor quantidade de cartões que devem ser retirados da caixa, para garantirmos que pelo menos três destas somas sejam iguais? A) 51 B) 52 C) 53 D) 54 E) 55 13. Se x e y são números reais positivos, qual dos números a seguir é o maior? A) xy

2

B) x + y

2

C) (x + y)

x3 + y3 D) x + y(x + y) E) x+ y

2

2

14. Na figura, as distâncias entre dois pontos horizontais consecutivos e as distâncias entre dois pontos verticais consecutivos são iguais a 1. A região comum ao triângulo e ao quadrado tem área:

14 15 a a +1 15. Sejam a e b números reais positivos tais que < 1. Então b b +1 a a a A) é igual a + 1. B) é igual a . C) é menor que . b b b a D) é maior que mas menor que 1. E) pode ser maior que 1. b A)

9 10

B)

15 16

C)

8 9

D)

11 12

E)

16. Veja o problema 16 do nível 1. 17. Quantos são os retângulos que têm os pontos A e B como vértices, e cujos vértices estão entre os pontos de interseção das 9 retas horizontais com as 9 retas verticais da figura abaixo?

B

A

A) 3

B) 4

C) 7

D) 2

EUREKA! N°10, 2001

8

E) 5


Sociedade Brasileira de MatemĂĄtica

18. Veja o problema 14 do nível 1. 19. De Itacimirim a Salvador, pela estrada do Coco, são 60 km. Às 11 horas, a 15 km de Salvador, då-se um acidente que provoca um engarrafamento, que cresce à velocidade de 4 km/h, no sentido de Itacimirim. A que horas, aproximadamente, devemos sair de Itacimirim para chegar a Salvador ao meiodia, sabendo que viajamos a 60 km/h, exceto na zona de engarrafamento, onde a velocidade Ê 6 km/h? A) 10h43min B) 10h17min C) 10h48min D) 10h53min E) 11h01min 20. Colocamos em ordem crescente os números escritos nas casas brancas do tabuleiro a seguir (estamos mostrando apenas as suas quatro primeiras linhas). Assim, por exemplo, o nono número da nossa lista Ê 14. Qual Ê o 2000o número da nossa lista?

‌ A) 3931

10 ‌

B) 3933

5 11 ‌

2 6 12 ‌

1 3 7 13 ‌

4 8 14 ‌

C) 3935

9 15 ‌

16 ‌

D) 3937

‌ E) 3939

Ă‚Ă„ĂƒĂ†Ă…\ĂƒĂ†Ă‡Ă‰ĂˆĂ‹ĂŠ ĂŒ T # Ă? ½Ă? 1) D 2) A 3) A 4) B 5) C

6) D 7) D 8) B 9) A 10) B

11) E 12) C 13) C 14) D 15) D

EUREKA! N°10, 2001

9

16) D 17) E 18) C 19) A 20) D


Sociedade Brasileira de MatemĂĄtica

Â? Â? lžl Â&#x;iklž ¢¥ ÂŁ g g¼¤ m g h lžik lžm g j g  ZÂŚ l§ g ¨Â?ŠÂžÂŞÂŹÂŤ ­ ÂŽÂŹÂŻ ° ¹²Ž ¹³ª ­ ´ ¾¡œ Ž¹²¸° š ŠÂžÂş ÂŻÂ&#x;ÂŽ[Âş Šq° Âť ° ¹³Ž½Ÿ ž Âż1Ă€ ÂŽ ­Ă’Ă‘ 1. Veja o problema 13 do nĂ­vel 2. 2. Veja o problema 9 do nĂ­vel 2. 3. Veja o problema 14 do nĂ­vel 2. 4. Escrevemos uma lista com todos os nĂşmeros inteiros de 1 a 30, inclusive. Em seguida, eliminamos alguns destes nĂşmeros de forma que nĂŁo sobrem dois nĂşmeros tais que um seja o dobro do outro. Qual ĂŠ a quantidade mĂĄxima de inteiros que podem permanecer na lista? A) 15 B) 18 C) 19 D) 20 E) 21 5. Veja o problema 15 do nĂ­vel 2. 6. Seja f uma função real que tem as seguintes propriedades: i) Para todos x, y reais, f(x + y) = x + f(y); ii) f(0) = 2. Quanto vale f(2000)? A) 0 B) 2 C) 1998 D) 2000

E) 2002

7. HĂĄ trĂŞs cartas viradas sobre uma mesa. Sabe-se que em cada uma delas estĂĄ escrito um nĂşmero inteiro positivo. SĂŁo dadas a Carlos, Samuel e TomĂĄs as seguintes informaçþes: i) todos os nĂşmeros escritos nas cartas sĂŁo diferentes; ii) a soma dos nĂşmeros ĂŠ 13; iii) os nĂşmeros estĂŁo em ordem crescente, da esquerda para a direita. Primeiro, Carlos olha o nĂşmero na carta da esquerda e diz: “NĂŁo tenho informaçþes suficientes para determinar os outros dois nĂşmeros.â€? Em seguida, TomĂĄs olha o nĂşmero na carta da direita e diz: “NĂŁo tenho informaçþes suficientes para determinar os outros dois nĂşmeros.â€? Por fim, Samuel olha o nĂşmero na carta do meio e diz: “NĂŁo tenho informaçþes suficientes para determinar os outros dois nĂşmeros.â€? Sabendo que cada um deles sabe que os outros dois sĂŁo inteligentes e escuta os comentĂĄrios dos outros, qual ĂŠ o nĂşmero da carta do meio? A) 2 B) 3 C) 4 D) 5 E) NĂŁo hĂĄ informaçþes suficientes para determinar o nĂşmero. EUREKA! N°10, 2001

10


Sociedade Brasileira de Matemática

8. Veja o problema 16 do nível 2. 9. Veja o problema 12 do nível 2. 10. A notação x significa o maior inteiro que não supera x. Por exemplo, 3,5 = 3 e 5 = 5. O número de inteiros positivos x para os quais

x + x  = 10 é: 1 2

1 3

A) 11

B) 12

C) 13

D) 14

E) 15

11. Veja o problema 20 do nível 2. 12. Veja o problema 18 do nível 1. 13. A figura abaixo mostra o logotipo de uma empresa, formado por dois círculos concêntricos e por quatro círculos de mesmo raio, cada um deles tangente a dois dos outros e aos dois círculos concêntricos. O raio do círculo interno mede 1 cm. Então o raio do círculo externo deverá medir, em cm:

A) 2 2 + 3

B)

2 +2

C) 4 2 + 1

D) 3 2

E)

2 +1

14. Veja o problema 8 do nível 2. 15. Veja o problema 10 do nível 2. 16. Dois nadadores, inicialmente em lados opostos de uma piscina, começam simultaneamente a nadar um em direção ao outro. Um deles vai de um lado a outro da piscina em 45 segundos e o outro em 30 segundos. Eles nadam de um lado para outro por 12 minutos, sem perder qualquer tempo nas viradas. Quantas vezes eles passam um pelo outro (indo no mesmo sentido ou em sentidos opostos) durante este tempo, contando as vezes em que se encontram nos extremos da piscina. A) 10 B) 12 C) 15 D) 18 E) 20 17. A soma de dois números naturais é 29. O mínimo valor para a soma de seus quadrados é: A) 785 B) 733 C) 647 D) 421 E) 334 18. Veja o problema 1 do nível 2. 19. Veja o problema 17 do nível 2. EUREKA! N°10, 2001

11


Sociedade Brasileira de MatemĂĄtica

20. Veja o problema 10 do nĂ­vel 1. 21. Na figura temos que os triângulos ABC e A’B’C’ sĂŁo equilĂĄteros e a regiĂŁo destacada ĂŠ um hexĂĄgono regular. A razĂŁo entre a ĂĄrea da regiĂŁo destacada e a ĂĄrea do triângulo ABC ĂŠ igual a: A C’ B’ O B C A’

A) 1

2 B) 3

4 C) 5

D)

2 2

E)

3 2

22. Veja o problema 14 do nível 1. 23. Veja o problema 19 do nível 2. 24. Seja P(x) = a2000x2000 + a1999x1999 + a1998x1998 + ‌ + a1x + a0. Então a2000 + a1998 + a1996 + ‌ + a0 Ê igual a A)

P(1) − P(−1) 2

B)

D) P(0) â‹… P(1)

P(1) + P(−1) 2

C) P(2000) + P(1998) + ‌ + P(0)

E) P(–1) ⋅ P(1)

25. Quantos números de três algarismos (que não começam com 0) possuem um algarismo que Ê a mÊdia aritmÊtica dos outros dois? A) 121 B) 117 C) 112 D) 115 E) 105

Ă‚Ă„ĂƒĂ†Ă…\ĂƒĂ†Ă‡Ă‰ĂˆĂ‹ĂŠ ĂŒ T # Ă? ½Ă“ 1) C 2) A 3) D 4) D 5) D

6) E 7) C 8) D 9) C 10) E

11) D 12) C 13) A 14) B 15) B

16) E 17) D 18) D 19) E 20) A

EUREKA! N°10, 2001

12

21) B 22) C 23) A 24) B 25) A


Sociedade Brasileira de MatemĂĄtica

Â? Â? lžl Â&#x;iklž ¢¥ ÂŁ g g¼¤ m g h lžik lžm g j g  ZÂŚ l§ g ¨Â?ŠÂžª¡ ­ Ž¡¯Ă”° ¹²Ž ¹³ª ­ ´ ¾¡œ Ž¹²¸° ¹³Ž3Ă• ´3ž ¸3° Âť ° Âą[ÂŽ Âź ž Âż1Ă€ ÂŽ ­[Ă ! & b * ĂŽ Ă– De quantas maneiras diferentes podemos construir um paralelepĂ­pedo usando exatamente 24 blocos cĂşbicos de medidas 1 Ă— 1 Ă— 1? Obs: Blocos de dimensĂľes 2 Ă— 3 Ă— 4 e 2 Ă— 4 Ă— 3 devem ser considerados iguais.

! & b * Ă? Ă–

O retângulo ao lado estå dividido em 9 quadrados, A, B, C, D, E, F, G, H e I. O quadrado A tem lado 1 e o quadrado B tem lado 9.

D

I

G C

Qual ĂŠ o lado do quadrado I?

F H

A B

! & b * Ă“ Ă–

E

Pintamos de vermelho ou azul 100 pontos em uma reta. Se dois pontos vizinhos são vermelhos, pintamos o segmento que os une de vermelho. Se dois pontos vizinhos são azuis, pintamos o segmento de azul. Finalmente, se dois pontos vizinhos têm cores distintas, pintamos o segmento de verde. Feito isto, existem exatamente 20 segmentos verdes. O ponto na ponta esquerda Ê vermelho. É possível determinar com estes dados a cor do ponto na ponta direita? Em caso afirmativo, qual a cor deste ponto?

! & b * Ă— Ă–

Desejamos escrever os inteiros de 1 a 10 nas casas do desenho ao lado de tal forma que quaisquer quatro números alinhados aparecem em ordem crescente ou decrescente. a) Mostre uma maneira de dispor os números respeitando estas condiçþes. b) Quais números podem aparecer nas pontas da estrela? c) Quais números podem aparecer nas outras cinco posiçþes?

! & b * Ă˜ Ă– Qual ĂŠ o menor inteiro positivo que ĂŠ o dobro de um cubo e o quĂ­ntuplo de um quadrado?

EUREKA! N°10, 2001

13


Sociedade Brasileira de MatemĂĄtica

! & b * Ă™ Ă–

Qual ĂŠ o maior inteiro positivo n tal que os restos das divisĂľes de 154, 238 e 334 por n sĂŁo iguais?

SÂ?RvĂš%Ă› Ăœ"Ă?vĂž'SĂ„Ă&#x; MĂ S½ĂžÂłQvĂ›"ĂĄkĂ&#x; MãâbMkS½Ăž\ä*ĂĄ ĂĽ1̽ĂžÂ?ÚÉç

(] ^'è% É$ É! V& * ĂŽ3Ă–

Sejam a ≤ b ≤ c as dimensĂľes do paralelepĂ­pedo. Temos que a, b, c âˆˆĂŠ * e

abc = 24. Como abc ≼ a.a.a ⇔ a ≤ 24, temos a ≤ 2, ou seja a = 1 ou a = 2 . Se a = 1, bc = 24. As possibilidades para b e c são b = 1 e c = 24; b = 2 e c = 12; b = 3 e c = 8; b = 4 e c = 6. Se a = 2, bc = 12. As possibilidades para b e c com b ≼ 2 são b = 2 e c = 6; b = 3 e c = 4. Assim, hå 6 maneiras de construirmos o paralelepípedo. 3

(] ^'è% É$ É! V& * Ă?3Ă–

O quadrado A medida de lado 1cm enquanto que o quadrado B tem medida de lado 9cm. DaĂ­ que as longitudes dos lados dos quadrados restantes sĂŁo: C = 10cm E = 8cm. F = 7cm G = 4cm. D = 14cm. I = 18cm.

(] ^'è% É$ É! V& * Ă“3Ă–

Temos que os segmentos verdes dividem os pontos da reta em conjuntos de pontos com cores iguais, sendo que o primeiro conjunto Ă esquerda contĂŠm pontos vermelhos, o segundo conjunto contĂŠm pontos azuis, o terceiro conjunto contĂŠm pontos vermelhos, e assim por diante. Como hĂĄ 20 segmentos verdes, temos 21 conjuntos de pontos. Assim, como o 21Âş conjunto contĂŠm pontos vermelhos, o ponto na ponta direita ĂŠ vermelho.

(] ^'è% É$ É! V& * Ă—3Ă–

5

6

1

4

7

3

1

10

4

3

8

10

8 6

5

2

7

2

9

9

1) 1 e 2 ocupam pontas vizinhas. É fĂĄcil ver que colocando o 2 no meio ou em uma ponta "oposta"a 1 o problema nĂŁo tem solução. EUREKA! N°10, 2001

14


Sociedade Brasileira de MatemĂĄtica

2) 9 e 10 ocupam pontas vizinhas. Pelo mesmo raciocinio anterior. 3) Uma vez que 1 e 2 estão colocados o 3 estå no meio, entre o 1 e o 2. Observe que colocar o 3 em qualquer outra posição leva a um absurdo. 4) Uma vez que 1, 2 e 3 estão colocados, fica claro que o 4 Ê vizinho ao 3. 5) Se 1, 2, 3 e 4 jå estão colocados, 5 pode estar no meio ou em uma ponta, e o mesmo ocorre com o 6. (ver figuras) Quando um deles estå numa ponta, o outro estå no meio. 6) O 7 estå no meio. Respostas: a) Ver figuras b) 1, 2, 9 e 10 obrigatórios mais 5 ou 6. c) 3, 4, 7, 8 obrigatórios mais 5 ou 6.

(] ^'è% É$ É! V& * Ă˜3Ă–

Decomponha N em primos = 2 a2 3 a3 ... Dobro de um cubo quer dizer que todos os ai sĂŁo mĂşltiplos de 3 exceto a2 que deixa resto 1 na divisĂŁo por 3. QuĂ­ntuplo de um quadrado quer dizer que todos sĂŁo pares exceto a5. Os menores expoentes possĂ­veis sĂŁo entĂŁo a2 = 4; a5 = 3 e os outros a3 = a7 =...= 0. Resposta: N = 24 53 = 2000.

(] ^'è% É$ É! V& * Ă™3Ă–

Dois nĂşmeros deixam o mesmo resto quando divididos por n se e sĂł se sua diferença ĂŠ mĂşltipla de n. Logo, as diferenças 238 – 154 = 84 e 334 – 238 = 96 sĂŁo ambas mĂşltiplas de n. Como n ĂŠ o maior possĂ­vel, concluĂ­mos que n deve ser o maior divisor comum de 84 e 96, que ĂŠ 12.

EUREKA! N°10, 2001

15


Sociedade Brasileira de MatemĂĄtica

Â? Â? lžl Â&#x;iklž ¢¥ ÂŁ g g¼¤ m g h lžik lžm g j g  ZÂŚ l§ g ¨Â?ŠÂžª¡ ­ Ž¡¯Ă”° ¹²Ž ¹³ª ­ ´ ¾¡œ Ž¹²¸° ¹³Ž3Ă• ´3ž ¸3° Âť ° Âą[ÂŽ Âź ž Âż1Ă€ ÂŽ ­HĂ? ! & b * ĂŽ Ă– Qual ĂŠ o menor inteiro positivo que ĂŠ o dobro de um cubo e o quĂ­ntuplo de um quadrado?

! & b * Ă? Ă–

De quantas maneiras diferentes podemos construir um paralelepĂ­pedo usando exatamente 216 blocos cĂşbicos de medidas 1 Ă— 1 Ă— 1? Obs: Blocos de dimensĂľes 2 Ă— 3 Ă— 36 e 2 Ă— 36 Ă— 3 devem ser considerados iguais.

! & b * Ă“ Ă–

F

D

No retângulo ABCD, E ĂŠ o ponto mĂŠdio do lado BC e F ĂŠ o ponto mĂŠdio do lado CD. A interseção ∧

C G E

o

de DE com FB ĂŠ G. O ângulo EAF mede 20 . ∧

Quanto vale o ângulo EGB ?

A

B

! & b * Ă— Ă–

O retângulo ao lado estå dividido em 9 quadrados, A, B, C, D, E, F, G, H e I. O quadrado A tem lado 1.

D

I

G

Qual ĂŠ o lado do quadrado I?

C F H

A

! & b * Ă˜ Ă–

B

E

Listamos os inteiros de 1 a n. Desta lista apagamos o inteiro m. A mÊdia dos n – 1 números restantes Ê

! & b * Ă™ Ă–

134 . Determine n e m. 11

O campeonato Venusiano de futebol Ê disputado por 10 times, em dois turnos. Em cada turno cada equipe joga uma vez contra cada uma das outras. Suponha que o Vulcano FC vença todas as partidas do 1o. turno. Caso não vença o 2o. turno, o Vulcano FC jogarå uma final contra o vencedor do 2o. turno, na qual terå vantagem caso faça mais pontos que o adversårio durante todo o campeonato (vitória vale 3 pontos, empate vale 1 ponto e derrota 0 pontos). EUREKA! N°10, 2001

16


Sociedade Brasileira de MatemĂĄtica

a) Determine o menor n tal que, se o Vulcano FC fizer exatamente n pontos no segundo turno, garantirĂĄ pelo menos a vantagem na final (independente de contra quem e com que placares conquiste os n pontos). b) Determine o menor n tal que, se o Vulcano FC fizer pelo menos n pontos no segundo turno, garantirĂĄ pelo menos a vantagem na final (independente de contra quem e com que placares conquiste os n pontos).

SÂ?RvĂš%Ă› Ăœ"Ă?vĂž'SĂ„Ă&#x; MĂ S½ĂžÂłQvĂ›"ĂĄkĂ&#x; MãâbMkS½Ăž\ä*ĂĄ ĂĽ1̽ĂžÂ?Ăš ĂŞ (] ^'è% É$ É! V& * ĂŽ3Ă– (] ^'è% É$ É! V& * Ă?3Ă–

Veja a solução do problema 5 do nível 1.

Sejam a ≤ b ≤ c as medidas do paralelepĂ­pedo. Temos entĂŁo que a, b e c sĂŁo inteiros positivos e abc = 216 . Como a â‹… b â‹… c ≼ a â‹… a â‹… a ⇔ a ≤ 6 e a | 216, temos a = 1, a = 2, a = 3, a = 4 ou a = 6. Se a = 1, temos b â‹… c = 216. As possibilidades neste caso sĂŁo b = 1 e c = 216; b = 2 e c = 108; b = 3 e c = 72; b = 4 e c = 54; temos a = 2, b = 6 e c = 36; b = 8 e c = 27; b = 9 e c = 24; b = 12 e c = 18. Se Temos entĂŁo as possibilidades b â‹… c = 108, com b ≼ 2. b = 2 e c = 54; b = 3 e c = 36; b = 4 e c = 27; b = 6 e c = 18; b = 9 e c = 12. Se a = 3, temos b â‹… c = 72, com b ≼ 3. Temos entĂŁo as possibilidades b = 3 e c = 24; b = 4 e c = 18; b = 6 e c = 12; b = 8 e c = 9. Se a = 6, a Ăşnica solução ĂŠ b = c = 6. Temos, assim, 19 maneiras de construirmos o paralelepĂ­pedo. Observação: pode-se verificar que o nĂşmero de soluçþes de b.c = r, com b ≤ c naturais, ĂŠ  d (n ) , onde [x ] denota o menor nĂşmero inteiro maior ou igual  2 

 

a x e d (n ) ĂŠ o nĂşmero de divisores de n. Assim, b â‹… c = 216 tem  d (216 ) = 8 soluçþes; b â‹… c = 108 com b ≼ 2 tem

 d (108 ) soluçþes  2  −1= 5  

aqui a solução b = 1 e c = 108 ); b â‹… c = 72 com b ≼ 3 tem



2



(descontamos  d (72 )  2 −2=4  

soluçþes (eliminamos b = 5 e c = 72 e b = 2 e c = 36 ); b â‹… c = 54 com b ≼ 4 tem  d (54 ) − 3 = 1 solução (eliminamos b = 1, b = 2 e b = 3 ) e b â‹… c = 36  

2

 

com b ≼ 6 tem  d (36 ) − 4 = 1 solução (elimina-se b = 1, 2 ,3 ou 4).  2 

EUREKA! N°10, 2001

17


Sociedade Brasileira de MatemĂĄtica

(] ^'è% É$ É! V& * Ă“3Ă– D

y

F

C G

^

^

^

2) E A B = E D C = y

θ

E

^

D E C = 90 ° − y = θ + x

x 20

A

^

1) F A D = F B C = x ⇒ x + y = 70°

o

y

90° − ( x + y ) = θ ⇒ θ = 20°

x B

(] ^'è% É$ É! V& * Ă—3Ă– Seja x o lado de B. O lado de C = x – 1, D = x + 5, E = x – 1, F = x – 2, G = 4, H = 2x – 3, I = x + 9 (=D + G) mas tambĂŠm ĂŠ 3x – 9 (=F + H – G). Assim x + 9 = 3x – 9 e x = 9. Assim, o lado de I ĂŠ 18.

(] ^'è% É$ É! V& * Ă˜3Ă–

A mÊdia aritmÊtica dos inteiros de 1 a n Ê (n + 1)/2. Quando se apaga um destes números, a menor mÊdia possível Ê a dos números de 1 a (n – 1), que Ê n / 2, e a maior Ê a dos números de 2 a n, que Ê n/2 +1. Logo, deve-se ter 2 n n 4 4 e, portanto, n Ê igual a 23 ou < 12 < + 1 o que fornece 22 ≤ n ≤ 24 11 11 2 11 2

24. Mas a mĂŠdia dos nĂşmeros restantes ĂŠ uma fração de denominador 11. Logo, a quantidade de nĂşmeros que restam no quadro deve ser mĂşltipla de 11. Portanto, n sĂł pode ser igual a 23. Finalmente, a soma dos nĂşmeros que restam ĂŠ 22 x 12 2/11 = 268.A soma dos nĂşmeros de 1 a 23 ĂŠ 23 x 12 = 276. Logo, o nĂşmero apagado foi m = 276 – 268 = 8.

(] ^'è% É$ É! V& * Ă™3Ă– No pior caso, o 2o. colocado do 1o. turno faz 24 pontos no 1o. turno. Se o Vulcano FC fizer 23 pontos no 2o. turno, ele ganharĂĄ 7 jogos e empatarĂĄ 2, e o 2o. colocado no 1o. turno chegarĂĄ a um mĂĄximo de 25 pontos (pois no mĂĄximo empatarĂĄ com o Vulcano FC) no segundo turno. Assim, o Vulcano FC terĂĄ vantagem na decisĂŁo, nesse caso. Note que se o Vulcano FC fizer 24 pontos no 2o. turno perdendo para o 2o. colocado do 1o. turno, este pode fazer 27 pontos no 2o. turno e ganhar a vantagem para a decisĂŁo. Se o Vulcano FC fizer 22 pontos ou menos e o Klingon FC tiver feito 24 pontos no 1o. turno poderĂĄ fazer 27 pontos no 2o. turno, somando 51 pontos, mais que os 49 (ou menos) pontos do Vulcano FC. Assim, a resposta da segunda pergunta ĂŠ n = 25, enquanto a resposta da 1a. pergunta ĂŠ n = 23.

EUREKA! N°10, 2001

18


Sociedade Brasileira de MatemĂĄtica

Â? Â? lžl Â&#x;iklž ¢¥ ÂŁ g g¼¤ m g h lžik lžm g j g  ZÂŚ l§ g ¨Â?ŠÂžª¡ ­ Ž¡¯Ă”° ¹²Ž ¹³ª ­ ´ ¾¡œ Ž¹²¸° ¹³Ž3Ă• ´3ž ¸3° Âť ° Âą[ÂŽ Âź ž Âż1Ă€ ÂŽ ­¡Ă‘ ! & b * ĂŽ Ă– ! & b * Ă? Ă– ! & b * Ă“ Ă–

Veja o problema 1 do nĂ­vel 2. Veja o problema 4 do nĂ­vel 2.

O trapĂŠzio ABCD tem bases AB e CD. O lado DA mede x e o lado BC mede 2x. A ∧

∧

∧

soma dos ângulos DAB e ABC Ê 120o. Determine o ângulo DAB .

! & b * Ă— Ă– ! & b * Ă˜ Ă–

Veja o problema 6 do nĂ­vel 2.

1 1 1 1 1 1 + 2 + 1 + 2 + 2 + ... + 1 + + ĂŠ racional; 2 2 1 2 2 3 2000 20012 p escreva-o na forma , p e q inteiros. q O nĂşmero

1+

! & b * Ă™ Ă–

Para efetuar um sorteio entre os n alunos de uma escola (n > 1) se adota o seguinte procedimento. Os alunos são colocados em roda e inicia-se uma contagem da forma "um, DOIS, um, DOIS,...". Cada vez que se diz DOIS o aluno correspondente Ê eliminado e sai da roda. A contagem prossegue atÊ que sobre um único aluno, que Ê o escolhido. a) Para que valores de n o aluno escolhido Ê aquele por quem começou o sorteio? b) Se hå 192 alunos na roda inicial, qual Ê a posição na roda do aluno escolhido?

SÂ?RvĂš%Ă› Ăœ"Ă?vĂž'SĂ„Ă&#x; MĂ S½ĂžÂłQvĂ›"ĂĄkĂ&#x; MãâbMkS½Ăž\ä*ĂĄ ĂĽ1̽ĂžÂ?Ăš ĂŤ (] ^'è% É$ É! V& * ĂŽ3Ă– Veja a solução do problema 1 do nĂ­vel 2. (] ^'è% É$ É! V& * Ă?3Ă– Veja a solução do problema 4 do nĂ­vel 2. (] ^'è% É$ É! V& * Ă“3Ă– Tracemos DM // BC (vide figura abaixo). Como ∠AMD = ∠ABC e ∠DAM + ∠AMD = ∠DAM + ∠ABC = 120° tem-se que ∠ADM = 60°. Como AD = x e BC = 2x, sendo P o ponto mĂŠdio de DM, entĂŁo, AD = DP = x e ADP ĂŠ um triângulo equilĂĄtero, isto ĂŠ, AP = x. Portanto APM ĂŠ um triângulo EUREKA! N°10, 2001

19


Sociedade Brasileira de MatemĂĄtica

isĂłsceles com ∠PAM = ∠AMP e como ∠DPA ĂŠ um ângulo externo do triângulo APM temos 60° = ∠DPA = ∠PAM + ∠AMP = 2. ∠AMP = 2. ∠ABC. Portanto, ∠ABC =30° e ∠DAB =120° – ∠ABC = 90°. D

C

P

A

(] ^'è% É$ É! V& * Ă—3Ă– (] ^'è% É$ É! V& * Ă˜3Ă– S=

2000

∑

1+

a =1

=

2000

∑

a =1

M

B

Veja a solução do problema 6 do nível 2.

2000 1 1 a 4 + 2 a 3 + 3a 2 + 2 a + 1 + = ∑ 2 a 2 (a + 1)2 a 2 (a + 1) a =1

1  a 2 + a + 1 2000  = ∑ 1 + 2  +a a2 + a a a =1 

= 2000 +

2000

1

1 



1

1

1



1

1 

∑  a − a + 1  = 2000 + 1 − 2  +  2 − 3  + ... +  2000 − 2001  a =1

= 2000 + 1 −

1 2000 = 2000 + 2001 2001

ĂŹ Ă­ ĂŽ(ĂŻ Ă°'Ăą%íÉò íÉó Ă´ Ă­VĂľ ĂŽ Ăś á*ø Ăš3Ăş

a) Para que o primeiro da fila seja o escolhido Ê preciso, antes de mais nada, que haja um número par de alunos (caso contrårio, ele serå eliminado quando começar a segunda rodada). Mais precisamente, o primeiro da fila Ê o escolhido se e só se, a cada rodada, a fila tem um número par de alunos. Portanto, o primeiro da fila Ê escolhido se e só se o número de alunos Ê uma potência de 2. b) Como 192 = 26 . 3, nas primeiras 6 rodadas a fila tem um número par de alunos. Após estas 6 rodadas, a fila se reduz a três alunos e Ê fåcil verificar que o escolhido Ê o terceiro deles. Resta, portanto, determinar quem são os alunos que restam após as primeiras 6 rodadas. Na primeira rodada, sobrevivem 1, 3, 5, 7, ..., 191. De um modo geral, sobrevivem à rodada de ordem n (n = 1, 2, ..., 6) os números da forma 2n . k + 1. Portanto, após 6 rodadas os sobreviventes são 1, 65 e 129 e o aluno escolhido Ê o de número 129.

EUREKA! N°10, 2001

20


Sociedade Brasileira de MatemĂĄtica

Ăť Ăťvߞß Ă˝Â&#x;Ăžkߞÿ ߞÞ Ăź jĂż Ăż Ăź

"!$#% #& $ '$( )$ *#%+*! ,- . /0 $1 ! 2 !$#& 3547698 $ ;:

ó ô í þ ÎbÜ á*ø5< ú Paulo tem três dados comuns idênticos nos quais a soma dos números em duas faces opostas Ê sempre igual a 7. Ele cola os dados, de modo que cada par de faces coladas tenha o mesmo número, e depois os coloca sobre uma mesa não transparente, conforme indica a figura. A soma dos números em todas as onze faces visíveis Ê 36. Qual Ê a soma dos números das três faces que estão em contato com a mesa?

ó ô í þ ÎbÜ á*ø5= ú Um triângulo equilåtero pode ser recortado em triângulos equilåteros menores. A figura abaixo mostra como recortar um triângulo equilåtero em 7 triângulos equilåteros. Mostre como recortar um triângulo equilåtero em 20 triângulos equilåteros menores.

ó ô í þ ÎbÜ á*ø5> ú Isabel tem dois baralhos, cada um com 50 cartas. Em cada um dos baralhos estão escritos os números de 1 a 100 (em cada carta estão escritos dois números, um em cada face da carta). Por um defeito de fabricação, a distribuição dos números nas cartas não Ê a mesma nos dois baralhos (por exemplo, em um dos baralhos o 1 aparece na mesma carta do 2; no outro, o 1 aparece com o 76). Mostre como Isabel deve fazer para que, ao colocar as 100 cartas sobre uma mesa, as faces voltadas para cima mostrem todos os números de 1 a 100.

ó ô í þ ÎbÜ á*ø5? ú

Considere a seguinte tabela 5 Ă— 5, preenchida com os nĂşmeros de 1 a 25. 1 6 11 16 21

2 7 12 17 22

3 8 13 18 23

EUREKA! N°10, 2001

21

4 9 14 19 24

5 10 15 20 25


Sociedade Brasileira de MatemĂĄtica

Em cada fileira horizontal e em cada fileira vertical, trocamos o sinal de 2 nĂşmeros, de forma que, em cada fileira horizontal e em cada fileira vertical, haja 3 nĂşmeros positivos e 2 nĂşmeros negativos. Somamos, entĂŁo, todos os nĂşmeros da tabela. Calcule os possĂ­veis valores dessa soma.

@ ACBED F G%H;@JI KMLNH O PQH;RSO KUTVKW@ HCX Y[Z]\^H B`_

ó ô í þ ÎbÜ á*ø5< ú Veja a solução do problema 1 do nível 2.

ó ô í þ ÎbÜ á*ø5= ú(Ï í Î(ï ð'ù íZò Ü`þ;a ô%þ ø ô%ø"a ô%ø þ Ü`Ï ø%ô ø;b c ø5dCÏ øfehg í Ï i(Ï ókj Esta Ê uma forma de dividir um triângulo equilåtero em 20 triângulos equilåteros menores. Dividindo o maior triângulo em 4 partes (triângulos equilåteros menores), dois de esses quatro menores triângulos em nove outros menores triângulos equilåteros, obteremos 20 triângulos equilåteros: 2 maiores e 18 menores.

Ăł Ă´ Ă­ Ăľ ĂŽbĂś á*ø5> Ăş Veja a solução do problema 2 do nĂ­vel 2. Ăł Ă´ Ă­ Ăľ ĂŽbĂś á*ø5? Ăş(ĂŹ Ă­ ĂŽ(ĂŻ Ă°'Ăą Ă­Zò øÔþ øEefl%ø

Escrevemos os nĂşmeros da tabela na seguinte forma: 0+1 5+1 10 + 1 15 + 1 20 + 1

0+2 5+2 10 + 2 15 + 2 20 + 2

0+3 5+3 10 + 3 15 + 3 20 + 3

0+4 5+4 10 + 4 15 + 4 20 + 4

0+5 5+5 10 + 5 15 + 5 20 + 5

Cada nĂşmero ĂŠ da forma 5a + b, com 0 ≤ a ≤ 4 e 1 ≤ b ≤ 5. Depois de trocar de sinal temos que em cada linha hĂĄ dois nĂşmeros negativos, se em cada linha fazemos a soma sĂł das partes Âą 5a temos que a soma dessa linha ĂŠ 5a (jĂĄ que hĂĄ trĂŞs 5a e dois − 5a ) e a soma de todas as linhas considerando somente os nĂşmeros 0 +5 +10 + 15 + 20 = 50. Agora consideremos os nĂşmeros Âą b . Em cada coluna hĂĄ dois nĂşmeros b que trocaram de sinal e trĂŞs que nĂŁo, portanto a soma dos nĂşmeros dessa coluna ĂŠ b e a soma das colunas considerando somente as partes Âą b , ĂŠ: 1 + 2 + 3 + 4 + 5 = 15. Logo a soma total ĂŠ sempre 65. EUREKA! N°10, 2001

22


Sociedade Brasileira de MatemĂĄtica

Ăť Ăťvߞß Ă˝Â&#x;Ăžkߞÿ ߞÞ Ăź jĂż Ăż Ăź

"!$#% #& $ '$( )$ *#%+*! ,- . /0 $1 ! 2 !$#& 3547698 $ nm

ó ô í þ ÎbÜ á*ø5< ú

Veja o problema 1 do nĂ­vel 1.

ó ô í þ ÎbÜ á*ø5= ú

Isabel tem dois baralhos, cada um com 50 cartas. Em cada um dos baralhos estão escritos os números de 1 a 100 (em cada carta estão escritos dois números, um em cada face da carta). Por um defeito de fabricação, a distribuição dos números nas cartas não Ê a mesma nos dois baralhos (por exemplo, em um dos baralhos o 1 aparece na mesma carta do 2; no outro, o 1 aparece com o 76). Mostre como Isabel deve fazer para que, ao colocar as 100 cartas sobre uma mesa, as faces voltadas para cima mostrem todos os números de 1 a 100.

ó ô í þ ÎbÜ á*ø5> ú

Em uma folha de papel a reta r passa pelo canto A da folha e forma um ângulo Îą com a borda horizontal, como na figura 1. Para dividir este ângulo Îą em trĂŞs partes iguais, executaremos as seguintes construçþes: a) inicialmente, marcamos dois pontos B e C sobre a borda vertical de modo que AB = BC; pelo ponto B traçamos a reta s paralela Ă borda (figura 2); b) a seguir, dobramos o papel, ajustando-o de modo que o ponto C coincida com um ponto C’ sobre a reta r e o ponto A coincida com um ponto A’ sobre a reta s (figura 3); chamamos de B’ o ponto com o qual B coincide. Mostre que as retas AA’ e AB’ dividem o ângulo Îą em trĂŞs partes iguais. r

r

r C’ C B

ó ô í þ ÎbÜ á*ø5? ú

Îą A

A

Figura 1

A’

B

Îą

Îą A

B’

C s

Figura 2

Figura 3

É possĂ­vel encontrar duas potĂŞncias de 2, distintas e com o mesmo nĂşmero de algarismos, tais que uma possa ser obtida atravĂŠs de uma reordenação dos dĂ­gitos da outra? EUREKA! N°10, 2001

23


Sociedade Brasileira de MatemĂĄtica

@ ACBED F G%H;@JI KMLNH O PQH;RSO KUTVKW@ HCX Y[Z]\^H BQo Ăł Ă´ Ă­ Ăľ ĂŽbĂś á*ø5< Ăş ĂŹ Ă­ ĂŽ ĂŻ%Ă° Ăą Ă­Zò'Ăś ø ĂŽbĂśqpr'l Ă­ Ă´ Ă´E s ø ø'Ăľ Ă´%Ăś ĂŻ5d9efb %g Ăś Ă´;t7bq i Ă´fuvj Sejam a e b os nĂşmeros das faces coladas. Como a ĂŠ o nĂşmero da face oposta Ă face de b, no dado central, temos que a + b = 7. A soma dos nĂşmeros das faces de cada dado ĂŠ 21, entĂŁo a soma dos nĂşmeros das faces de todos os dados ĂŠ 63, mas a soma das faces coladas ĂŠ 2 (a + b) = 14, e a soma das faces visĂ­veis ĂŠ 36, temos, entĂŁo, que a soma de nĂşmeros das faces em contato com a mesa ĂŠ: 63 – 36 – 14 = 13. Resposta: a soma ĂŠ 13.

Ăł Ă´ Ă­ Ăľ ĂŽbĂś á*ø5= Ăş ĂŹ Ă­ ĂŽ ĂŻ%Ă° Ăą íÉò Ăśwckb g'Ă­ Ă´"x ø%Ăľ Ă´fb)Ăś ĂŽ^y ĂŽ Ăśkb e%Ăś d1á8Ă­ x7b ò%ø ĂŹwl Ă´ ĂŻfz(Ăś ĂŹ i ĂŹ[Ăł{j

Podemos fazer isso pegando qualquer carta de qualquer baralho, colocando sobre a mesa e vendo seu verso. Depois disso procuramos a carta de mesmo nĂşmero do verso (procurando no outro baralho, jĂĄ que foi usada no primeiro baralho). Fazemos com esta carta o mesmo que foi feito com a primeira carta. Continua-se a fazer isso atĂŠ fechar um ciclo (um mesmo nĂşmero que jĂĄ saiu em um baralho sair no outro). Exemplo: baralho:

verso:

1

2

1

2

x

y

z

w

y

z

w

x

Ciclo fechado.

Quando um ciclo for fechado pega-se outra carta e começa um novo ciclo. Fazendo isso atÊ o final das cartas as faces voltadas para cima mostrarão todos os números de 1 a 100.

Ăł Ă´ Ă­ Ăľ ĂŽbĂś á*ø5> Ăş Veja solução do problema 1 do nĂ­vel 3. Ăł Ă´ Ă­ Ăľ ĂŽbĂś á*5ø ? Ăş(ĂŹ Ă­ ĂŽ(ĂŻ Ă°'Ăą Ă­Zò Ăś`ĂŹ ø'á8ĂŻ%Ăś Εþ ø%Ă´ Ăľ'Ă­VĂŹ 5ø | kĂś b g'Ă­VĂŹ 5ø d9|(Ă­ Ă´}g ø'ĂŽbĂśkz ø5ihl Ăśkj

Sejam A = 2 m e A' = 2 n onde A' ĂŠ uma reordenação dos dĂ­gitos de A, suponha sem perda de generalidade que A > A' , daĂ­ A ĂŠ um mĂşltiplo de A' pois possui os mesmos fatores primos. EntĂŁo temos que A = A'â‹…k , onde k ∈ Z , k > 1 (pois caso nĂŁo fosse ⇒ A' = A , o que seria um absurdo, pois A e A' sĂŁo distintos.) e k ĂŠ uma potĂŞncia de dois, pois A sĂł possui fatores primos iguais a 2, daĂ­ k = 2,4,8 ⇒ A = 2 A' ou A = 4A' ou A = 8A'. Como a soma de seus dĂ­gitos ĂŠ a mesma, A e A' deixam o mesmo resto (mod 9) e sua diferença ĂŠ divisĂ­vel por 9, mas A – A' sĂł pode se: A', 3A', 7A', onde nenhuma desas diferenças ĂŠ divisĂ­vel por 9. DaĂ­ nĂŁo existem tais nĂşmeros. Nenhum nĂşmero ĂŠ divisĂ­vel por 9, pois cada um desses nĂşmeros nĂŁo possui pelo menos dois fatores primos iguais a 3. EUREKA! N°10, 2001

24


Sociedade Brasileira de MatemĂĄtica

Ăť Ăťvߞß Ă˝Â&#x;Ăžkߞÿ ߞÞ Ăź jĂż Ăż Ăź

~V  €}‚hƒ „h…{†0‡ „ ‡v€hƒ]ˆn‰vŠ}„h‡W‹h†%Œ „h Ž�q„h�  †%� †h‡q„ ‘{’;“-”•„0ƒq–

ó ô í þ ÎbÜ á*ø5< ú ó ô í þ ÎbÜ á*ø5= ú

Veja o problema 3 do nĂ­vel 2.

Seja Ďƒ (n) a soma de todos os divisores positivos de n, onde n ĂŠ um inteiro positivo (por exemplo, Ďƒ (6) = 12 e Ďƒ (11) = 12). Dizemos que n ĂŠ quase perfeito se Ďƒ (n) = 2n − 1 (por exemplo, 4 ĂŠ quase perfeito, pois Ďƒ ( 4) = 7). Sejam n mod k n

o resto da divisĂŁo de n por k e s( n) = ∑ n mod k (por exemplo: s(6) = 0 + 0 + 0 + k =1

2 + 1 + 0 = 3 e s(11) = 0 + 1 + 2 + 3 + 1 + 5 + 4 + 3 + 2 + 1 + 0 = 22). Prove que n ĂŠ quase perfeito se, e somente se, s( n) = s (n − 1).

ó ô í þ ÎbÜ á*ø5> ú

Seja f uma função definida nos inteiros positivos da seguinte forma: Dado n, escrevemos n = 2 a â‹… (2b + 1), com a e b inteiros e definimos f (n) = a 2 + a + 1. Determine o menor inteiro positivo n tal que f (1) + f (2) + ... + f (n) ≼ 123456.

ó ô í þ ÎbÜ á*ø5? ú

A avenida Providência tem infinitos semåforos igualmente espaçados e sincronizados. A distância entre dois semåforos consecutivos Ê de 1.500m. Os semåforos ficam abertos por 1 min 30s, depois fechados por 1 min, depois abertos por 1 min 30s e assim sucessivamente. Suponha que um carro trafegue com velocidade constante igual a v, em m/s, pela avenida Providência. Para quais valores de v Ê possível que o carro passe por uma quantidade arbitrariamente grande de semåforos sem parar em qualquer um deles?

Ăł Ă´ Ă­ Ăľ ĂŽbĂś á*ø5— Ăş

Seja X o conjunto de todas as seqßências a = (a1 , a 2 ,..., a 2000 ) tais que a i ∈ {0,1,2} se 1 ≤ i ≤ 1000 e a i ∈ {0,1} se 1001 ≤ i ≤ 2000. Dados a e b em X, definimos a distância d (a, b) entre a e b como sendo o nĂşmero de valores de i, 1 ≤ i ≤ 2000, tais que a i ≠bi . Determine o nĂşmero de funçþes f : X → X que EUREKA! N°10, 2001

25


Sociedade Brasileira de MatemĂĄtica

preservam distância, isto Ê, tais que d ( f (a ), f (b)) = d (a, b), para quaisquer a e b em X.

ó ô í þ ÎbÜ á*ø Ú ú Seja C um cubo de madeira. Para cada um dos 28 pares de vÊrtices de C cortamos o cubo C pelo plano mediador dos dois vÊrtices do par. Em quantos pedaços fica dividido o cubo? Nota: Dados dois pontos A e B no espaço, o plano mediador de A e B Ê o conjunto dos pontos do espaço cujas distâncias a A e B são iguais. Em outras palavras: Ê o plano perpendicular ao segmento AB passando pelo ponto mÊdio de AB.

@ ACBED F G%H;@JI KMLNH O PQH;RSO KUTVKW@ HCX Y[Z]\^H BQ˜

Ăł Ă´ Ă­ Ăľ ĂŽbĂś á*ø5< Ăş(ĂŹ Ă­ ĂŽ(ĂŻ Ă°'Ăą Ă­Zò Ăś á8ø%Ă´}g;™ ø Ăłkš ø Ă´%øf›fu Ă­Zò Ăś á8Ă­VĂ´'ø%Ăś ĂŹ d]| Ă­ Ă´}g'ø ĂŽ(Ăśkz(ø"i}l Ăśkj

r C’ B’ B

A’

X Îą

A

K

P

Veja que AP = A' P , entĂŁo ∆AA' P ĂŠ isĂłsceles. Seja PAˆ A' = Îą entĂŁo AAˆ ' B = Îą ( BA' // AP). Note que ∆APX ≥ ∆A' PX , DaĂ­: XAˆ P = XAˆ ' P Îą + XAˆ A' = 2Îą XAˆ A' = Îą Agora observe que: BAˆ X = 90 − 2Îą → BXˆA = 2Îą EUREKA! N°10, 2001

26


Sociedade Brasileira de MatemĂĄtica

B Aˆ X = 90 − 2Îą → B ' Xˆ A' = 2Îą Donde segue que os pontos A, X, B' sĂŁo colineares. Como CB = C ' B ' , AB = A' B ' e CB = AB, temos que C ' B ' = A' B ' . EntĂŁo AB' ĂŠ mediana e altura do ∆C ' AA' , sendo, conseqĂźentemente, bissetriz do ∆C ' AA'. DaĂ­: C ' Aˆ B ' = B ' Aˆ A' = PAˆ A' = Îą .

Ăł Ă´ Ă­ Ăľ ĂŽbĂś á*ø5= Ăş(ĂŹ Ă­ ĂŽ(ĂŻ Ă°'Ăą Ă­Zò Ăś{|(ø%Ăľ Ă´;Âœ lfb)Ă­\ĂŹkb Â?VĂŻ Ăśkb)Ă´ øÔþ%Ăśke%Ăśkckb)ò Ăś ĂŹwd]| Ă­ Ă´hg ø ĂŽ Ăśkz ø5ihl%Ăśkj

Fixe n. Seja a i = n mod i e bi = (n − 1) mod i n

n

Temos s (n) = ∑ a i e s( n − 1) = ∑ bi i =1

Veja

i =1

que

se

d n,

por

definição,

a d = 0,

e

que

n ≥ 0(mod d ) ⇒ n − 1 ≥ −1(mod d ) ⇒ bd = d − 1 (jĂĄ que 0 ≤ bd ≤ d − 1) (inclusive se d = 1) AlĂŠm disso se t /\ n, a t > 0 e ĂŠ fĂĄcil ver que bt = at − 1. Sendo assim: n

n−1

i =1

i =1

s(n) = s(n −1) ⇔ ∑ai = ∑bi ⇔ ∑ad + ∑at + an = ∑bd + ∑bt ⇔ ∑(bd − ad ) = d |n d ≠n

t \/ n

d|n d ≠n

t \/ n

d |n d ≠n

= ∑ (at − bt ) ⇔ ∑ (d − 1) = ∑1 t /| n

t /| n

d |n d ≠n

Seja f(n) o nĂşmero de divisores de n. Temos:

∑(d −1) = ∑1 ⇔∑d − ∑1 = ∑1 ⇔ (Ďƒ (n) − n) − ( f (n) −1) = n − f (n) â‡”Ďƒ (n) − n − f (n) +1 =

d|n d ≠n

t /| n

d|n d ≠n

d|n d ≠n

t /| n

= n − f ( n ) ⇔ Ďƒ ( n ) = 2 n − 1. De modo que s (n) = s (n − 1) ⇔ Ďƒ (n) = 2n − 1.

ó ô í þ ÎbÜ á*ø5> ú(Ï í Î(ï ð'ù íZò Ü`ï'Î}b Ï Ï Ü Ï á*Ü ò Ükb ô%í Ï`ò'Ü ø Î þ%ïf� ï%Ü ôf� ï Ü d]| í ôhg ø Î(Ükz(ø"i}l'Ükj

Considere as representaçþes binĂĄrias dos nĂşmeros, ex: 17 = (10001); 24 = (11000) e 5 = (101) Seja n na base 2 igual a (...a i ...a 3 a 2 a1 a 0 ), onde a i = 0 ou a i = 1, ∀i ∈ Z + se 2 j > n ⇒ a j = 0.

EUREKA! N°10, 2001

27


Sociedade Brasileira de Matemática

n = 2 a ⋅ (2b + 1) ⇔ a é a quantidade de zeros à direita na sua representação binária. Exemplo: a para o 24 é 3, já a = 0 para o 17 e o 5. Isto vem exatamente do que significa a representação de um número em uma dada base. (*) Seja S k = f (1) + f (2) + f (3) + ... + f (2 k ) Como a só depende da quantidade de zeros no final (*), temos que se 2 j > n, n ≥ 1 então f (2 j + n) = f (n), pois terão a mesma quantidade de zeros à direita na base 2. Assim, S k = f (1) + f (2) + ... f (2 k −1 − 1) + f (2 k −1 ) + f (2 k −1 + 1) + f (2 k −1 + 2) + ... + f (2 k ) S k = ( f (1) + f (2) + ... + f (2 k −1 − 1) + f (2 k −1 )) + ( f (1) + f (2) + ... + f (2 k −1 − 1)) + f (2 k )

S k = ( S k −1 ) + ( S k −1 − f (2 k −1 )) + f (2 k )

[

] [

]

S k = S k −1 + S k −1 + − (k − 1) 2 − (k − 1) − 1 + k 2 + k + 1 S k = 2 ⋅ S k −1 + 2 ⋅ k S k = 2 ⋅ ( S k −1 + k ). Primeiros Sk's: S0 = 1, S1 = 4, S2 = 12, S3 = 30, S4 = 68, S5 = 146, S6 = 304, S7 = 622, S8 = 1260, S9 = 2538, S10 = 5096, S11 = 10214, S12 = 20452, S13 = 40930, S14 = 81888, S15 = 163806 +∞

Seja g (n) = f (1) + f (2) + ... + f (n), provaremos que g ( n) = ∑ a i ⋅ S i , onde i =1

n = (...a j ...a 5 a 4 a 3 a 2 a1 a 0 ). Seja j o maior possível tal que a j = 1. n = 2 j + a j −1 ⋅ 2 j −1 + ... + a1 ⋅ 2 + a 0 ⋅ 2 0

g(n) = ( f (1) + f (2) + ...+ f (2 j )) + ( f (2 j +1) + f (2 j + 2) + ...+ f (2 j + a j−1 ⋅ 2 j−1 + ...a0 )) g (n) = ( S j ) + f (1) + f (2) + ... + f (a j −1 ⋅ 2 j −1 + ... + a 0 )) De modo análogo, tomamos o maior j 0 , tal que j > j 0 e a j0 = 1.

g(n) = S j + ( f (1) + f (2) + f (3) + ... + f (2 j0 )) + ( f (2 j0 +1) + ...+ f (2 j0 + a j0 −1 ⋅ 2 j0 −1 + ... + a0 )) g ( n) = S j + ( S j0 ) + ( f (1) + f ( 2) + ... + f (a j0 −1 ⋅ 2 j0 −1 + ... + a 0 ))

De maneira análoga, fazemos (vamos baixando) para todos os ai's = 1.

EUREKA! N°10, 2001

28


Sociedade Brasileira de Matemática +∞

Como a i = 1 ou a i = 0, podemos escrever g (n) = ∑ a i ⋅ S i i =1

Para termos o menor n, tal que g (n) ≥ 123456 Temos que conseguir uma soma de S k 's ≥ 123456, com os menores k's possíveis, pois isto se refletirá em (...a i ...a 2 a1 a 0 ) com os menores i's possíveis. Mas isto é uma tarefa fácil se tomarmos os Sk's calculados na página anterior e também sabendo que: S k > 2 ⋅ S k −1 > S k −1 + 2S k − 2 ... > S k −1 + S k − 2 + S k − 3 + ... + S 0

Daí, temos que a soma procurada é: S14 + S13 + S 7 + S 2 + S1 = 81888 + 40930 + 622 + 12 + 4 = 1234456

Assim, o menor n tal que g ( n) ≥ 123456 é (110000010000110)2 n = 214 + 213 + 2 7 + 2 2 + 21 = 16384 + 8192 + 128 + 4 + 2

n = 24710 O menor inteiro positivo tal que f (1) + ... + f (n) ≥ 123456 é 24710.

k f ¡f¢ ö ÷*ø5? ú}£ ¢}¤f¥;¦f § ø ¡ øEefl%ø Suponha que no tempo 0 os sinais se abram e que o carro passe pela primeira vez por um sinal no tempo t 0 ≥ 0 (mediremos o tempo sempre em segundos). Os sinais estarão abertos entre os tempos 150k e 150k + 90 e fechados entre os tempos 150k + 90 e 150(k + 1), para todo inteiro k. O carro passará pelos sinais 1500 nos tempos t 0 + r , para todo inteiro não negativo r . Assim, a condição v necessária e suficiente para que o carro encontre sempre o sinal aberto é que t0 10 3 + r seja igual a um inteiro mais um número entre 0 e para todo r 150 v 5 10 inteiro. Isso é claramente possível se é inteiro (com qualquer t 0 entre 0 e 90) v 10 e se é a metade de um inteiro ímpar (com qualquer t 0 entre 0 e 15). v Vamos mostrar que esses são os únicos casos possíveis. EUREKA! N°10, 2001

29


Sociedade Brasileira de MatemĂĄtica

10r ĂŠ igual a um inteiro mais um nĂşmero pertencente v

Primeiro mostraremos que

10  2 = j + Îą , com j inteiro e Îą ∈ [0,1). Se a  0,  para algum r0: seja v  5 2 0 < Îą < , tomamos r0 = 1. 5 1 3 1 Se < Îą < ,2Îą = 1 + β , com 0 < β < , e tomamos r0 = 2. 2 5 5 3 Se < Îą < 1, tomamos β = 1 − Îą e k inteiro tal que kβ < 1 < ( k + 1) β . Como 5 2 3 β < , temos kβ > , e podemos tomar r0 = k . 5 5 2 1 3 4 Se ≤ Îą < , temos < ≤ 2Îą < 1, e podemos proceder como no caso anterior, 5 2 5 5 tomando β = 1 − 2Îą e r0 = 2k . Para finalizar, vamos mostrar que, nesses casos, existe k inteiro positivo tal que t0 10 3  + k ĂŠ igual a um inteiro mais um elemento de  ,1. 150 v 5  10 2 De fato, existe m inteiro tal que r0 = m + β , com 0 < β < , e existem e j 5 v t0 t0 inteiros com + β < j≤ + ( + 1) β , donde 150 150 t0 t 10 3 r0 = lm + 0 + β = ( m + j − 1) + Îł , onde < 1 − β < Îł < 1. + 150 v 150 5 20 Assim as possĂ­veis velocidades sĂŁo v = m / s, para cada inteiro positivo k. k

¨

Š

Š

Š

Š

Š

žkÂ&#x;f ¥f¢ Ăś á*ø5— Ăş}ÂŁ  ¢}¤fÂĽ;ÂŚf § Ăśw™ ¤ á ÂĄ Ăś Â&#x; g  £kb ¢ c ø5e øfc Ăś$ÂŁ d]ÂŁ ÂŚf rž ø ¤f¢h ihÂŁ ž j

Vamos observar um caso particular primeiro: Sabemos que: d ( f (0,0,0,...,0), f (1,0,0,...,0)) = 1 e d ( f (1,0,0,...,0), f (2,0,0,0,...,0)) = 1 e d ( f (2,0,0,...,0), f (0,0,0,...,0)) = 1 Seja A = f (0,0,...,0), B = f (1,0,0,...,0) EUREKA! N°10, 2001

30


Sociedade Brasileira de Matemática

e C = f (2,0,0,...,0) A = ( a1 , a 2 ,..., a 2000 ) e B = (b1 , b2 ,..., b2000 ) e C = (c1 , c 2 ,..., c 2000 )

ª

Deve existir um único i1 ∈ e 1 ≤ i1 ≤ 2000, tal que: a i1 ≠ bi1 , vamos provar que i1 ≤ 1000. Deve existir um único i1 ' tal que bi1 ' ≠ ci1 ' e se fosse i1 ≠ i1 ' teríamos que d ( A, C ) = 2, um absurdo, logo i1 = i1 ' Logo temos: A = (..., a i1 ,...) B = (..., bi1 ,...) e como a i1 ≠ bi1 ≠ ci1 ≠ a i1 logo i1 ≤ 1000. Vamos provar que se: x 0 = f (0, a 2 ' , a 3 ' ,..., a 2000 ' ) = ( x1 , x 2 ,..., x 2000 ) então xi1 = a i1 . Suponhamos por absurdo que xi1 ≠ a i1 (por simetria, consideramos xi1 = bi1 ) Se

d ( A; x0 ) = m,

A = f (0,0,0,...,0)

então

d ( B; x 0 ) = m + 1,

pois

B = f (1,0,0,0,0,...,0) e

x0 = f (0, a 2 ' , a 3 ' ,..., a 2000 ' ) mas d ( B, x 0 ) = m − 1 (pois

xi1 = bi1 ) que é um absurdo, logo xi1 = a i1 Analogamente verificamos que se

x1 = f (1, b2 ' , b3 ' ,..., b2000 ' ) = ( y1 , y 2 ,..., y 2000 ) então y i1 = bi1 Vamos generalizar o argumento (nós só fizemos para o 1o. termo): Teorema 1: Seja

At = f (0,0,...,0,0,...) = (a1 , a 2 ,..., a 2000 ) Bt = f (0,0,...,1,0,...) = (b1 , b2 ,..., b2000 ) C t = f (0,0,...,2,0,...,0) = (c1 , c 2 ,..., c 2000 ) onde t ≤ 1000 : e alteramos apenas o t-ésimo termo no domínio. Então se x' = f ( x1 ,..., x 2000 ) = ( y1 , y 2 ,..., y 2000 ) y it = a it se xt = 0 (onde it é posição que muda de At para Bt) y it = bit se xt = 1 y it = cit se xt = 2 Demonstração: Análoga à anterior (basta trocar algumas variáveis e copiar a demonstração acima). EUREKA! N°10, 2001

31


Sociedade Brasileira de Matemática

É claro que i1 , i 2 ,..., i1000 são todos distintos. Na verdade (i1 ,..., i1000 ) é uma permutação de (1,2,...,1000). Consideramos agora as seguintes 2000-uplas. A j = f (0,0,...,0) = (a1 , a 2 ,..., a 2000 ) B j = f (0,0,...,1,0,0,0,...,0) = (b1 ,..., b2000 ) onde j > 1000 e colocamos o 1 na j-ésima posição. Sabemos que d ( A j , B j ) = 1 ⇒ ∃t ∈

«

tal que:

a t ≠ bt e esse t é único! É claro que t > 1000 (pois se fosse t < 1000, existiria w ≤ 1000 tal que i w = t , um absurdo, pois o valor de posição wi da imagem é determinado exclusivamente pelo valor da posição w da 2000-upla do domínio da função t (devido ao teorema 1).

Vamos chamar esse t de i j , assim como fizemos anteriormente. Seja x' = f ( x1 , x j ,..., x 2000 ) = ( y1 ,..., y j ,..., y 2000 ) De forma semelhante à anterior, mostramos que: y i j = a i j se x j = 0 y i j = bi j se x j = 1

Para contar o número de funções f : X → X , basta contar o número de permutações de {1,2,...,1000} vezes o número de permutações de {1001,...,2000} × (3!)1000 × (2!)1000 que é 1000!× 1000!× 121000 pois para determinarmos uma função f : X → X basta escolher: (i1 ,..., i 2000 ) que é uma permutação de (1,2,...,1000) e (i1001 ,..., i 2000 ) que é uma permutação de (1001,...,2000) e escolher os valores apropriados de (a it , bit , c it ), para 1 ≤ t ≤ 1000 (1000 permutações de {0,1,2}) e de (a it , bit ), para 1001 ≤ t ≤ 2000 (1000 permutações de {0, 1}).

EUREKA! N°10, 2001

32


Sociedade Brasileira de MatemĂĄtica

žkÂ&#x;f ¥f¢VÂŹk­ ø Ăš Ăş}ÂŁ  ¢}¤fÂĽ;ÂŚf §fÂŹ l;™ Â&#x; b ÂŁqg;b øfe ÂŽvøhg øfe%ø ÂĄEÂŹ d]b g'øfx ¤ øfÂœqi Â&#x; uvj Plano mediador de dois vĂŠrtices adjacentes (PMVA). Existem 12 arestas, logo sĂŁo 12 pares de vĂŠrtices adjacentes, mas 4 pares possuem o mesmo plano mediador. Portanto sĂŁo 12 : 4 = 3 planos.

Plano mediador de dois vĂŠrtices opostos de uma face (PMVOF).

Plano mediador de dois vĂŠrtices opostos (PMVO).

Repare que todos os planos mediadores juntos determina em cada face a seguinte figura:

PMVO

PMVA PMVOF

Como o centro do cubo ĂŠ interseção de todos os PMs e todas as interseçþes entre retas da figura ao lado sĂŁo extremidades das interseçþes entre PMs, ao ligarmos as interseçþes entre PMs, teremos vĂĄrias pirâmides cujo vĂŠrtice comum ĂŠ o centro do cubo e as bases sĂŁo os triângulos da face. Como sĂŁo 16 Ă— 6 = 96 triângulos no total, o cubo fica dividido em 96 pirâmides.

EUREKA! N°10, 2001

33


Sociedade Brasileira de Matemática

¯E¯E°±°h² ³$°±´{µE¶±·¹¸¹·»º¹¼7·¹½E°±³.¾E°±¼¹·»¸7¾r´ ·¹¿$¾E´{À¹¿$°±Á¹· fÂqÃvÄ}Å ÆSÇqÈhÉ¹Ê ËhÌ ÍkÂvÅ <Nd] Î Â ù Î ÃqÏqÐÒÑ Âvà j NOME Maricy Miki Hisamoto Yuriy Thallickson Bincovsky Guilherme Rohden Echelmeier Gabriel Tavares Bujokas Heitor Silva Lima Lacerda Camila Santos Costa Thomás Yoiti Sasaki Hoshina Júlia Ribeiro Lamardo Augusto Ossamu Shitani Thiago de Paula Garcia Caixeta Vitor Rezende Faria Vitor Humia Fontoura Adriano Jorge Braun Vieira Neto Mariana de Camargo Penteado Luiz Müller Floris Uyttenhove Raul Máximo Alexandrino Nogueira Rudá Moreira de Lima e Silva Fabio Eigi Imada Blandina Lavor Barbosa Bezerra Felipe Sanches Varroni Nicoli Gavassa Rafael Fonseca de Campos Bruna Aguilar Trotta Daniel Folador Rossi Max Douglas Peixoto da Silva Tiago Abreu Tavares de Sousa Paulo André Carvalho de Melo Eduardo Martins de Figueiredo João Marcos Nobuo Umetsu Hansen Gabriel Salvagno Adalberto Delgado Neto Gilberto Marques Arsiolli Felipe Leon Peres Camargo Shalders Martin Alexander Barrios Gundelach Louise Nagashima Omi Édipo Martins Sípoli André Ikeda Cantão Paulo Henrique Macera Lucas de Barros Navarro Ana Paula Seno Pinheiro Eduardo Fischer Germano Bezerra de Menezes Pinho Wagner Silveira Aniceto Gabryel Melo Lutz Diego Frade Bernardes Átila Pereira Ricarte Frederico de Souza Frydman Pedro Thiago Ezequiel de Andrade Hugo Siqueira Robert Pinto Bárbara Gomes Arabe Saraiva Domingos Gomes de Aguiar Neto Fernanda Mary Sonoki Carolina Lisboa Borgo Raphael Rodrigues Mata Gil Henriques Gustavo Schmidt Joau e Silva Anderson Cipriano de Lima Rafael Santos Correia de Araújo Paulo Henrique Gonçalves dos Santos Hanna Kirihara e Silva Luciana Salomão Vilar Thaís Viveiro Raffaello Couto Caser

CIDADE – ESTADO São Paulo – SP Curitiba – PR Itajaí – SC São Paulo – SP Fortaleza – CE Salvador – BA Rio de Janeiro – RJ São Paulo – SP São Paulo – SP Colatina – ES Goiânia – GO Salvador – BA Fortaleza – CE São Paulo – SP Vitória – ES Vitória – ES Fortaleza – CE Unaí – MG S.J. dos Campos – SP Vila Velha – ES São Paulo – SP São Paulo – SP Atibaia – SP Belo Horizonte – MG São Mateus – ES Fortaleza – CE Campina Grande – PB Rio de Janeiro – RJ Vitória – ES Jundiaí – SP Jundiaí – SP Fortaleza – CE Três Lagoas – MS Vitória – ES Rio de Janeiro – RJ São Paulo – SP Curitiba – PR Curitiba – PR S.J. dos Campos – SP Salvador – BA Ourinhos – SP Encantado – RS Fortaleza – CE Campo Grande – MS Goiânia – GO Fortaleza – CE Fortaleza – CE Salvador – BA Fortaleza – CE Fortaleza – CE Santos – SP Fortaleza – CE São Paulo – SP Montanha – ES Salvador – BA Vassouras – RJ Juiz de Fora – MG Jaboatão dos G.– PE Salvador – BA São Paulo – SP Florianópolis – SC São Carlos – SP São Paulo – SP Vitória – ES

EUREKA! N°10, 2001

34

PRÊMIO Ouro Ouro Ouro Ouro Ouro Prata Prata Prata Prata Prata Prata Prata Prata Prata Prata Bronze Bronze Bronze Bronze Bronze Bronze Bronze Bronze Bronze Bronze Bronze Bronze Bronze Bronze Bronze Bronze Bronze Bronze Bronze Menção Honrosa Menção Honrosa Menção Honrosa Menção Honrosa Menção Honrosa Menção Honrosa Menção Honrosa Menção Honrosa Menção Honrosa Menção Honrosa Menção Honrosa Menção Honrosa Menção Honrosa Menção Honrosa Menção Honrosa Menção Honrosa Menção Honrosa Menção Honrosa Menção Honrosa Menção Honrosa Menção Honrosa Menção Honrosa Menção Honrosa Menção Honrosa Menção Honrosa Menção Honrosa Menção Honrosa Menção Honrosa Menção Honrosa Menção Honrosa


Sociedade Brasileira de Matemática

¯E¯E°-°0² ³$°±´{µE¶±·¹¸7·»º¹¼¹·¹½E°±³$¾E°±¼¹·»¸¹¾C´{·¹¿$¾E´ À¹¿$°-Á7· O ÓkÔ$ÕNÖ × Ø$ÙEÚ X ÛNÜ Ý$Ó7Öqo5Þ ß à áâÓ ã à áâÔ$ä$å*æ Ó$Ô.ç NOME Alex Corrêa Abreu Fabio Dias Moreira Larissa Cavalcante Queiroz de Lima Rafael Daigo Hirama Samuel Barbosa Feitosa Jorge Peixoto de Morais Neto Israel Franklim Dourado Carrah Davi Máximo Alexandrino Nogueira Vitor Gabriel Kleine Juliana Gomes Varela Ayran Ayres Barbosa Loriato Daniel Haanwickel Junqueira Raquel Lamboglia Guimarães Paulo Roberto Sampaio Santiago Telmo Luis Correa Jr. Adalberto Studart Neto João Marcos da Cunha Silva Thiago Costa Leite Santos Kiyoshi Horie Filho Antônio Monteiro Guimarães Jr. Adriano Brasileiro Silva Renato Mendes Coutinho Diogo dos Santos Suyama Gustavo Ferruzzi Martucci Henry Wei Cheng Hsu Thiago Pinheiro Faury Otacílio Torres Vilas Boas Vitor Sarmento Mesquita Felipe Netto de Santana Patricia Akemi Komura Regiane Cristina Yamanari Guilherme Honda Saito Milton Eiji Kato Vinicius Antonio Batagello Thiago Mizuta Daniel Costa Garcia Henrique Castro Noronha Raphael Henrique Ribas Dafne de Albuquerque Simão Michel Renato Manzolli Ballestero Fernanda Ramos Correia Fernando Santos Simões Ferreira Lucas Lolli Saui Luciano Lacerda Silveira Samara Anny Maia Fava Gustavo Eufrásio Farias Vinicius Figueiredo de Castro Luís Eduardo de Godoi Helder Seiji Kato Márcio Jun Hisamoto Vinicius Augusto Paccola Erika Famini Silva

CIDADE – ESTADO Niterói – RJ Rio de Janeiro – RJ Fortaleza – CE Campinas – SP Fortaleza – CE Goiânia – GO Fortaleza – CE Fortaleza – CE Mogi das Cruzes – SP Fortaleza – CE Vitória – ES Salvador – BA Fortaleza – CE Salvador – BA Santo André – SP Fortaleza – CE Fortaleza – CE São Paulo – SP Ourinhos – SP Campina Grande – PB Fortaleza – CE Americana – SP Belo Horizonte – MG Piracicaba – SP São Paulo – SP São Paulo – SP Salvador – BA Fortaleza – CE Rio de Janeiro – RJ São Paulo – SP Guararapes – SP São Paulo – SP São Paulo – SP Araçatuba – SP São Paulo – SP Goiânia – GO Valinhos – SP Curitiba – PR Fortaleza – CE Araraquara – SP Salvador – BA Vitória – ES Florianópolis – SC Campo Grande – MS Fortaleza – CE Fortaleza – CE Rio de Janeiro – RJ S.J. dos Campos – SP São Paulo – SP São Paulo – SP Matão – SP Salvador – BA

EUREKA! N°10, 2001

35

PRÊMIO Ouro Ouro Ouro Ouro Prata Prata Prata Prata Prata Prata Prata Prata Prata Prata Prata Bronze Bronze Bronze Bronze Bronze Bronze Bronze Bronze Bronze Bronze Bronze Bronze Bronze Bronze Menção Honrosa Menção Honrosa Menção Honrosa Menção Honrosa Menção Honrosa Menção Honrosa Menção Honrosa Menção Honrosa Menção Honrosa Menção Honrosa Menção Honrosa Menção Honrosa Menção Honrosa Menção Honrosa Menção Honrosa Menção Honrosa Menção Honrosa Menção Honrosa Menção Honrosa Menção Honrosa Menção Honrosa Menção Honrosa Menção Honrosa


Sociedade Brasileira de Matemática

¯E¯E°-°0² è$°±é{µE¶±ê¹¸7껺¹¼¹ê¹½E°±è$¾E°±¼¹ê»¸¹¾Cé{깿$¾Eé À¹¿$°-Á7ê ë ÓkÔ$ÕNÖ × Ø$ÙEÚ ì ÛNÜ Ý$Ó7ÖqíîÞðï;ÛfÔ æ ÛEÚ ñ`ä$Ùòæ ÚEç NOME Ulisses Medeiros de Albuquerque Sérgio Tadao Martins Daniel Massaki Yamamoto Humberto Silva Naves Carlos Stein Naves de Brito Lucas Heitzmann Gabrielli Daniel Mourão Martins Leonardo Augusto Zão Fabrício Siqueira Benevides Christian Lyoiti Watanabe Rodrigo Roque Dias Rui Facundo Vigelis Thiago Barros Rodrigues Costa Ronaldo Ikaro Farias Araújo Daniel Nobuo Uno Daniel Pinheiro Sobreira João Alfredo Castellani Fajardo Freire Daniel Pessoa Martins Cunha Guilherme Fujiwara Gilberto Kirk Rodrigues Artur Duarte Nehmi Rodrigo Villard Milet Eduardo Moraes de Morais Carlos Sartori Ferreira Filho Thiago da Silva Sobral Silvano José Gomes Flumignan Mateus Ymanaka Barretto Hugo Pinto Iwata Paulo Ribeiro de Almeida Neto Diêgo Veloso Uchôa Artur Radoman de Oliveira Yuri Gomes Lima Rafael Tajra Fonteles Maurício de Carvalho Paiva Bernardo Freitas Paulo da Costa Eduardo Barbosa Araújo Antônio Davi Macêdo de Castro Eduardo Famini Silva Arnaldo João do Nascimento Júnior Augusto Quadros Teixeira Einstein do Nascimento Jr. Marcos Soares de Souza Caio Augusto P. del Bianco Licciardi Luiz Antonio Felinto Cruz Diego Alvarez Araujo Correia Afonso de Paula P. Rocha Tibério Bittencourt de Oliveira

CIDADE – ESTADO Fortaleza – CE São Paulo – SP São Paulo – SP São Paulo – SP Goiânia – GO São Paulo – SP Fortaleza – CE Nilópolis – RJ Fortaleza – CE Itaguaí – RJ São Paulo – SP Fortaleza – CE Fortaleza – CE Fortaleza – CE São Paulo – SP Fortaleza – CE Salvador – BA Fortaleza – CE São Paulo – SP Rio de Janeiro – RJ São Paulo – SP Rio de Janeiro – RJ São Paulo – SP Rio de Janeiro – RJ Fortaleza – CE P. Prudente – SP São Paulo – SP S.J. do Rio Preto – SP Ananindeua – PA Teresina – PI Rio de Janeiro – RJ Fortaleza – CE Teresina – PI Belém – PA Rio de Janeiro – RJ Fortaleza – CE Fortaleza – CE Salvador – BA Duque de Caxias – RJ Belo Horizonte – MG Fortaleza – CE Rio de Janeiro – RJ Atibaia – SP Fortaleza – CE Fortaleza – CE Fortaleza – CE Goiânia - GO

EUREKA! N°10, 2001

36

PRÊMIO Ouro Ouro Ouro Ouro Ouro Prata Prata Prata Prata Prata Prata Prata Prata Prata Prata Prata Bronze Bronze Bronze Bronze Bronze Bronze Bronze Bronze Bronze Bronze Bronze Bronze Bronze Bronze Bronze Menção Honrosa Menção Honrosa Menção Honrosa Menção Honrosa Menção Honrosa Menção Honrosa Menção Honrosa Menção Honrosa Menção Honrosa Menção Honrosa Menção Honrosa Menção Honrosa Menção Honrosa Menção Honrosa Menção Honrosa Mençao Honrosa


Sociedade Brasileira de MatemĂĄtica

óôÜþ á5ø Ú ú óôßÝ ú ú ý Þ5Ý óÿþ ý Ý ô

!" # $ % & ' () !*$ ♌ +-, .0/ 1 2 3 4 / 576 / 8 9 : 5 9 ; < No texto a seguir fazemos uma breve introdução ao conceito de nĂşmero irracional. Na sua maior parte o texto serĂĄ acessĂ­vel a alunos da Ăşltima sĂŠrie do primeiro grau. As duas Ăşltimas seçþes talvez requeiram um pouco mais de maturidade embora nĂŁo exijam nenhum conhecimento prĂŠvio adicional. Para simplificar a exposição nos restringiremos a nĂşmeros positivos. A extensĂŁo dos fatos abordados ao contexto geral de nĂşmeros positivos, negativos e 0 nĂŁo requer nenhuma dificuldade adicional. Pode-se imaginar que a idĂŠia de nĂşmero inteiro positivo tenha surgido num estĂĄgio primĂĄrio da civilização, juntamente com a necessidade da prĂĄtica da contagem. Por exemplo, era necessĂĄrio a um pastor saber contar de algum modo o nĂşmero de animais no seu rebanho. A maneira de representar o resultado dessa contagem era no inĂ­cio bastante diferente da que usamos agora e ĂŠ provĂĄvel que no começo cada pessoa tivesse sua maneira prĂłpria de fazĂŞ-lo. Contar significa estabelecer um modo de comparar quantidades de elementos de conjuntos distintos. Por exemplo, a quantidade de pedrinhas em um saco com a quantidade de animais num rebanho, ou a quantidade de alimentos conseguidos em uma caçada ou em colheita com a quantidade de membros da tribo. TambĂŠm nĂŁo ĂŠ difĂ­cil imaginar que a ideia de fração tenha surgido na evolução da civilização humana, primeiramente e de forma mais elementar, com a ocorrĂŞncia usual da necessidade de um determinado grupo de pessoas partilhar um ou mais bens de propriedade comum entre seus membros. E num estĂĄgio mais avançado, dentre outras motivaçþes possĂ­veis, com a necessidade de as pessoas trocarem entre si bens de tipos distintos. Por exemplo, um pastor deseja trocar com um agricultor peles de carneiro por sacos de milho numa razĂŁo de 3 peles de carneiro para cada grupo de 7 sacos de milho. Por outro lado, a idĂŠia de um “nĂşmeroâ€? que nĂŁo seja nem inteiro nem fração ĂŠ, em princĂ­pio, muito menos natural que a daqueles e surge num estĂĄgio muito mais avançado da civilização com a necessidade da prĂĄtica da medição. Por exemplo, medir as dimensĂľes ou a ĂĄrea de um terreno, comparar as distâncias entre pares de pontos distintos, etc. Procuraremos, a seguir, mostrar as propriedades bĂĄsicas destes nĂşmeros “estranhosâ€? em contraste com as propriedades, na maior parte jĂĄ bem conhecidas, daqueles mais intuitivos, os inteiros e as fraçþes.

EUREKA! N°10, 2001

37


Sociedade Brasileira de MatemĂĄtica

= >'? @ A)BDC"BFE G HI@ J K C L M G HN@"A Os nĂşmeros reais positivos podem ser representados no sistema decimal por uma seqßência de algarismos – elementos do conjunto {0, 1, 2, 3, 4, 5, 6, 7, 8, 9} – Separados por uma vĂ­rgula. Assim, se a N , a N −1 ,..., a 0 , a −1 , a − 2 , a −3 ,..., sĂŁo algarismos quaisquer, um nĂşmero real positivo representado no sistema decimal tem a forma a N a N −1 a N − 2 ...a1 a 0 , a −1 a − 2 a −3 ..., (1) onde a N > 0. Nessa representação, Ă esquerda da vĂ­rgula temos sempre um nĂşmero finito de algarismos, porĂŠm Ă direita podemos ter uma infinidade de algarismos. Por exemplo, 783,5231 representa o nĂşmero obtido como resultado da expressĂŁo (2) 7 Ă— 10 2 + 8 Ă— 10 1 + 3 Ă— 10 0 + 5 Ă— 10 −1 + 2 Ă— 10 −2 + 3 Ă— 10 −3 + 1 Ă— 10 −4 . Por outro lado, a fração 154 tem representação decimal 0, 1545454‌ com uma 999

infinidade de algarismos Ă direita. Essa representação se traduz como resultado de uma expressĂŁo com infinitas parcelas (3) 1 Ă— 10 −1 + 5 Ă— 10 −2 + 4 Ă— 10 −3 + 5 Ă— 10 −4 + 4 Ă— 10 −5 + 5 Ă— 10 −6 + ... 154 Essa expressĂŁo significa exatamente que se quisermos aproximar no sistema 999

decimal com “precisĂŁo de 8 casas decimais, por exemplo, devemos tomar como aproximação o nĂşmero 0,15454545 que ĂŠ resultado da expressĂŁo

1 Ă— 10 −1 + 5 Ă— 10 −2 + 4 Ă— 10 −3 + 5 Ă— 10 −4 + 4 Ă— 10 −5 + 5 Ă— 10 −6 + 4 Ă— 10 −7 + 5 Ă— 10 −8.

(4) Claro, o nĂşmero 0, 1545454‌ ĂŠ o que chamamos de uma dĂ­zima periĂłdica e por isso pode ser obtido como uma fração 154 . 999

OQPSR BT@ E OSU V BFE"B U O E"@ A O C B R HI@WC L M G HN@ U X OZY7[ BF\ G^]SC G E @ _ Neste caso, assim como no periódico, temos uma infinidade de algarismos à direita da vírgula e assim só nos Ê possível escrever a representação decimal atÊ uma certa casa decimal, porÊm, diferentemente do que acontece no caso periódico, não hå repetição indefinidamente de um determinado grupo de algarismos e, assim, o número em questão não pode ser obtido como uma fração p com e e q diferente de 0. Os números que podem ser obtidos como fraçþes são q

chamados racionais; os que não podem ser obtidos como fraçþes são chamados irracionais. EUREKA! N°10, 2001

38


Sociedade Brasileira de MatemĂĄtica

` > [FO \ P R B [ \"BFE G^AF@ H O A C O A U a HbBF\ O A G \"\ @ E G O U @ G^AF_ Responderemos esta pregunta atravÊs de um exemplo. Euclides provou que o número positivo cujo quadrado Ê 2, isto Ê, o número positivo x que satisfaz a x 2 = 2, (5) equação não Ê racional. Euclides argumentou da seguinte forma: Suponhamos que o número x satisfazendo (5) seja racional. Então existem inteiros positivos p e q, 2 2 2 primos entre si, tais que p 2 = 2. ou seja p = 2q . (6) q 2 Portanto p Ê par e p tambÊm Ê par; p pode ser escrito na forma p = 2k. Assim,

(2k ) 2 = 2q 2 ⇔ 2k 2 = q 2 ⋅

(7)

Pela mesma razão que acabamos de expor, concluímos que q tambÊm deve ser par. Mas isto nos leva a uma contradição pois p e q são primos entre si por hipótese! Assim, a suposição de que x = p nos leva a uma contradição e, q portanto, deve ser descartada, considerada falsa. Chegamos à conclusão que 2 , que Ê como representamos o número positivo cujo quadrado Ê 2, Ê um número irracional!!

c >'E O H OQO ? V B)\d@ [ \ Ofe G HI@ g h B)AD\ @"E G OSU @ G^A [ @ \"@

2

Podemos obter aproximaçþes cada vez melhores de 2 (o nĂşmero x que satisfaz (5)) atravĂŠs do seguinte procedimento que ĂŠ um caso particular de um esquema inventado por Newton conhecido como mĂŠtodo de Newton. (Com base nesse mĂŠtodo podemos programar as mĂĄquinas de calcular para produzirem aproximaçþes de 2 tĂŁo precisas quanto o avanço da eletrĂ´nica nos permitir). primeiro “chutamosâ€? um nĂşmero x0 como uma primeira aproximação de x que nos pareça razoĂĄvel; por exemplo, x0 = 1. Em seguida observamos que

x 2 − x02 = ( x + x0 )( x − x0 ) ≅ 2 x0 ( x − x0 ), onde o sĂ­mbolo ≅ significa â€œĂŠ aproximadamente igual aâ€?. Assim,

x 2 − x02 ≅ 2 x0 ( x − x0 ), EUREKA! N°10, 2001

39


Sociedade Brasileira de MatemĂĄtica

e, portanto, dividindo a “equação aproximadaâ€? por 2x 0 e arranjando os termos, obtemos

x 2 − x02 x≅ + x0 . 2 x0 2 substituindo x = 2 e x0 = 1 em (8), obtemos x ≅

(8)

2 −1 3 +1= . 2 2

3 Assim temos uma segunda aproximação x1 = . Encontramos tambĂŠm x 2 : 2 x2 ≅

9 4 + 3 ⇔ x ≅ − 1 ⋅ 1 + 3 ⇔ x ≅ − 1 + 3 ⇔ x 2 ≅ 17 . Da mesma 2 2 3 2 4 3 2 12 2 12

2−

forma, podemos obter uma quarta aproximação x3 , fazendo x3 =

x 2 − x 22 2 − (17 / 12) 2 17 288 − 289 17 288 − 289 + 2 Ă— 289 577 = + = = . + x2 = + 2 Ă— 12 Ă— 17 408 2 x2 17 / 6 12 2 Ă— 12 Ă— 17 12

Assim, x3 = 577 seria a aproximação seguinte: Sua representação decimal Ê a 408

dĂ­zima periĂłdica x3 = 1,4142156862745 m n n n p n 098039 m k k l i i ...j 9.... n n o 21568627 perĂ­odo

Agora se você pegar uma måquina de calcular e pedir (atravÊs dos devidos comandos) que ela calcule 2 , você obterå, se sua måquina puder exibir 33 dígitos (incluindo a vírgula ou ponto), a expressão decimal 1,4142135623730950488016887242097. Horrível, não Ê? Você obterå uma expressão ainda maior se sua måquina puder exibir mais dígitos. Repare como nossas aproximaçþes x1 , x 2 e x3 estão cada vez mais próximas desse número!

q > O A U a HbBF\ O AD\"@ E G OSU @ G A [FO C"BFHrAFBF\dB U"R HIB)\ @ C O A Isto significa que podemos dispor os nĂşmeros racionais numa sucessĂŁo da forma r1 , r2 , r3 ,..., com uma infinidade de elementos. Podemos interpretar este fato como significando que a quantidade de nĂşmeros racionais, embora sendo infinita, ĂŠ de uma “ordem de infinitudeâ€? equivalente a dos nĂşmeros naturais 1, 2, 3‌. O argumento para a demonstração desse fato ĂŠ devido a Georg Cantor. EUREKA! N°10, 2001

40


Sociedade Brasileira de MatemĂĄtica

Como todo racional tem uma representação única como fração

p q

com p e q

inteiros positivos primos entre si, basta que saibamos enumerar os pares ordenados (p, q) de naturais primos entre si. A forma de obter essa enumeração estå descrita pela figura abaixo:

A enumeração Ê obtida seguindo-se o caminho indicado pelas flechas, iniciando a partir de (1,1), tendo o cuidado de descartar os pares de naturais que não são primos entre si, como, por exemplo, (2,2), (4,2), (3,3) etc.. Com isso, teríamos r1 =

3 1 1 1 2 = 1, r2 = , r3 = = 2, r4 = = 3, r5 = , etc. 1 2 1 3 1

s >'\ B [ \ B)A)B U V @ g X O C"BFE G HN@ J-C O AD\"@ E G O U @ G A Hå pouco dissemos que não era possível pôr uma dízima não periódica em forma de fração p com p e q naturais primos entre si. Vamos dar uma explicação para q

este fato. Fixemos um natural q. Quando dividimos um número qualquer N > q pelo número q. Obtemos como resto da divisão um elemento do conjunto (finito) {0, 1, 2,‌, q – 1}. Tomemos como exemplo q = 7 e N = 17; nesse caso os restos possíveis pertencem ao conjunto {0, 1, 2, 3, 4, 5, 6}. Agora vamos recordar o algoritmo da divisão com esse exemplo específico:

EUREKA! N°10, 2001

41


Sociedade Brasileira de MatemĂĄtica

t u u t v w xyvSw-zZ{ZuZtfv|w}zZ{Zu t#~ f€ w}z wZ€ tfv  f€ {S v € { {F€ vƒ‚ t)€ u „DÂ… O que acontece ĂŠ que os restos possĂ­veis sĂŁo elementos do conjunto finito de q elementos {0, 1,‌, q – 1}(no exemplo acima q = 7). Assim, em no mĂĄximo q iteraçþes do algoritmo ou acabamos repetindo um elemento do conjunto de restos possĂ­veis (no exemplo acima o primeiro a se repetir foi o 3), ou o 0 ocorre como resto e o processo termina. No primeiro caso, a partir daĂ­ passamos a repetir os restos ocorridos anteriormente na mesma ordem (3, 2, 6, 4, 5, 1, no exemplo acima). As casas decimais no quociente por sua vez tambĂŠm se repetem o obtemos entĂŁo uma dĂ­zima periĂłdica. No segundo caso, obtemos simplesmente um nĂşmero finito de casas decimais.

† >'\ B [ \ B)A)B U V @ g X O C"BFE G HN@ J-C O ADG \"\ @ E G OSU @ G^A Todo nĂşmero irracional positivo possui uma representação decimal Ăşnica por meio de uma dĂ­zima nĂŁo periĂłdica. Para simplificar vamos nos restringir aos nĂşmeros entre 0 e 1. JĂĄ sabemos que um nĂşmero cuja representação decimal possui uma quantidade finita de casas decimais pertence ao conjunto dos racionais. Da mesma forma aprendemos que um nĂşmero cuja representação decimal ĂŠ uma dĂ­zima periĂłdica ĂŠ tambĂŠm um nĂşmero racional. Por outro lado, vimos no item anterior que as representaçþes decimais de um racional sĂŁo necessariamente de um dos dois tipos: ou possuem uma quantidade finita de casas decimais, ou “terminamâ€? em uma dĂ­zima periĂłdica. Logo, uma representação decimal para um nĂşmero irracional tem necessariamente que ser uma dĂ­zima nĂŁoperiĂłdica. Afirmamos que essa representação ĂŠ Ăşnica. Repare que isso nĂŁo ocorre em geral com os racionais. Por exemplo, 0, 21 e 0, 20999‌ representam o mesmo racional 21 . Suponhamos que um irracional x entre 0 e 1 possua duas 100

representaçþes decimais distintas: EUREKA! N°10, 2001

42


Sociedade Brasileira de MatemĂĄtica

x = 0, a −1 a − 2 a −3 ...,

(10)

x = 0, b−1b− 2 b−3 ...,

(11)

Se essas representaçþes sĂŁo distintas certamente existe um p ∈ ‡

tal que

a − k = b− k , para k = 0,..., p − 1, e a − p ≠b− p . Para fixar idĂŠias vamos assumir entĂŁo que a − p ≼ b− p + 1 e por (10) e (11)

x ≼ 0, a −1 a − 2 ...a − p , x ≤ 0, a −1 a − 2 ...b− p 999... = 0, a −1 a − 2 ...(b− p + 1),

(12) (13)

jĂĄ que b− k = a − k se k = 0,..., p − 1 e b− k ĂŠ sempre menor ou igual a 9. Mas (12) e (13) implicam que a − p = b− p + 1 e x = 0, a −1 a − 2 ...a − p . PorĂŠm nesse caso x ĂŠ racional e chegamos a uma contradição! ChegarĂ­amos a uma contradição semelhante tambĂŠm se tivĂŠssemos assumido b− p > a − p , argumentando da mesma forma apenas trocando os papĂŠis dos a − k e b− k . A contridição tem origem no fato de termos suposto que havia duas representaçþes decimais distintas para o mesmo irracional x. Logo essa possibilidade tem que ser descartada, considerada falsa, e assim concluĂ­mos que todo irracional possui uma representação decima Ăşnica como dĂ­zima nĂŁo-periĂłdica.

A G \ \"@ E G O U @ G A U X O‰[FO C B)HŠA)BF\ B "U R I H BF\"@ C O A ˆ>O Isto significa que nĂŁo podemos dispor os nĂşmeros irracionais numa sucessĂŁo s1 , s 2 , s3 ,..., mesmo admitindo uma infinidade de elementos. Quer dizer, diferentemente dos racionais, a “ordem de infinitudeâ€? da quantidade dos nĂşmeros irracionais ĂŠ maior que a dos nĂşmeros naturais. ConcluĂ­mos daĂ­ que existem muito mais nĂşmeros irracionais do que racionais! Vamos tentar justificar nossa afirmação sobre a nĂŁo-enumerabilidade dos irracionais. O argumento ĂŠ uma adaptação de uma idĂŠia tambĂŠm devida a G. Cantor. Suponhamos que fosse possĂ­vel dispor os irracionais numa sucessĂŁo s1 , s 2 , s3 ,..., . Basta considerarmos apenas os irracionais entre 0 e 1. Criamos um nĂşmero irracional x, tambĂŠm entre 0 e 1, atravĂŠs de uma representação decimal (portanto, uma dĂ­zima nĂŁo periĂłdica) da seguinte forma. O nĂşmero x tem representação decimal dada por x = 0, x −1 x − 2 x −3 ... onde x − p ĂŠ escolhido dentro do conjunto {0, 1, ‌, 9} de modo que x − p ĂŠ diferente de ( s p ) − p onde este Ăşltimo ĂŠ o algarismo que aparece na casa decimal de ordem p do irracional s p (p-ĂŠsima EUREKA! N°10, 2001

43


Sociedade Brasileira de MatemĂĄtica

elemento da sucessĂŁo s1 , s 2 ,...s p ,...). A escolha de cada x p tambĂŠm deve atender a condição de nĂŁo permitir que nenhum grupo de algarismos dentre os jĂĄ escolhidos x −1 , x − 2 ,..., x − ( p −1) possa se tornar o gerador de uma dĂ­zima periĂłdica. Desta forma obtemos uma dĂ­zima nĂŁo periĂłdica representando um Ăşnico irracional que, no entanto, nĂŁo pode constar na lista s1 , s 2 , s3 ,..., . De fato, se

‹

x = s r , para algum r ∈ , entĂŁo como x − r ≠( s r ) − r terĂ­amos um absurdo (uma contradição)!.

Œ >'BFA V R C O A R [ J BFHIB U V @"\ � O G \ \"@ E G O U @ J

Ď€

O nĂşmero Ď€ ĂŠ definido como sendo a ĂĄrea limitada por um cĂ­rculo de raio 1. Ele ĂŠ certamente o irracional transcendente mais conhecido. A expressĂŁo transcendente significa, neste contexto, um nĂşmero irracional que nĂŁo ĂŠ raiz de nenhuma equação polinomial com coeficientes inteiros. Por exemplo, os irracionais 2 ,1 + 3 nĂŁo sĂŁo transcendentes pois sĂŁo raĂ­zes das equaçþes polinomiais

x 2 = 2, x 2 − 2 x − 2 = 0, respectivamente. Neste Ăşltimo caso dizemos que os nĂşmeros sĂŁo algĂŠbricos. A demonstração de que Ď€ ĂŠ um nĂşmero irracional, apesar de nĂŁo ser trivial, pode ser feita usando-se apenas o cĂĄlculo diferencial elementar que ĂŠ ensinado no primeiro perĂ­odo dos cursos de ciĂŞncias exatas. A primeira demonstração de que Ď€ ĂŠ irracional sĂł foi obtida em 1766 por J. H. Lambert, de forma nĂŁo completamente rigorosa, tendo sido finalmente (re)obtida de modo rigoroso pelo famoso matemĂĄtico A. M. Legendre e publicada em 1855. A prova de que Ď€ ĂŠ transcendente ĂŠ muito mais complexa e sĂł foi obtida em 1882 por F. Lindermann. O fabuloso matemĂĄtica grego Arquimedes foi o primeiro a obter uma aproximação razoĂĄvel de Ď€ por numeros racionais. Ele provou que 10 1 3+ <Ď€ < 3+ , 71 7 usando dois polĂ­gonos regulares de 96 lados, um inscrito e outro circunscrito a um cĂ­rculo de raio 1. Podemos obter aproximaçþes cada vez melhores de Ď€, com o auxĂ­lio de uma mĂĄquina de calcular bastante rudimentar, capaz apenas de fazer as operaçþes bĂĄsicas (+, –, â‹… ) e mais a operação de extrair raiz quadrada, da seguinte forma. A idĂŠia ĂŠ aproximarmos o cĂ­rculo de raio 1 por polĂ­gonos regulares de 2 n lados inscritos neste cĂ­rculo. Primeiramente, ĂŠ fĂĄcil verificar que para a ĂĄrea e o perĂ­metro do polĂ­gono regular de 2 n lados inscritos num cĂ­rculo de raio 1 temos EUREKA! N°10, 2001

44


Sociedade Brasileira de Matemática

1 Perímetro × 4 − l 2 , 4 onde l é o comprimento do lado do polígono. Como l se aproxima mais e mais de 0 a medida que n cresce, vemos que para o círculo de raio 1 devemos ter (fazendo l = 0 na fórmula acima) Área = 1 Perímetro Área =

4

Assim, podemos também definir π como sendo a metade do perímetro do círculo de raio 1. Por outro lado, usando o teorema de Pitágoras que diz que em um triângulo retângulo o quadrado da hipotenusa é a soma dos quadrados dos catetos, se l n denota o comprimento do lado do polígono regular de 2 n lados, é fácil mostrar que l n +1 =

2−

4 − l n2 .

(14)

Para n = 2 temos o polígono regular de 4 lados, quadrado, inscrito no círculo de raio 1, cujo lado, facilmente obtido usando-se o teorema de Pitágoras, é l2 = 2. Por meio de (14) obtemos sucessivamente l3 = 2 − 2 , l4 = 2 − 2 + 2 , l5 = 2 − 2 + 2 + 2 , l6 = 2 − 2 + 2 + 2 + 2 , l7 =

l8 =

2−

2+

2+

2+

2+

2 ,

2− 2+ 2+ 2+ 2+ 2+ 2 ,

Para obter uma boa aproximação de π calculemos, por exemplo, o valor da 8 metade do perímetro do polígono de 2 = 256 lados, inscrito no círculo de raio 1, cujo lado tem comprimento igual a l8 . Podemos obter um valor aproximado para l8 executando a seguinte seqüência de operações numa calculadora EUREKA! N°10, 2001

45


Sociedade Brasileira de MatemĂĄtica

2 sqrt + 2 = sqrt + 2 = sqrt + 2 = sqrt + 2 = sqrt + 2 = sqrt + / – + 2 = sqrt, e obtemos

l8 = 0.0245430765714398521588165239020064. Agora, multiplicaremos o resultado obtido para l8 por 256, que ĂŠ o nĂşmero de lados da polĂ­gono em questĂŁo, e em seguida dividimos por 2 o que nos dĂĄ

Ď€~ − 3.14151380114430107632851505945682 o que fornece uma aproximação com erro menor que 0, 0001 jĂĄ que ĂŠ sabido que 3, 1415 < Ď€ < 3, 1416.

� �)‘)’ “•”)– —™˜ š ›)– ’�œ “ ‘)˜ “ ’

: Exceto pelas duas Ăşltimas seçþes, o texto acima foi elaborado a partir de um “pedidoâ€? de minha filha, Marina, atualmente na 8a. sĂŠrie do primeiro grau, urgida por um trabalho de casa em grupo passado por sua professora. O referido trabalho, felizmente, resultou bastante diferente do que foi exposto acima, que acabou servindo apenas como uma entre vĂĄrias referĂŞncias usadas pelo grupo. No entanto, as 7 primeiras seçþes foram bem compreendidas por ela e seu grupo; as duas Ăşltimas foram escritas depois que o prazo para a entrega do trabalho havia esgotado e, portanto, nĂŁo chegaram a ser “testadasâ€?. Para concluir gostaria de deixar aqui meus agradecimentos ao estimado professor e colega Elon Lages Lima pelas sugestĂľes sobre uma versĂŁo preliminar destas notas.

B e BF\ E L^E G O AFž 1) Usando o mesmo argumento de Euclides descrito em 2. prove que

3, 5 e 7 são irracionais. 2) Usando o mÊtodo de Newton, descrito em 3, obtenha aproximaçþes correspondentes ao x3 do texto para os irracionais 3 , 5 , 7 e compare com o resultado fornecido pela måquina de calcular. 3) Pesquise sobre a vida e a obra dos grandes matemåticos mencionados no texto: Arquimedes, Pitågoras, Euclides, Isaac Newton e Georg Cantor. 4) Prove a fórmula (14). EUREKA! N°10, 2001

46


Sociedade Brasileira de MatemĂĄtica

ó Â&#x; ÝÒø¢¥DÂŁ ý¼¤ ýô Ă˝ Ăł Ăş ĂšD¤ óúŒ¤ Ăł ø¢§ Ăľd¤ Ăł Š¨

O comitĂŞ editorial de ÂŞ*ÂŤ-ÂŹSÂŞ*­"ÂŽ ! agradece aos inĂşmeros elogios recebidos pela criação desta seção bem como ao crescente nĂşmero de leitores que nos enviam soluçþes para os problemas da mesma. A realização de inĂşmeras olimpĂ­adas nesta ĂŠpoca do ano torna mais importante a preparação para as mesmas e por isto resolvemos apresentar neste nĂşmero somente novos problemas acompanhados dos nomes dos leitores que nos enviaram soluçþes de problemas anteriores. No prĂłximo nĂşmero de ÂŞ*ÂŤ}ÂŹ ÂŞ*­"ÂŽ ! voltaremos ao normal apresentando problemas e soluçþes. Continuamos salientando que estamos Ă disposição na OBM para aqueles que estiverem interessados na solução deÂľ algum problema Âś Âľ Âľ particular. Âľ Para tanto, basta contactar a OBM, seção ÂŻb°)Âą ² Âł"´ ¯¸¡ š ÂŻÂƒ¡ ¯º² Âť Âź ÂŻ , atravĂŠs de carta ou e-mail.

ĂŞ 7ĂŞ ĂŞ

ĂŞ

‘f½ Â?)‘)“ Â?

°)ž

“ Âż Âś ˜ ‘f½ Â?)Â’

Primeiramente vamos aos problemas propostos deste nĂşmero 61.

\ Ă Ă‚ Ă‚'Ăƒ Ă„ Y ` Ă…'Ă…'Ă… Æ

Ă€

Sejam a, b, c números reais tais que as equaçþes x 2 + ax + 1 = 0 e x 2 + bx + c = 0 possuem exatamente uma raiz real comum e as equaçþes x 2 + x + a = 0 e x 2 + cx + b = 0 tambÊm possuem exatamente uma raiz comum. Determine a soma a + b + c .

\ Ă Ă‚ Ă‚'Ăƒ Ă„ Y ` Ă…'Ă…'Ă… Æ Ă€

62.

2 3 1000 Determine a soma  1  +  2  +  2  +  2  + â‹… â‹… â‹… +  2  3 3 3 3  3 













onde, como usual, x  Ê o maior inteiro que não supera x .

\ Ă Ă‚ Ă‚'Ăƒ Ă„ Y ` Ă…'Ă…'Ă… Æ Ă€

63.

Sejam a e b números reais não nulos que satisfazem à equação

(

) (

a 2b 2 a 2b 2 + 4 = 2 a 6 + b 6

)

Mostre que pelo menos um deles nĂŁo ĂŠ racional. 64.

Ă€

\ Ă Ă‚ Ă‚'Ăƒ Ă„ Y ` Ă…'Ă…'Ă… Æ

Seja M o conjunto que consiste dos 2000 nĂşmeros 11, 101, 1001, ... . Mostre que pelo menos 99% dos elementos de M nĂŁo sĂŁo primos.

EUREKA! N°10, 2001

47


Sociedade Brasileira de MatemĂĄtica

65.

66.

67.

\ Ă Ă‚ Ă‚'Ăƒ Ă„ Y ` Ă…'Ă…'Ă… Æ

Ă€

Sejam ABCD um paralelogramo com ∠A = 60Âş e O o circuncentro do triângulo ABD . A reta AO intersecta a bissetriz externa do ângulo ∠C do paralelogramo em K . Determine AO OK .

BFĂˆĂŠĂ‰ĂŒĂ‹ Ă? ĂŽ Ă? Y `'Ă?'Ă? Ă?'Ă‘

Ç

Sejam ABC um triângulo acutângulo com ∠ACB = 60Âş , AD e BE duas de suas alturas que se intersectam no ponto H . Mostre que o circuncentro do triângulo ABC pertence Ă bissetriz dos ângulos ∠AHE e ∠BHD .

BFĂˆĂŠĂ‰ĂŒĂ‹ Ă? ĂŽ Ă? Y `'Ă?'Ă? Ă?'Ă‘

Ç

Sobre o lado AC do triângulo ABC , toma-se um ponto D distinto de A e C . Sejam O1 e O2 os centros dos círculos circunscritos aos triângulos ABD e CBD respectivamente. Mostre que os triângulos O1 DO2 e ABC são semelhantes.

68.

BFĂˆ'Ă’ Ă“#Ă”FĂ• Ă? ĂŽ Ă? Y ` Ă?'Ă?'Ă? Ă‘

Ç

Determine todos os inteiros n para os quais o valor da expressĂŁo abaixo ĂŠ inteira. 25 + 2

Ç

70.

Ç

BFĂˆ'Ă’ Ă“#Ă”FĂ• Ă? ĂŽ Ă? Y ` Ă?'Ă?'Ă? Ă‘

69.

71.

72.

625 − n + 4

25 − 2

625 − n 4

Determine todas as funçþes f : Ă– → Ă–

tais que

f ( f (n )) = n e f ( f (n + 2 ) + 2 ) = n para todo inteiro n e f (0 ) = 1 .

? Ă—'Ă’ Ă? Ă˜ĂšĂ™ Ăˆ Y `'Ă? Ă?'Ă? Ă‘

Os triângulos equilåteros ABF e CAG são construídos externamente sobre a hipotenusa AB e sobre o cateto CA do triângulo retângulo ABC . Se M Ê o ponto mÊdio de BC , determine BC se MF = 11 e MG = 7 .

? Ă—'Ă’ Ă? Ă˜ĂšĂ™ Ăˆ Y `'Ă? Ă?'Ă? Ă‘

Ç

Determine todos os pares de inteiros positivos satisfazem à equação

Ç

? Ă—'Ă’ Ă? Ă˜ĂšĂ™ Ăˆ Y `'Ă? Ă?'Ă? Ă‘

(m − n )2 (n 2

(m, n )

que

)

− m = 4m 2 n

Quantos pares (n, q ) satisfazem a igualdade

n!  {q }=  2000  2

  com n inteiro positivo e q um número racional não inteiro tal que

0 < q < 2000 , onde {x} = x − x  ?

EUREKA! N°10, 2001

48


Sociedade Brasileira de MatemĂĄtica

73.

HIĂ“ Ă’ Ă› ĂœĂŠĂ”FĂŽ Ă? Y ` Ă?'Ă? Ă?'Ă‘

Ç

O ortocentro H de um triângulo ABC nĂŁo pertence a nenhum de seus lados. Sabendo que a medida de AH ĂŠ igual ao raio do cĂ­rculo circunscrito ao triângulo ABC , determine a medida do ângulo ∠A .

HIĂ“ Ă’ Ă› ĂœĂŠĂ”FĂŽ Ă? Y ` Ă?'Ă? Ă?'Ă‘ Ç

74.

75.

Resolva em Ă?

(x

2

a equação

− 3 x − 2) − 3(x 2 − 3 x − 2) − 2 − x = 0 2

HIĂ“ Ă’ Ă› ĂœĂŠĂ”FĂŽ Ă? Y ` Ă?'Ă? Ă?'Ă‘

Ç

Dado o nĂşmero 2000 , calcule a soma das dĂŠcimas potĂŞncias dos algarismos deste nĂşmero e continue fazendo o mesmo com o nĂşmero obtido e assim sucessivamente. Mostre que entre os nĂşmeros obtidos existem pelo menos dois nĂşmeros iguais.

HIĂ“ Ă’ Ă› ĂœĂŠĂ”FĂŽ Ă? Y ` Ă?'Ă? Ă?'Ă‘

Ç

a, b, c satisfazem à relação a + b + c = 0 . Mostre que o número 2a 4 + 2b 4 + 2c 4 Ê um quadrado

76.

Os

nĂşmeros

inteiros

perfeito. 77.

Ă? ĂĄ âãĂŽ Ă? Y ` Ă?'Ă? Ă?'Ă‘

ÇĂ&#x;Ăž Ă

Ă? ĂĄ âãĂŽ Ă? Y ` Ă?'Ă? Ă?'Ă‘

ÇĂ&#x;Ăž Ă

4

1000

Determine

3

para

os

quais

2

\ ä'ĂĽ ĂŚ ç Ă’ ĂŽ è Ă?'Ăˆ V è ĂŠ ä è'Ă?Â?äêBFĂˆ Ă’ Ă“#Ă”FĂ?'è Ă? Y `'Ă?'Ă? Ă?'Ă‘

Uma função

\ ä'ĂĽ ĂŚ ç Ă’ ĂŽ è Ă?'Ăˆ V è'ĂŠ ä'è Ă?ÏäíBFĂˆ Ă’ Ă“#Ă”FĂ?'è Ă? Y `'Ă?'Ă? Ă?'Ă‘

Se n ĂŠ nĂşmero natural, mostre que

Ê tal que f :Í → Í k f (n ) = 1 se n Ê ímpar, e f (n ) = k para todo inteiro par n = 2 l , onde k Ê um número natural e l um número ímpar . Determine o maior número natural n para o qual f (1) + f (2 ) + ⋅ ⋅ ⋅ + f (n ) ≤ 123456

Ç

Ç

80.

4â‹…3 + 3â‹… 4 2n

81.

)

todos os inteiros n n − 4n + 14n − 20n + 10 ĂŠ um quadrado perfeito.

78.

79.

(

Se A = 1000 + 1000 2 + 1 , determine o 2000–Êsimo algarismo apĂłs a vĂ­rgula da sua representação decimal.

Ç

2n

ĂŠ divisĂ­vel por 13 se, e somente se, n ĂŠ par.

G âÚÎ Y ` Ă?'Ă? Ă?'Ă‘

Dois cĂ­rculos se intersectam nos pontos A e B . Uma reta l que passa por A ĂŻ intersecta estes cĂ­rculos nos pontos C e D . Se M e N sĂŁo os pontos mĂŠdios dos arcos BC e BD que nĂŁo contĂŠm A e K ĂŠ o ponto mĂŠdio de CD , mostre que ∠MKN = 90Âş .

EUREKA! N°10, 2001

49


Sociedade Brasileira de Matemática

82.

G âÚÒ Ï'Í Û Ï Y ` Ð'Ð'Ð Ñ Ç

(

Sejam x ≥ 0 , y ≥ 0 números reais tais que x + y = 2 . Mostre

)

que x y x + y 2 ≤ 2 . 2

83.

84.

2

2

á Ó È'Ò ÜñÔFÎ Ï ò7ó'Ð Ð'Ð Ñ

Çßð à

Em um tubo de ensaio há exatamente uma ameba. A cada segundo algumas das amebas dividem-se em sete novas amebas ou morre exatamente uma das amebas. Determine o período mínimo de tempo após o qual o número de amebas no tubo de ensaio será igual a 2000 .

á Ó È'Ò ÜñÔFÎ Ï ò7ó'Ð Ð'Ð Ñ

Çßð à

Mostre que todo número racional positivo pode ser representado sob a forma

r=

a3 + b3 c3 + d 3

onde a, b, c e d são inteiros positivos. 85.

86.

Çßô

Ó Ò Ë Í Î Ï'òßó'Ð'Ð Ð'Ñ

Os lados AC e BC de um triângulo ABC possuem a mesma medida. Sejam P um ponto do interior do triângulo tal que ∠PAB = ∠PBC e M o ponto médio de AB Mostre que ∠APM + ∠BPC = 180º .

Çßð

Í á Ò ÏñÉÌä'âÚâÚÏ'òßó'Ð Ð'Ð'Ñ

Dois círculos C1 e C 2 se intersectam nos pontos M e N e possuem uma tangente comum sendo P e Q respectivamente os pontos de tangência com os círculos. Se N é o ponto mais próximo de PQ e a reta determinada por PN intersecta C 2 novamente em R , mostre que MQ é a bissetriz do ângulo ∠PMR .

87.

Çßð

Í á Ò ÏñÉÌä'âÚâÚÏ'òßó'Ð Ð'Ð'Ñ

Para cada inteiro positivo k , definamos a seqüência (a n ) por

a0 = 1 e a n = kn + (− 1) a n−1 para n ≥ 1 . Determine todos os valores de k para os quais 2000 é um termo da seqüência. n

88.

Çßð

Í á Ò ÏñÉÌä'âÚâÚÏ'òßó'Ð Ð'Ð'Ñ

Sejam x, y, z números reais positivos tais que xyz = 32 . Determine o valor mínimo de

x 2 + 4 xy + 4 y 2 + 2 z 2 Ï'Ò è ö'Í Î è'Ï ò7ó Ð'Ð'Ð Ñ 89. Çßõ Determine todas as funções f : ÷ → ÷ propriedade :

EUREKA! N°10, 2001

50

que possuem a


Sociedade Brasileira de MatemĂĄtica

f (xf (x ) + f ( y )) = ( f (x )) + y para todos os nĂşmeros reais x e y . 2

90.

ø þ Ú

Ă’ ú™Ý è"Ăź

ÚÊý

ò7ó'Þ Þ'Þ

) Seja ABC um triângulo tal que BC AB + BC = AB − BC AC

Determine a razĂŁo ∠A : ∠C .

ÿ¸ÿíÿ

Enviaram soluçþes de problemas anteriores os seguintes leitores da ! !" #$ !% & % ( " 4 ( +, / + ( + % &: "; < .=* (&<" +, > ? ( + % ( ? .=* :& + ("7D=*+ E F, +,+ ( % K ? .LM( N O"4 P 8":" +, ; T& U" . 5 +, ? X ( + "( +, .) +,0 ) XD .(" *L + ( Z O2 " ! ( ; ( #6(" Z O" +[ ( #6( ; +,O$( ' / ( +\ (! 7 ; +,O&^ ;* + ! (! + :" 4 ; +,O& ;* + ! (! + :& 4 ; +, Z . !; ( ( +, ' +, ? ' _ (!% " ' +, ? .LD +, ;* (&# Q 4 ? !M :" !M / +

` O2( ' +, ? (" ! (!a !;* + b +\ "X. .(" Z .

' (*)" ( +, .-!' ) 5 + #6 ("7 *-! 8 5 + #6 ("7 *-! 8 5 + #6 ("7 *-! 8 Q ( + (" *-! R "V " .-! = #$ / *-!W "V " .-! 5 + #6 ("7 *-! 8 5 + #6 ("7 *-! 8 ' (*)" ( +, .-!' ) ' (*)" ( +, .] ' ) Q ( + (" *-! R Q ( + (" *-! R 5 + #6 ("7 *-! 8 "V " .-! ' (*)" ( +, .-!' ) 5 + #6 ("7 *-! 8 Q ( + (" *-! R 5 + #6 ("7 *-! 8

+, / 021&3 +, / 021&9 +, / 029 @"A2B"C A$B @ +, / 029 G"A21"9 A$1"3 A2B 1 A$B @"AIH J"A$H"9 +, / 0"J&A29"3 A29&S A21 J"AI9 @ +, / 021&H A21"S A$B"H A2B 3 A$B"C AIH"9 A$H"3 +, / 021&9 A2B"1 A$B"H A2H 3 +, / 021&9Y .H"S +, / 021&C A2B"1 A$B"H +, / 021&9 A21"H A$1"3 A2B 1 A$B"H AIB"C A$H J A2H"9 A$H B +, / 021&9 A2B"1 A$B"B A2B C A$H J"AIH"9 +, / 021&9 A2B"B +, / 0"@&A21"C A2B&H A2H J +, / 029&3 A29"C A$B"B A2B S A$H G +, / 02B&C +, / 029 G"A2B"1 +, / 021&9 A21"3 A$B"1 A2B H A$H J"AIH"1 A$H"BMA2H"3 A$H C +, / 021&9Y .H"3 +, / 021&9 A21"1 A$B"S A2HMJ"A$H"9 AIH"3 A$H"S +, / 021&9Y .H"3

cedgfihkjmlonqp2lsr tvuWwyxMz|{ zy}~z } uD z|{€zW Â‚{ z ƒ…„ w  Â‡Â†Âˆu z ‰ { zˆ‰Šz  Â‹Â‰ÂŒwÂŽÂ? Â?*Â?mÂ?oÂ?’‘YÂ? Â?“Â? ”*”•Â?oÂ?&– — Â? ˜š™*™m›ˆ˜*˜!ÂœÂ?›ˆ˜ žD˜•›ŠÂ&#x; ™*™•›ŠÂ&#x;Y Âœ Â›ÂŒÂĄ ¢WÂŁ ¤ ¢Œ¼ ¢ÂŒ§ ÂĽW¨D§ ¢ÂˆŠÂ“¢ £ªŠ ¢ ÂŤÂ…ÂŹ ­ ÂŁÂŞÂŽ.ÂŻW£°¢ § Âą ¢ÂˆŠ ²£ ³´¯WÂŁ ÂľÂˆÂś ¡M¸ š ÂşÂŒÂť ¾½Ÿ ÂťWÂľož Âż ¡ Ă€ žÂŠà ´WÂľ Ă‚ Ăƒ Ă„°Ă… ÆMÇ Ăˆ Ă…WÉ½ĂŠ Ă‹ ĂŒĂŽĂ?|Ăˆ Ă…WɓĂ?ÂŒĂ?Ă‘Ă?Ă’Ă?ÂŒĂ“Ă•Ă”.Ă–6Ç Ă…WÉoĂ—MĂ…ÂˆĂŒĂ˜Ă? É É Ă™°ĂšÂˆĂˆ Ă… ĂšÂˆĂˆ Ă– Ă?ˆĂ?ÂŠĂ™ÂˆĂ?.Ă?ÂˆĂ› Ăœ Ă? Ăž Ă&#x; Ă Ă?Â?ĂĄ Ă&#x;Â?â"ĂŁÂ?Ă?ĂŽä~ĂĽÂ“ĂŚÂˆç Ăž ä Ă?½è Ăž ĂŠ"ĂĽ êÍä Ă?½ÏšĂ&#x; ĂŞ Ă­ ç ĂŽ*ĂĽ ĂŻ&Ă° Ă?ĂŽĂą,ò|Ă°ĂłĂ&#x; ôšþ!Ă&#x; ĂĄ&ç Ă&#x;“ä Ă&#x;°Ü Ă?Â’Ă&#x;½áDç êøð&Ă&#x;ĂŽĂšiú’Ý Ăź

EUREKA! N°10, 2001

51


Sociedade Brasileira de MatemĂĄtica

ýÎÞvĂż ĂŽĂ˝ oĂž °ÿ Ă˝ Ăž gĂžvĂ˝ ĂžvĂ˝

!#" $% & ' !# $%( $%)+* $ , " $.-/ $%* 0213 ( $4, " )50 " $6$ " $% *7 -/" )8* $ 9

41) Se a e b sĂŁo nĂşmeros reais positivos, entĂŁo a b + b a > 1.

:<;>=@?>ACB5;ED>;>FHG5=@? I>;JF D>;EKML<N O<PCQSR>T5UVTWFCX@I>GHY O<P@Z Observe que se “aâ€? ou “bâ€? for maior ou igual a 1 a desigualdade ĂŠ imediata. EntĂŁo analisaremos o caso em que 0 < a, b < 1.

1 1 ,b= , u > 0 e v > 0. (1 + u ) (1 + v) 1 1 1 >* = = EntĂŁo a b = b 1+ u â‹…b 1+ u â‹…b (1 + u ) Seja a =

1 1+ v = e  1  1+ u + v 1+ u â‹…  1+ v 

1+ u e portanto: 1+ u + v 1+ v 1+ u 1 ab + ba > + = 1+ > 1. 1+ u + v 1+ u + v 1+ u + v

Analogamente b a =

*Agora o problema se reduz a mostrarmos que (1 + u ) b < 1 + ub para 0 < b < 1. Para isso basta termos uma noção de cålculo diferencial

  1 f (u ) = 1 + bu − (1 + u ) b ⇒ f ' (u ) = b − b(1 + u ) b −1 = b 1 − > 0. 1− b   (1 + u )  Como f ĂŠ crescente no intervalo (0,1) ⇒ f (u ) > f (0) = 0 e portanto segue imediatamente a demonstração. 45) Existe uma seqßência infinita de: a) NĂşmeros reais b) NĂşmeros inteiros Tais que a soma de quaisquer dez termos consecutivos ĂŠ positiva, enquanto que para todo n a soma dos primeiros 10n + 1 termos consecutivos ĂŠ negativa?

[<\>]@^>_C`5\Ea>b cJ\>dV\>e5fCgSdh\EiMjke5lEm>^>n@eHoMb5gS\EpSq4rs\ta>bHu5e5vJbWr@dV\xwHqMu5y a) Sim. Tome a10 k = 1 +

1 1 para todo inteiro positivo k e a n = − se n nĂŁo ĂŠ k 9 10

mĂşltiplo de 10.

EUREKA! N°10, 2001

52


Sociedade Brasileira de Matemática

1 1 1 1 1 + + ... + n − = − < 0 e, se 10 100 9 10 9 ⋅ 10 n 1 10k < n ≤ 10( k + 1), a n + a n +1 + ... + a n + 9 = k +1 > 0. 10 b) Não. Se n > a1 , a1 + a2 + ... + a10n+1 = a1 + (a2 + ... + a11 ) + (a12 + ... + a21 ) + Temos a1 + ... + a10 n +1 =

+ (a10n− g + ... + a10n+1 ) > a1 + 1 + 1 + ... + 1 > a1 + n > 0. 47) Dada uma circunferência Γ , trace as tangentes a ela por um ponto exterior, A, tocando-a em M e N. Trace a reta r passando por A e tocando Γ em B e C. Se D é o ponto médio de MN , prove que MN é a bissetriz de ∠BDC.

[<\>]@^>_C`5\Ea>b zM]@{C^>a>r@\EiMdh|C\>v |5}Jb5d<pSu5^Jv>a>r@eC~Ww [< <y L

r*

O’ N

r P

C

B

A

D

O Γ

M

Consideremos a inversão na circunferência Γ de centro O. Notemos que o ponto D, pela sua definição, é o inverso do ponto A em relação a Γ , B e C coincidem com seus inversos em relação a Γ . Assim a figura inversa da reta r é a circunferência r* que contém O. Assim sendo as circunferências Γ e r* compartilham a corda BC . A reta perpendicular a BC pelo ponto médio P passa por O e O’ (centro de r*). Por esse motivo os arcos RL e CL indicados na ___

figura acima são congruentes, ( OL é um diâmetro de r*). Notemos agora que o fato de ser reto o ângulo NDˆ O implica que a reta pelos pontos N e D passa por L. Isso completa nossa demonstração: os ângulos inscritos

BDˆ L e CDˆ L ”enxergam” arcos congruentes do mesmo círculo, melhor, circunferência, r*, são congruentes por esse motivo. 49) É dado um polígono regular de n lados. EUREKA! N°10, 2001

53


Sociedade Brasileira de Matemática

Assinale aleatoriamente, no seu interior, um ponto M. Sendo x1 , x 2 ,..., x n as distâncias de M a cada um dos lados, verifique que: 1 1 1 2π + + ... > , onde a é a medida do lado do poligono. x1 x 2 xn a

[<\>]@^>_C`5\Ea>b iMv>gS\Jv>r@\xzMeClxr v>}>eHoMb5gS\EpS >\>dVgSe5]@b5j eHw zM <y Sejam O o centro e A1 , A2 , ...., An os vértices do polígono. Podemos supor, sem perda de generalidade, que xi é a distância de M ao lado Ai Ai +1 (aqui,

An +1 = A1 ). Então n

n

i =1

i =1

A( A1 A2 ... An ) = ∑ A( Ai MAi +1 ) = ∑

axi a = ( x1 + x 2 + .... + x n ) (*) 2 2

Por outro lado, sendo r o raio do círculo Γ inscrito no polígono, temos também n

n

ar ar (**) = n× 2 i =1 i =1 2 Comparando (*) e (**), concluímos que x1 + x 2 + .... + x n = nr . Agora, note que o perímetro de Γ é menor que o do polígono, o que nos dá 2 r < na . Então n2a x1 + x 2 + .... + x n = nr < 2 A( A1 A2 ... An ) = ∑ A( Ai OAi +1 ) = ∑

Finalmente, sabemos (ver artigo “Desigualdades Elementares, A. Caminha”, x1 , x 2 , ...., x n Eureka 5) que para os reais positivos tem-se

1 1 1 n2 + + .... + ≥ , e daí x1 x 2 x n x1 + x 2 + .... + x n

1 1 1 n2 n2 2 + + .... + ≥ > 2 = . x1 x 2 x n x1 + x 2 + .... + x n n /(n 2 a / 2π ) a 50) Calcule o determinante: MDC (1, 1) MDC (1,2) MDC (1, n) MDC (2,1) MDC (2,2) MDC (2, n)

MDC (n,2) MDC ( n, n)

MDC (n,1) Onde MDC (a, b) é o máximo divisor comum de a e b.

EUREKA! N°10, 2001

54


Sociedade Brasileira de Matemática

[<\>]@^>_C`5\Ea>b {5dV|5rs\xiM 5iMf5fCe5aEz4\J}>b5vtbH >{Cm>r@\xa bH ]@r 5bCr@dVe zM\>fCgSe pSqMr@\Ea b uCe5v>bCr@dV\xwHq%uCy Convenção: det (n) é igual ao determinante desejado quando este tem n linhas e colunas. Inicialmente escreveremos o determinante com uma quantidade de linhas suficiente para percebermos como o problema se comporta ao mudarmos o valor de n. 1 1 1 1 1 1 1 1 1 2 1 2 1 2 1 2 1 1 3 1 1 3 1 1 1 2 1 4 1 2 1 4 1 1 1 1 5 1 1 1

Determinante quando n = 8

1 2 3 2 1 6 1 2 1 1 1 1 1 1 7 1 1 2 1 4 1 2 1 8

Pode-se perceber que em cada linha k (1 ≤ k ≤ n ) do determinante, os elementos dessa linha considerada repetem-se com período k, pois mdc(x,k) = mdc(x + k, k) Como mdc(1,n) = 1 ∀n ∈ , a primeira linha é 1 1 1 ... 1 Além disso, para todo p primo, mdc(n, p) = 1 se n < p, logo a p-ésima linha é da forma 1 1 1 ... 1 p, e a p-ésima coluna idem quando tomamos n = p Logo, fazendo Lp = Lp – L1 vemos que det(p) = (p-1) det(p-1), sendo det(p) o determinante desejado quando o número de linhas é primo. Obs: Fazer Lp = Lp – L1 significa trocar a linha p do determinante pela combinação linear (linha p – linha 1). Essa notação será usada outras vezes no decorrer dessa resolução. Calculemos então o determinante para alguns valores pequenos de n: det(2) = 1.det(1) = 1; det(3) = 2.det(2) = 2; Para calcular det(4), note que podemos fazer L4 = L4 – L2 para concluir que det(4) = 2.det(3) = 4, e portanto det(5) = 4.det(4) = 16; det(6) é um pouco mais difícil de calcular, mas notando a similaridade da L6 com L2 e L3, pode-se perceber que fazendo L6 = L6 – L3 + L1 – L2 reduzimos a ordem do determinante de forma que det(6) = 2.det(5) ( pois a6,6 vira 6 – 3 + 1 – 2 = 2 e para i < 6, a6,i = 0). Daí, det(7) = 6.det(6) também fica determinado. Para det(8) é fácil ver que a combinação linear a ser utilizada é L8 = L8 – L4 e para det(9), a combinação linear é L9 = L9 – L3. É razoável supor que sempre é possível encontrar uma combinação linear que reduza de 1 a ordem do determinante. Note que os números das linhas utilizadas nas combinações lineares sempre foram divisores do número de linhas do determinante correspondente ( por exemplo, no caso n = 6 usamos as linhas 1, 2, 3 e 6 ). EUREKA! N°10, 2001

55


Sociedade Brasileira de Matemática

Mais do que isso, os sinais que antecedem as linhas nas combinações lineares parecem estar ligados à função µ de Möbius! De fato, no caso do 6, temos: L6 = µ (6) L6 + µ (3)L3 + µ (2)L2 + µ (1)L1. Entretanto, essa combinação linear falha para L8. No caso, entre os divisores de 8, apenas 1 e 2 têm µ não nulo. Mas estamos usando exatamente L4 e L8. Uma maneira de satisfazer esse caso e o do 6 é fazer:

8 8

8 4

L8 = µ   L8 + µ   L4 = L8 – L4. Note que para n = 6 temos:

6 6

6  3

6 1

6 2

L6 = µ   L6 + µ   L3 + µ   L2 + µ   L1 = L6 – L3 – L2 + L1, como desejávamos. Tentaremos então provar que fazer a substituição Ln =

n

∑ µ d L

d

é suficiente

d |n

para reduzir a ordem ( note que para d = n isso dá Ln). Temos então que as entradas da última linha zeram, com exceção daquele que também está na última coluna do determinante. De fato, consideramos a coluna x, x ≠ n. ( que na última linha equivale à entrada mdc(x, n)) Quero calcular

n

∑ µ  d .mdc( x,d ). d |n

Vamos tentar usar a fórmula da inversão de Möbius (no caso, g(d) = mdc(x, d)). f (d )∀n > 0 , o Se acharmos uma função “f” tal que g ( n ) =

∑ d |n

problema termina porque aí f ( n ) =

n

∑ µ d .g (d ). d |n

Tomando então f :

 f (d )= 0 se d não divide x , , sendo ϕ a função de  f (d )= ϕ (d ) se d divide x

Euler temos:

∑ f (d ) = ∑ ϕ(d ) = ∑ ϕ(d ) d |n

d |n d|x

(***)

= mdc(x, n) = g(n) ∀ n .

d |mdc ( x ,n )

EUREKA! N°10, 2001

56


Sociedade Brasileira de Matemática

E se x < n, temos que n não divide x, e portanto

n

∑ µ d .mdc( x , d ) = f (n)= 0

f (n )= 0 . Logo,

para x < n. Se, entretanto, x = n, então

d |n

f (n )= ϕ(n ) ( pois n divide x = n ) e

n

∑ µ d .mdc( n, d ) = ϕ(n ) portanto, a d |n

última linha passa a ser da forma 0 0 0 0 ... ϕ(n ) e então n

det (n )= ϕ(n ). det (n − 1) Logo, det (n )= ∏ ϕ(i )= ϕ(1).ϕ(2 ).ϕ(3)...ϕ(n ) . i =1

(***) pois temos ϕ(d )= n .

como conseqüência da fórmula da inversão de Möbius,

d |n

51) Três feirantes foram vender melancias. Um levou 10; outro 16; o terceiro, 26. Todos venderam algumas melancias pelo mesmo preço até o meio dia. Depois disso, os três baixaram o preço, mas continuaram vendendo por preços iguais. Quando voltaram para casa, após venderem todas as melancias, cada um tinha a mesma quantia de dinheiro; 35 mil cruzeiros. Por quanto foi vendida cada melancia antes e após o meio-dia?

[<\>]@^>_C`5\Ea>b Mr@ C >\x <bC] \>f5\E M|5}> >eHpS b5dVbCf5r@v>eHwH < @y Sejam A1 , A2 e A3 os vendedores que venderam 10, 16 e 26 melancias respectivamente, xi a quantidade de melancias que o vendedor Ai vendeu antes do meio dia e y o preço antes do meio dia e y1 o preço após o meio dia. Portanto, teremos:

I) y ⋅ x1 + (10 − x1 ) y1 = 35000  II) y ⋅ x 2 + (16 − x 2 ) ⋅ y1 = 35000 III) y ⋅ x + (26 − x ) ⋅ y = 35000 3 3 1  De I) e II)

⇒ y ⋅ ( x1 ) + (10 − x1 ) y1 = y ⋅ x 2 + (16 − x 2 ) y1 ⇒ y ( x1 − x 2 ) = (6 + x1 − x 2 ) y1 Analogamente de II), III) e I), III) teremos y ( x1 − x3 ) = (16 + x1 − x3 ) y1 e y ( x 2 − x3 ) = (10 + x 2 − x3 ) ⋅ y1

EUREKA! N°10, 2001

57


Sociedade Brasileira de MatemĂĄtica

x1 − x 2 = w1

y â‹… w1 = (6 + w1 ) â‹… y1

Faça x1 − x3 = w2 ⇒ y â‹… w2 = (16 + w2 ) â‹… y1 ⇒

x 2 − x3 = w3

y â‹… w3 = (10 + w3 ) â‹… y1

(6 + w1 ) â‹… y1 16 + w2 6 + w1 y â‹… w1 16 w2 6 w1 = ⇒ = ⇒ + ⇒ + ⇒ y â‹… w2 (16 + w2 ) â‹… y1 w2 w1 w2 w2 w1 w1 16 6 8 3 ; Imagine, agora, essas duas fraçþes nas suas formas = ⇒ = w2 w1 w2 w1 irredutĂ­veis e como os seus numeradores deverĂŁo ser iguais e o nĂşmero 8 nĂŁo tem o fator “3â€? entĂŁo w1 ĂŠ mĂşltiplo de "3" ⇒ w1 = 3k ( com k ∈ – ) se

k ≠1 ⇒ w1 ≼ 6 ⇒ w2 ≼ 16 (Absurdo! pois w2 = x1 − x 2 < 10, pois x1 < 10) logo k = 1 e w1 = 3 ⇒ w2 = 8 e w3 = 5 . Como todos os vendedores venderam algumas melancias antes e apĂłs o meio dia temos que 1 ≤ x1 ≤ 9 e 1 ≤ x3 ≤ 25 mas w3 = x1 − x3 = 8 ⇒ x3 = 1 e x1 = 9 ⇒ x 2 = 6. Lembrando que y â‹… w1 = (6 + w1 ) â‹… y1 ⇒ y = 3 â‹… y1 e assim substituindo em I) 3 y1 â‹… x1 + (10 − 9) â‹… y1 = 35000 ⇒ 28 y1 = 35000 ⇒ y1 = 1250 cruzeiros e y = 3750 cruzeiros.

[<b5Â?>^Jr@lx\>fHe5Â?J^>e5dVa>eCv a>\xeHf5\>]s^ _5`5\Ea>\x—Jdh\Jm>]€b5lEe ˜C™š—>^>mJ]@r@|5eCa>\tv>eH‚<Â’MqM‚#›JiÂ?Âœ7o ž ‰5Â&#x; iMÂ?>dVe5a>bC|5b5lE\>fHgSe5lxm  ClÂĄ\Eb5v>‹Cr€\xa>eCf f5\>]@^>_C¢>b5f b eH|5\>]€eCm>\>dVe5_C`5\Ea b¤£ ÂĽ

ÂĽ Ăš ÂŚ/§ ¨CŠ#ÂŞÂŹÂŤ ­%¨CÂŽÂ?Ăş ÂŻ Š Ăť Ăť ºÊú ÂĽ Ăš ÂŚ/§ ¨CŠ#ª§ Âľ5ÂŻ ÂŚ Ăť Âż ¨Âš Âś Ăš#¡ § ¸FĂš š Ăť ¨CÂŚ Ăƒ Ăš Ăť ÂŚ/Âż § ¨CŠ#³¨5ÂśCÂŚ ¨Â?Ă€ ÂŻ Ă CÂś5ÂŻ ÂĽ Š#Ă‚ ÂŽ-Ăť Ăš ĂŠ Ăť ÂŻ Ăş ¨ÂšĂť ª¤§ F¸ Ăš ÂŚĂ„ÂŻ Ă…4ÂŞ<'Âş ڌ Ăš Ăť Ă†ĂˆĂ‡/¨ Ăş ¨5ÂŚ.ÂŚ/ÂŻ Ăš Ăť Ă 5Š ÂŻ Ă Â?Âś5¨ ÂĽ ° Ăš Š ÂŽÂ?ÂŻ à ¨Â?žkÆCĂ ÂŚ É ÂŻ ÂŚ/ Ă Ăš Ăť Ă 5Âż ÂśC¨ Ăš Œ€ ¸ Ăš § Ă‹5¨Â? Âł Ăš ÂŽÂ?¨5Š É ¨CÂŚ/ÂŽ Ăš ­%ÂŻ ÂŚ/Ă? § Âś5¨Â?Ă? ÂŻ ÂŚ/§ Ă? Ă 5¨ ­%ÂŻ ÂŚ/Ă? § Âś5¨Â?Ă? ÂŻ ÂŚ/§ Ă? Ă 5¨Â?ž Æ5Ă CĂ? ¨CÂŚ ­%Ă? § Âş Âľ5ÂŻ ÂŚ Ă“³¤ ž ¨5Ă‹5Ă CŠ ¨CĂ Ă‚5ĂŽ Ă? Ă? Ă 5¨ ° ĂŽ5Ă CÂŻ Š Ă? ÂŚ/ĂŽ5ÂśCÂŻ Ă Ă–Ă“VÂŻ Ă€#Ă™ Ă? Ă 5ĂŽCÂŻ § Ă€#ĂŽCÂŚ/Ă? § § ¨MĂ—Š Ă? Ç/§ Ă? ŠĂ˜¸ Ă? § Âś5¨ÂšĂ€#ÂŻ § ¨Âš¡ Âź5¨5Ă CĂš5ĂŽCĂ? Ă? ÂśW¨ Ă? Ă? ĂŽC§ ¨Â?Âż Âś5ÂŻ ¡ ¨5ĂŽ5Ă… Ă? Ăƒ ¡ ¨5Âľ5ÂŚĂ„Ă? Ă CĂ‹5¨ ³¨5Âş Âś5ÂŚĂ„Ă? ¨Â?³¨5ĂšCĂŽ5ÂŻ Ă? Ă? Š š Ă? ÂŻ Ă ĂˆĂ“ĂœÂŻ<Ă?ĂžĂ? § Š ¨CĂ Â?Ă€#¨CĂŽ5ÂŚ/Ă? ­%Ă? ¯²Ă“ĂœĂ?

Ăş ø+° ÚÝ Ăš §²¹ Ăť Âł °´ ø+° Ăť § ÂťCÂź5¨C§ Š½¹H³Ý ž ´ ø Âł ¨Â?Ăş Âś5ÂŻ ž Ăš Ă 5ÂŻ ÂĽÂŹÂŚ/ĂŠ ¨MÂąH³ž ´ ø+É ¨CÂŚ Ăş Ăš § ÂŻ Ă… ½Ú Âą ÂĽÂŹĂŠ ´ ø+É ¨CÂŚ Ăş Ăš § ÂŻ Ă… ½Ú Âą Ăť ´ ø ¡F Ăš Ă # Ăš Ă€N Ăš ÂŚ <Ăš Âą ³¡ ´ ø ¡ ĂŒ ¨Â?Ă?F Ăš Âş ĂŽC§ ¨MÂąH¡ Ă? ´

Ă‘+Ă’ Ă“VĂ? Âź5Ă? ÂŚ/Ă? Ă?#Âś5Ă? ¡ ÂŻ ÂŚĂ„ÂŚ/Ă?<¹Ô¡>Ă? ´ Ă‘+¡ ĂŠ ĂŒ Ă• ¨Â?Ă? Ă? ĂŽC§ ¨MÂąH¡ Ă? ´ Ă‘ ÂŞ ´ Ă‘+³¼ Ă? ¨Â?Âś5ÂŻ ž Ă? Âş Ă 5ÂŻ Ă? ÂŚ/¨MÂąH³ž ´ Ă‘ Ă™ ¨5ŠĂ˜Ă“VĂ? ³Ă? Ă? ´ Ă‘ § Ă› ÂŽÂ?ÂźCĂ? Ă?<ÂąH¡ Ă? ´ Ă‘ ° Ă?²Ă“ĂœĂ? §²¹ Âł °´ Ă‘+¡ ĂŒ ¨Â?Ă? Ă? ĂŽC§ ¨MÂąH¡ Ă? ´ Ă‘+Ă€#Ă? Ă 5Ă? ĂŽ5Š½¹HÂŞÂŹĂ€ ´

EUREKA! N°10, 2001

58


Sociedade Brasileira de MatemĂĄtica

°Þ °ÿ ý Þ gÞvý

ĂžvĂ˝

Ă&#x; Ă ÂŹĂĄ âäã.ĂĽ ĂŚ ç è ĂĄ ĂŠEåÞê ĂŤ ĂĽ ĂŹ ĂĄ íÎçÞÍ â ĂŁĂ„ĂĽ ç Ă­ĂŽĂŠ ĂĄ ĂŞ ĂŻ Ă° Ăą ĂŤ ĂŠEĂŚ ĂĄ Êóò Ă­+ĂĄ Ă´ ĂŞ ĂŤ è ç ĂŠÂ?òäí+ĂĄĂ˜òäå ĂŠ ĂŹ ĂĄ ĂŠxÍþÊ6ĂŻ Ăś ĂŤ ĂŠ ĂŹ Ăą ĂŤ7ĂŠ ĂŚ Íáâ ĂĄ ĂŁ.ĂĄ ĂŠ äò Ă­+ĂĄ Ă´ ĂŞ ĂŤ è#ç ĂŠ>ò ç Ă­8ç ĂĄ ĂŠtòäí+ø²ÚWĂĽ è ĂĄ ĂŠ4â Ăş è#ĂŤ Ă­+ĂĄ ĂŠ . 52) Quatro retas se interceptam formando quatro triângulos conforme a figura abaixo. A

D E F

C

B

a) Prove que as circunferĂŞncias circunscritas aos quatro triângulos possuem um ponto em comum. b) Prove que os centros dessas quatro circunferĂŞncias sĂŁo concĂ­clicos (i.e. existe uma circunferĂŞncia que passa por todos eles). 53) Prove que num cĂ­rculo convexo dado e para o mesmo nĂşmero de lados. O polĂ­gono regular inscrito ĂŠ aquele cuja superfĂ­cie ĂŠ mĂĄxima. x 54) Sejam ( x n ) a seqßência definida por x1 = 2, x n +1 = 2 n , ∀n ≼ 1, e ( y n ) a 2001

y1 = 2001, y n +1 = 2001( yn ) , ∀n ≼ 1. Prove que existe c natural tal que y n ≤ x n + c para todo n ∈ Ăť e determine o menor c seqßência definida por com essa propriedade. 55) Seja S o conjunto de pontos interiores de uma esfera de raio 1 e C o conjunto de pontos interiores de um cĂ­rculo tambĂŠm de raio 1. Existe alguma função f : S → C tal que d ( A, B) ≤ d ( f ( A), f ( B)) para quaisquer pontos A, B ∈ S? (d (A, B) denota a distância euclidiana entre A e B).

Ăź Ă˝/Ăž5Ăż 5Ă˝/Ăž 5Ăž ĂœĂž CĂž5Ă˝ Ă˝ Ăž CĂ˝ %Ăž !#" $ &%(' " )+*, CĂ˝/Ăž5Ăż 5Ă˝/ Ăž 5 Ăž Ăœ Ăž 5ĂžC Ă˝ . / 10 Ă˝ 321 3!#0CĂž5Ă˝ 4 56 7%8 9 ) EUREKA! N°10, 2001

59


Sociedade Brasileira de MatemĂĄtica

;: 8<

B7B

ތÿ>=/ 8

?A@

•s•@•5Š>C>•@ˆH 7D@iMÂŽMiFEMq4iM[<•GCJ‚<•@qMi ÂŽ4Â‚ĂľÂˆ iM”>‚<ˆIHM”>•@zMi

o D@‘<‚<•@[LK M NHbPO J < dVr@lxbCr@dVe  Je5f5b Q [<b5Â?>^>vJa>e  Je5f5b Q SĂĄbado, 9 de junho SĂĄbado, 1 de setembro ”>bCdh|Cb5r@dVeH >e5fCb Q

SĂĄbado, 20 de outubro (nĂ­veis 1, 2 e 3) Domingo, 21 de outubro (nĂ­veis 2 e 3 - segundo dia de prova).

o D@‘<7‚ Cx’MoM•@‘<‚<qM[<•@”RH qM•@Š J < dVr@lEb5r@dVeH >e5fCb Q Såbado, 1 de setembro [<b5�J^>v>a>eH >e5fCb Q

Såbado, 20 e Domingo, 21 de outubro ♌

‘<•@•CŠ>C>•@ˆH 7D@iMÂŽ4i ÂŽMÂ‚ĂžÂˆHiM•@Š SĂĄbado, 12 de maio ♌ B C>•@•5Š>C>•sˆ 7D@iMÂŽMi •@oM”>‚<qMoMiMz4•sŠ

oMiJCxÂŽMÂ‚ĂžÂˆHiM”>‚<ˆIH4”J•@zMi

1 a 14 de julho Washington, Estados Unidos ♌

•@‘þŠSCJ•@ˆ 7Dsi4ÂŽMi •GEM‚<qMŠ iMˆ ‚<qM•@zMiMoMi ÂŽ4Â‚ĂľÂˆ iM”>‚<ˆIHM”>•@zMi Â’ oM•@‘<‚<qM[<•@”RHMqM•si SĂĄbado, 6 de outubro ♌♌♌ ♌♌ ♌

8TUT4V,W V

X

O leitor Diego Alvarez AraĂşjo Correia, de Fortaleza – CE, notou um erro tipogrĂĄfico na pĂĄgina 45 da revista Eureka No. 8. O enunciado correto do Teorema 2 (FĂłrmula de inversĂŁo de MĂśbius) ĂŠ o seguinte: Âľ (n / d ) g (d ). Se para todo n > 0 temos g (n) = ∑ f (d ) entĂŁo, para todo n, f (n) =

∑

d |n

d |n

EUREKA! N°10, 2001

60


Sociedade Brasileira de Matemática

@ þvþ 8< þ Îý P:F? þF< ;?$ý

Y[Z]\ ^G_ ` a b cd\[efa g h \ Y[^ \ ikj b Y[g { | ^ } b ~d\ `4` |U d\4\ c Y[ 6| g \R \ Z \ ^ b |U k|4cpa } b \4c6|U \ `4 4b] A | g b6` 6^ | a ^ | p\ ^Gg b `¢ 6^ | c | ^Ga 4b b]^ |4`,£$\ g Z | a ^ \ [g \U kcpa b Y[^ 4b] A U¦A| ^ [g \U k`¢~d\4| }na k | ^ [g |4b] b]^$ [^ |4` 4®U A a b c \ `¢¯d|4h |4` ° g a b ± | \ ³ ´ }µa \ ¶ bp k·4\ g h4|4`Rc |¢ \ ^Ga \ 6g bp^ ®U a b A| ^G^ | a ^ \R¶ 6a Z \ ^ ¸4| ` 6a g ¦kb ¶ a ` | g |Rcd|[Y[^ \Uikj bJ£ ^ \ } | \4c b ¶7 ` Z ¸ b ¼ h \U 6a g c6| A| ^½ A\U kc |4`Re \ \4c Á \4 Âp | g a A| k\U{6a b]g \¢ db j \4`[Y[^ \U a { a \ Á b ¸ bJ |U 6_ a b c |¢± | g b ¯d| } b Á b ¸ bJ 6^ \U k a ` 4b ± | g bÄÃ6a { b] A\ }µa ¼ ^ | k| ¯ \UÅn\4bpÅn\ Á b ` Ç p\ ^Gg b `¢£Ua k} bJÃk| a h \4` Á b ` Ç [g bdh |4`Re \ ^ \ a h \ Á b ` Ç ¶ \4` È \ ^U k\4`¢ 6a g ¦Ab Á b ` Ç Ã6 a É db `4\4`,£Ua ¦Ab Á b ` Ç fa | a ^ \ Y[g h4| ` ± \ ^ 4| g bJ p kʵa kb c6|RË g a h | a ^ \ Ã6a a bJ~ | ^½ A\U kc |4`¢ d| ÉA| ^G^ \ Ã6 AÉ6a \ g h4\¢±Ra ^ \U kc \Rcd|[Y Z b]^Ga Z ± \ ^ 4bp kcd| ` p\4h \ g \U k} | 6^ \U k·4\ £4\U{6g b dbdc]^Ga 6b ¶ \U k\4`4` a Z £4\U 6g b ~d|U ^ a Âp A|R [^½ kÉ ¯ | a h \ c | Ã6a Z \ Á ^GÏ | a k\ g c b ¶ |U ¼ ¦ a ^ b&Y[^ \Uŵ\UÅ a pa 4\ ^ c6b Y[Z]b]^Ga Z b {k| ^ } b fa Ék| & \ ^G^ b ` eUÇ ^ pa b [g ´ Acpa bÄ \UZ bd` efa g h a b c6|¢ d\ ^G^ b `¢± | g b 6\4c |U A| ^G^ | a ^ \R¶ b]Z |4` 6b Z ´4`¢± | AÇ Acd| É b cp^ a k|4` U\ g c6|U 6{ | ^ &Y[^ \UiAj b c \Refa g h4\ Ò \4 ] A| ^ £4| ^ | a ^ \¢ÃAb Èk|4` Ò \ g c | Z \ ^ ±RÏ+ \ A\ g g a

lnmpokq r lnmpok 6oAr l p 4 uR 4 , $ kw t u 4 u v xR R $ r lnmpo6 ¡ r ln¤ m p¥+ s u r l pu §4¨ s u ©n 4u4 w u w qUs As r lnmp¡p« qU U¬p 4 Ur lnm ¬6 y[r l pu §4¨ s u 4 w4² w r l pu §4¨ s u Us 4¨ w r lnmpo6 $ r lnmpokz º r lnm [¾4¿du À $u vA ku r lnmpo6¤ r lnmpo6¤ « r l z7 ²4u¢ $ w As u 4w 6« 4 w r lnmp yRr lnmpo6 dzfr lnmpoky r ln« py[ p¥nmd ¤ r lnmpok pr lnmpo6¤ r l s vk Rw ¬6s w µ 4 vks 4u r lnmpok pr lnmpo6 $ r lnmpok pr ln©µs A R¬d w v 4u, $ ¨4 4 Í4u r ln vA ku wR¬ § k s kw¢ 4Ì w 4u¢¤$w vAvAu vAr ln« ¡p¤ 4r l p 4 u¢ 4 Aw As u 4w ©+u ¨4u vkr l pu §4¨ s u U Aw4 r l « y[¥nmpo z r lnmpo6¤ r lnmp $ Ur lnmp¾4ok 4 w k 4 ¢ u4 4 Ñ4 s w r lnmp¾4ok 4 w k 4 R U ¨ s ²4 r ln vA ku wR¬ § k s kw¢o 4 w k zfu s Æ vkr ln¤ Ð s wRy s As ²4w A 4 R ¾+ Au 4u[ 4 Ry s s r

EUREKA! N°10, 2001

61

qUs tAu vkwRx yRz A s ,o u w xRyRz 4 4 w4 Rx ¡ w w Ax, ¡ s uR 4 [ Aw4 s u x, A s w ª+xR f¤ ©nw ­ w4 u x¢ d yRw4 w4 v xR y U 4u¢¤4w uRx[ ¤ Uw4 u[ 4 §Rx f¤ qUs ¹ s wRx 4 z u s »4 s w[xRz º pw4 R²4uRz7 w4 4 4xRy Au u ¤4 vkvAu4wRx ¤ ¬6 vks w x,¤U« 4 § x ¤4 yRw s 4¨ Æ x,¤ s uRz7 w 4 x¢ d U 4u¢©+ s v x[yR U 4uR w u v xR ¤ ok u s w4 ¹4²4u s v xR U pw4 R² s wRz w 4 4 Rx¢¤U 4 § x ¤4 ok u s w4 ¹4²4u s v xR U Uw ÌAw4 u µx¢ oku w Aw[x [ ¤4s w ks kw Î4wRxR ¤ U u v w ² u v x ¤ U u v w ² u v x ¤ ¡ u ÌAw,« ¨ 4w tA Rx qUu w 4 4u 4 w x¢ ¤4u u U 4¨ Rx As Ð Rx ¤U A w A s u x¢ 4 ¤4u u qU Í u x¢ º U 4uR [ s vk u ÌA 4u[xR Aw w ª+xRz º U Ry s skx¢


! " $# #$ % ' ( )*(*+-, .*/*0 1

! " $# #$ % 3 4 15)*6 7 .*/*0!8 9:. 15, 6 4*, 9;0

2

<= ! " $# #$ % > ? A@ B? #$ CB D$ E F ' ( )*(*+-, .*/*0 1 4%3 4*15) 6 7 .*/ 0%8 9;.*1H, 6 4 , 9:0 <= * > % " ?#$ D> ? D $B D I# ! > %@> ? B ' ( )*(*+-, .*/*0 1 4%3 4*15) 6 7 .*/ 0%8 9;.*1H, 6 4 , 9:0

G

L M=N$O:P Q R S E# ET ?D$U V . 9:6 0 1XW*)*Y-0AZ [*, ( 4 ? \ D % * E#? ] \ F$ # #$ ^ `_ ! $ #$ ! " $# $ 3 . b .*4 6dcH.feg9:.!h 0*(*7 4*6 4*1 i j k ? % j ' / )*.*9;/*0 cl4 (*m .*( > ! " #? $ n ? ]#? # C !F D$# >F p?q` E# E `_? > ! $ E ` * ` _ r ! $ $ ` * \ D$# >" % jB B #$ D$ ?# $ \ D ?

&

J*K J*G *a &*a a o K J K s 2 o 2 J


Sociedade Brasileira de MatemĂĄtica

t uwvx y z u

É com grande satisfação que informamos a excelente participação do Brasil na IMO. Pela primeira vez, os seis alunos da equipe ganharam medalhas, sendo 4 de prata e 2 de bronze. Na soma dos pontos, o Brasil ficou Ă frente de mais de 80% das naçþes representadas, incluindo muitos paĂ­ses de grande tradição olĂ­mpica. Veja a prova nesta edição e mais detalhes sobre a IMO no site http://imo.wolfram.com/

Isso só nos estimula ainda mais a fazer uma Eureka cada vez melhor e que atinja um público cada vez maior. Gostaríamos de agradecer mais uma vez o envio de grande número de problemas e soluçþes pelos leitores, o que Ê importante para a Eureka!, e nos anima a continuar o trabalho de proporcionar diversão e material de treinamento à nossa comunidade olímpica. Gostaríamos ainda de dizer que continuamos contando com o envio de artigos pelos leitores. Artigos para iniciantes são prioritårios. São tambÊm prioritårios artigos sobre temas olímpicos que atÊ hoje não apareceram na Eureka!, como Contagem, Grafos (nível avançado), Funçþes (nível intermediårio), Trigonometria aplicada à Geometria (pode ser uma compilação de problemas). Traduçþes (devidamente autorizadas, Ê claro) de artigos de boas revistas (com o mesmo perfil, Ê claro) serão muito bem recebidas. Finalmente agradecemos a valiosa ajuda dos professores Eduardo Tengan e Carlos Shine e dos estudantes Alex Cardoso Lopes, Guilherme I. C. Fujiwara e Rodrigo K. Yamashita na revisão da revista.

{x| }>~  €ƒ‚ „…}>| †

EUREKA! N°11, 2001

2


Sociedade Brasileira de MatemĂĄtica

‡ yˆy vAyˆ‰‹Š ÂŒ t t w‰ t yˆ Â?$ÂŽ Â?gÂ? ‘$Â? ÂŽ {‹’`“ Â” ‘$• _ r ! –

Sara escreveu no quadro negro um nĂşmero inteiro de menos de trinta algarismos e que termina em 2. CĂŠlia apaga o 2 do fim e escreve-o no inĂ­cio. O nĂşmero que fica ĂŠ igual ao dobro do nĂşmero que tinha escrito Sara. Qual ĂŠ o nĂşmero que Sara escreveu?

_ r ! — Vamos pegar um retângulo ABCD de papel; o lado AB mede 5 cm e o lado BC mede 9 cm. Fazemos trĂŞs dobras: 1- Levamos o lado AB sobre o lado BC e chamamos de P o ponto do lado BC que coincide com A. Forma-se entĂŁo um trapĂŠzio retângulo BCDQ. 2- Dobramos de forma que B e Q coincidam. Forma-se um polĂ­gono de 5 lados RPCDQ. 3- Dobramos de novo fazendo coincidir D com C e Q com P. Forma-se um novo trapĂŠzio retângulo RPCS. ApĂłs fazer estas dobras, fazemos um corte perpendicular a SC pelo seu ponto mĂŠdio T, obtendo o trapĂŠzio retângulo RUTS. Calcule a ĂĄrea da figura que aparece ao desdobrarmos o Ăşltimo trapĂŠzio RUTS.

_ r ! ˜

Temos três caixas, uma azul, uma branca e uma vermelha, e 8 bolinhas. Cada bolinha tem um número de 1 a 8, sem repetiçþes. Distribuímos as 8 bolinhas nas caixas, de maneira que hå pelo menos duas bolinhas em cada caixa. Logo, em cada caixa, somam-se todos os números escritos nas bolinhas contidas na caixa. Os três resultados denominam-se soma azul, soma branca e soma vermelha, segundo a cor da caixa correspondente. Encontre todas as possíveis distribuiçþes das bolinhas tais que a soma vermelha seja igual ao dobro da soma azul, e a soma vermelha menos a soma branca seja igual à soma branca menos a soma azul.

_ r ! ™

Utilizando exclusivamente números primos forma-se um conjunto com as seguintes condiçþes: 1Qualquer número primo de um algarismo pode estar no conjunto. 2Para que um número primo de mais de um algarismo esteja no conjunto, devem estar no conjunto o número que se obtÊm ao suprimir-lhe só o EUREKA! N°11, 2001

3


Sociedade Brasileira de MatemĂĄtica

primeiro algarismo e tambÊm o número que se obtÊm ao suprimir-lhe só o último algarismo. Determine, entre conjuntos que cumpram estas condiçþes, aquele que tem maior quantidade de elementos. Justifique por que não pode haver um com mais elementos. Lembre-se de que o número 1 não Ê primo.

_ r ! š Num tabuleiro de 8 casas, como na figura abaixo, hå inicialmente uma ficha em cada casa. Uma jogada consiste em escolher duas fichas e mover uma delas uma casa à direita e a outra, uma casa à esquerda. Se depois de 4 jogadas as 8 fichas estão distribuídas somente em 2 casas, determine quais podem ser estas casas e quantas fichas hå em cada uma delas.

_ r ! –

› ‘$œx� ’`ž`{‹’`“ ” ‘$•

Na minha calculadora, uma das teclas de 1 a 9 estĂĄ com defeito: ao pressionĂĄ-la aparece na tela um dĂ­gito entre 1 e 9 que nĂŁo ĂŠ o correspondente. Quando tentei escrever o nĂşmero 987654321, apareceu na tela um nĂşmero divisĂ­vel por 11 e que deixa resto 3 ao ser dividido por 9. Qual ĂŠ a tecla defeituosa? Qual ĂŠ o nĂşmero que apareceu na tela?

_ r ! — No trapÊzio ABCD, o lado DA Ê perpendicular às bases AB e CD. A base AB mede 45, a base CD mede 20 e o lado BC mede 65. Seja P no lado BC tal que BP mede 45 e seja M o ponto mÊdio de DA. Calcule a medida do segmento PM.

_ r ! ˜ Num tabuleiro de 3 fileiras e 555 colunas, pintam-se de vermelho 3 casas, uma em cada uma das 3 fileiras. Se escrevemos nas casas, ordenadamente por fileiras, da esquerda para a direita, os nĂşmeros de 1 a 1665 (na primeira fileira de 1 a 555, na segunda de 556 a 1110 EUREKA! N°11, 2001

4


Sociedade Brasileira de MatemĂĄtica

e na terceira de 1111 a 1665) hĂĄ 3 nĂşmeros que ficam escritos nas casas vermelhas. Escrevemos nas casas, ordenadamente por colunas, de cima para baixo, os nĂşmeros de 1 a 1665 (na primeira coluna de 1 a 3, na segunda de 4 a 6, na terceira de 7 a 9,‌, e na Ăşltima de 1663 a 1665) hĂĄ 3 nĂşmeros que ficam escritos nas casas vermelhas. Chamamos nĂşmeros vermelhos aos que em alguma das duas distribuiçþes ficam escritos nas casas vermelhas. Diga quais sĂŁo as 3 casas que devemos pintar de vermelho para que existam sĂł 3 nĂşmeros vermelhos. Mostre todas as possibilidades.

_ r ! ™ Em volta de um círculo situam-se dez moedas de 1 cm de raio como indicado na figura abaixo. Cada moeda Ê tangente ao círculo e às duas moedas vizinhas. Demonstre que a soma das åreas das dez moedas Ê o dobro da årea do círculo.

_ r ! š No quadro negro estĂŁo escritos os nĂşmeros naturais desde 1 atĂŠ 2001, inclusive. Temos que apagar alguns nĂşmeros de modo que entre os que ficam sem apagar seja impossĂ­vel escolher dois nĂşmeros distintos tais que o resultado de sua multiplicação seja igual a algum dos nĂşmeros que ficam sem apagar. Qual ĂŠ a quantidade mĂ­nima de nĂşmeros que devem ser apagados? Para esta quantidade, apresente um exemplo que mostre quais nĂşmeros sĂŁo apagados. Justifique por que nĂŁo obtemos a propriedade desejada se apagarmos menos nĂşmeros.

EUREKA! N°11, 2001

5


Sociedade Brasileira de Matemática

y y vAy t t ¡ ¢ t y £]¤>¥§¦l¨:© ª>«d¬ ­$® ª ¥*¯ ¨ ¤>¯ ® ¬ $ ` $ g { ` $ ° ± ²-³*´:µ ¶%µ ·%°?·*¸5· ¹*µ*·%º ¶*» ¸5³ ° ¶ µ*´;± Ã*¶!Ä*Å*²-± ¶%µ ·%Æ ³ Ç ÈÉ´;¶ Ë?¾*± ³ Ã*¶!µ*·!Ê ³*»*½ ³!À ³ ´:²5± ³!Æ ³Ì± ÍηfÈɳ ° » µ*Ð!¼%¶ ´:· ± ´:³!µ*·!Ä*± Ñ!³%·!º ± ½ Òd³ Ë?·*½ Ñ!¶%Ä »*± Ç Æ ¶*´;´:·*± ³!Ö5Å ¹*± ¶*´ ° ³ Ù*¾ ³*·*½-° ¶*µ ´:± Ã*» ·*Ç ¼%³ÚÈɳ Ë?¾*¶ Ñ%Ð Ç Û ± Üd¶ Ç5¾*±Hº ³ Ç5³*Ý-±5Û ¶ÌÇ5¾ ± ¹*³ ° ¶ µ*¶ ½ Þɶ%º ³ ¹fÈɶ Ç`Æ ¶ Ç Èɳ!¼%³ ß5³*´;³*¹*µ »*à ³ Ä ³*± Ç Ô Üd·*µ ³â× ± Òd³ ¸5¶*à ½ ¶fÒ ã ·*½ ± Ù ·%À ¶*¹*ß-³*½ Òd· Çä×=Ç5Ç5± Ç

¼%· µ*³*½ ¾ ³%µ ·%¿ »*´;¶ %¼ · µ*³*½ ¾ ³%µ ·%Ê ´;³fÈɳ ¼%· µ*³*½ ¾ ³%µ ·%Ê ´;³fÈɳ ¼!·*µ*³ ½ ¾*³!µ*·!Ó ´:¶ ¹*¸5· ¼!·*µ*³ ½ ¾*³!µ*·!Ó ´:¶ ¹*¸5· ¼!·*µ*³ ½ ¾*³!µ*·!Ó ´:¶ ¹*¸5· ¼!·*µ*³ ½ ¾*³!µ*·!Ó ´:¶ ¹*¸5· ¼%· ¹*ß5á ¶%Û?¶*¹*´;¶*Ç5³ ¼!·*¹*ß-á*¶!Û ¶*¹ ´:¶*Ç-³ ¼%· ¹*ß5á ¶%Û ¶ ¹*´;¶*Ç5³

À ¶*± Á ¹*± ³!Â>À ¿ À ¶*± Á ¹*± ³!Â>À ¿ Æ?¶*½ ³fÈ ± ¹*³!Â>Ï º Ô?¹*³*Õ Â>¼!À º ³ ¹fÈɶ$×=¹*µ ´:Ø!Â>º Ê º ³ ½ Òd³*µ ¶*´ Â>ÓÌ× °?± ¶%µ ·%Ö5³ ¹*· ± ´:¶!Â>° Ö À ¶*± Á ¹*± ³!Â>À ¿ º á ¶%Ö-¶*Ç5Ø!µ*¶ Ç`Æ ³*Ñ!Ù*¶ Ç Â º Ê Æ?³*Ñ!Ù*± ¹*³!À ´;³*¹ µ*·!Â>Ê Ó

¼%· µ*³*½ ¾ ³%µ ·%¿ »*´;¶ %¼ · µ*³*½ ¾ ³%µ ·%Ê ´;³fÈɳ ¼%· µ*³*½ ¾ ³%µ ·%Ê ´;³fÈɳ ¼!·*µ*³ ½ ¾*³!µ*·!Ó ´:¶ ¹*¸5· ¼!·*µ*³ ½ ¾*³!µ*·!Ó ´:¶ ¹*¸5· ¼!·*µ*³ ½ ¾*³!µ*·!Ó ´:¶ ¹*¸5· ¼!·*µ*³ ½ ¾*³!µ*·!Ó ´:¶ ¹*¸5· ¼%· ¹*ß5á ¶%Û ¶ ¹*´;¶*Ç5³ ¼!·*¹*ß-á*¶!Û ¶*¹ ´:¶*Ç-³ ¼!·*¹*ß-á*¶!Û ¶*¹ ´:¶*Ç-³

æ?± ÈÉ·*´;ç*±5 ° Ö ã ¶*´ Èɳ*½ · ¸5³!Â>Æ Ï À ¶*± Á ¹*± ³!Â>À ¿ º ³ ½ Òd³*µ ¶*´ Â>ÓÌ× ã ¶*´ Èɳ*½ · ¸5³!Â>Æ Ï º á ¶%Ö-¶*Ç5Ø!µ*¶ Ç`Æ ³*Ñ!Ù*¶ Ç Â º Ê °?± ¶%µ ·%Ö5³ ¹*· ± ´:¶!Â>° Ö À ¶*± Á ¹*± ³!Â>À ¿ º á ¶%Ê ³ »*½ ¶!Ârº Ê ã ¶*´ Èɳ*½ · ¸5³!Â>Æ Ï

$ x ` `{ ` $ ×=½ ·ÚÍXÆ ¶ ´:´:å ³â×=à ´:· » Ä ³*´:± Ç-Ç5³%Æ?³fÒd³ ½ ²5³*¹ÚÈÉ·%è é*µ ·%Ä̱ Ñ!³ À »*± ½ ¾*· ´:Ñ!·%°?¶*µ*´;± Ã*» ·*Ç º ³ ½ ·*´;Ñ%¶ ¿ Èɳ*²5Õ ½ ± ¶âË?¶*´;´:·*Ç ê ± ½ ³*Ç`Ó ¶ ³*Ç ë Ç-´:³*· ½5ã ´:³*¹ Ý5½ ± Ñíì ¶*» ´:³ µ*¶ Ä »*± Ç Ï µ »*³ ´:µ*¶!µ*·!À ¶ µ*¶ ± ã Ð*à ± ¶%ì ± ³ Ç ¼!¶*´;·*± ´:³ ×=¹ µ*´:Ø ± ³%Ä Å*²5± ¶!µ*·!Æ ³ Ç ÈÉ´:¶ Û · ¹*´ Ü îï· ±5Æ ¾*· ¹*Ã%Û Ç5» ì ³ÚÞɹ*·!µ*·$×=½ à*» ð*» ·*´:ð »*·%º ± Ñ%á ¶

EUREKA! N°11, 2001

6


Sociedade Brasileira de MatemĂĄtica

Ăą yˆy v%yÂ&#x;‰‹Š ÂŒ t t w‰ t E ‰nò Eyˆ t Ăł ¢u Ă´ÂĄv Enunciados e Resultado Brasileiro A XII OlimpĂ­ada de MatemĂĄtica do Cone Sul foi realizada na cidade de Santiago do Chile no perĂ­odo de 1 a 6 de julho de 2001. Dela participaram alunos de atĂŠ 15 anos dos seguintes paĂ­ses: Argentina, Brasil, Chile, Equador, Peru e Uruguai. A equipe brasileira foi selecionada atravĂŠs de provas realizadas em março e maio deste ano e foi liderada pelos professores Élio Mega e Carlos Yuzo Shine de SĂŁo Paulo - SP.

{ ÂŽ } | Ăľ]ܠ€ƒá ~ ‚¥~ á ‘$ø Ăľ  Ú } Ăş`„Ýá>|  Üg}d  Â„Â…á Ăş ÂŽ Ăź Ă˝ ‘  Þ |>€ƒ}  Þ¥~ ‚ÿ’`á | Ăż} Ăž €ƒ‚ „…† Ăş ÂŽ Ăź Âœ Ăľ  Ü }>„ Ăż} Ăľ j xá „…á Ăş ÂŽ Ăź • á „…  |>| á ` á á> Ăś dá>Ăž €ƒ } AĂľ }>  Â„Â… ‚ x~ } • ÿá Ăş ÂŽ Ăź ÂŽ á ƒá }>Ăś ž`á ‚ `  Â„Â… á ¥á Â?$ÂŽ`Â? Â? ‘$Â?gÂŽ {‹ž Â? ß ĂŹ Ă”?°r!Ă— " Âż # & ž*Âś ´:Âł Ç ¡!Ă‘%¡ Âą Âł*ĂŠ _ r ! –

Â?$ÂŽ`Ăź ]Ăź Â?$ÂŽ`Ăź ]Ăź { Â? ÂŽ {

Em cada casa de um tabuleiro quadriculado 2000 × 2000 deve-se escrever um dos três números: –1, 0 ou 1. Se, em seguida, somam-se os números escritos em cada linha e cada coluna, obtêm-se 4000 resultados. Mostre que Ê possível preencher o tabuleiro de modo que os 4000 resultados assim obtidos sejam todos distintos.

_ r ! —

Tem-se uma sucessĂŁo a1, a2, a3, ..., an, ... de nĂşmeros inteiros positivos, com as seguintes propriedades: i) Todo nĂşmero inteiro positivo aparece uma ou mais vezes na sucessĂŁo. ii) a1 = 1 iii) a3n+1 = 2an + 1 iv) an+1 ≼ an v) a2001 = 200 Calcule o valor de a1000. Obs. O enunciado deste problema estĂĄ incorreto, pois na verdade nĂŁo existe tal sequĂŞncia (tente demonstrar isto!). Entretanto, se suprimirmos a condição i) ĂŠ possĂ­vel resolver o problema (tente fazer isto tambĂŠm!). EUREKA! N°11, 2001

7


Sociedade Brasileira de MatemĂĄtica

_ r ! ˜ TrĂŞs triângulos acutângulos estĂŁo inscritos em uma mesma circunferĂŞncia, de modo que seus vĂŠrtices sĂŁo nove pontos distintos. Demonstre que se pode escolher um vĂŠrtice de cada triângulo de maneira que os trĂŞs pontos escolhidos determinem um triângulo cujos ângulos sejam menores que ou iguais a 90o.

› ‘$ÂœxÂ? Â’ ž`{‹ž Â? ß ĂŹ Ă” °rĂ—! " Âż$# & ž*Âś ´:Âł*Ç ¡%Ă‘!¡*Âą Âł ĂŠ _ r ! ™ Um polĂ­gono de ĂĄrea S estĂĄ contido no interior de um quadrado de lado a. Demonstre que hĂĄ pelo menos dois pontos do polĂ­gono que estĂŁo separados por uma distância maior que ou igual a Sa .

_ r ! š Ache todos os números inteiros positivos m tais que m + 2001⋅S(m) = 2m onde S(m) representa a soma dos algarismos de m.

_ r ! &% Seja g uma função definida para todo inteiro positivo n, que satisfaz i) ii) iii) iv)

g(1) = 1 g(n + 1) = g(n) + 1 ou g(n + 1) = g(n) – 1 para todo n ≼ 1 g(3n) = g(n) para todo n ≼ 1 g(k) = 2001 para algum inteiro positivo k.

Ache o menor valor possível de k entre todas as funçþes g que cumprem as condiçþes anteriores e demonstre que Ê o menor.

EUREKA! N°11, 2001

8


>F p('$ # `_ ! –

Sociedade Brasileira de MatemĂĄtica

Antes de procurarmos uma maneira de preencher um tabuleiro 2000 Ă— 2000, vamos preencher um tabuleiro menor, digamos, 2 Ă— 2. Uma maneira de preenchĂŞ-lo ĂŠ: 1

–1

→ 1 soma: 0

1

0

→ 1 soma: 1

↓

↓

soma: soma: 2 –1

Vamos tentar preencher agora um tabuleiro 4 × 4. Para isso, aproveitamos o tabuleiro 2 × 2. Para que as somas obtidos continuem iguais, colocamos 1's e –1's desta forma: → soma: 0

1

–1

1

–1

1

0

1

– 1 → soma: 1

1

1

–1

–1

↓

↓

soma: soma: 2 –1

Para completar o trabalho, basta preencher o subtabuleiro 2 × 2 do canto inferior direito: → soma: 0

1

–1

1

–1

1

0

1

– 1 → soma: 1

1

1

1

0

→ soma: 3

–1

–1

1

–1

→ soma: –2

↓

↓

↓

↓

soma: soma: soma: soma: 2 –1 4 –3

EUREKA! N°11, 2001

9


Sociedade Brasileira de MatemĂĄtica

Observe que podemos preencher um tabuleiro (2k + 2) × (2k + 2) a partir de um tabuleiro 2k × 2k. Digamos que no tabuleiro 2k × 2k as somas sejam iguais a – (2k – 1), – (2k – 2), ‌2k. Para preencher um tabuleiro (2k + 2) × (2k + 2), colocamos o tabuleiro 2k × 2k no canto superior esquerdo, preenchemos as 2k primerias casas da 2k + 1-Êsima linha e da 2k + 1-Êsima coluna con 1's e as 2k primeiras casas da 2k +-Êsima linha e da 2k + 2-Êsima coluna com –1's. No canto interior direito preenchemos da mesma forma que anteriormente:

2k Ă— 2k

1

1

–1

1

–1

)

)

1

–1 0 → soma:2k + 1

Nas linhas e colunas em (A) e (B) temos as somas de –(2k – 1) a 2k

(A)

1

‌

1

1

–1 –1

‌

–1

1 –1 → soma: –2k

(B)

↓

↓

soma soma 2k + 2 –(2k + 1)

Assim, temos todas as somas de (2k + 1) a 2k + 2. Logo ĂŠ possĂ­vel preencher um tabuleiro 2n Ă— 2n para todo n natural nĂŁo nulo; em particular, o tabuleiro 2000 Ă— 2000.

>F p('$ # `_ ! —

O enunciado do problema estĂĄ incorreto: Vamos mostrar que nĂŁo existe uma tal sucessĂŁo. Temos que a n +1 = a n ou a n +1 = a n + 1 (observe que, se a n +1 ≼ a n + 2, o nĂşmero inteiro positivo a n + 1 nĂŁo apareceria na sucessĂŁo). O nĂşmero da forma 3n + 1 mais prĂłximo de 2001 ĂŠ 2002 = 3â‹… 667 + 1. AlĂŠm disso, a 2002 = a 2001 = 200 ou a 2002 = a 2001 + 1 = 201. Sendo a 2002 = 2a 667 + 1, temos que a2002 ĂŠ Ă­mpar, logo a2002 = 201. Assim, 2a667 + 1 = 201 ⇔ a667 = 100. Mas 667 = 3 â‹… 222 +1, e temos 2a222 + 1 = 100 ⇔ a222 = 99 , que nĂŁo ĂŠ inteiro. Contradição. 2 TrĂŞs alunos da delegação brasileira deram uma solução equivalente Ă anterior. Alguns alunos argentinos resolveram o problema ignorando a condição (i). Mostremos que ĂŠ possĂ­vel calcular a2000 sem utilizar a condição (i). EUREKA! N°11, 2001

10


Sociedade Brasileira de MatemĂĄtica

De (iv), indutivamente mostra-se que a m ≼ a n para m ≼ n. Aplicando (iii) temos: a 4 = a 3⋅1+1 = 2a1 + 1 = 2 ⋅ 1 + 1 = 3 a13 = a 3⋅4 +1 = 2a 3 + 1 = 2 ⋅ 3 + 1 = 7 a 40 = a 3⋅13+1 = 2a13 + 1 = 2 ⋅ 7 + 1 = 15 a121 = a 3⋅40 +1 = 2a 40 + 1 = 2 ⋅ 15 + 1 = 31 a 364 = a 3⋅121+1 = 2a121 + 1 = 2 ⋅ 31 + 1 = 63 a1093 = a 3⋅364 +1 = 2a 364 + 1 = 2 ⋅ 63 + 1 = 127

Logo a1000 ≤ a1093 ⇔ a1000 ≤ 127

(I)

Agora estudemos a3. Seja a 3 = k . Temos: a10 = a 3⋅3+1 = 2a 3 + 1 = 2k + 1 a 31 = a 3⋅10 +1 = 2a10 + 1 = 2(2k + 1) + 1 = 4k + 3 a 94 = a 3⋅31+1 = 2a 31 + 1 = 2(4k + 3) + 1 = 8k + 7 a 203 = a 3⋅94 +1 = 2a 94 + 1 = 2(9k + 7) + 1 = 16k + 15 a 650 = a 3⋅283+1 = 2a 283 + 1 = 2(26k + 15) + 1 = 32k + 31 a 2561 = a 3⋅850 +1 = 2a850 + 1 = 2(32 k + 31) + 1 = 64k + 63 Como a 2551 ≼ a 2001 , temos 64k + 63 ≼ 200 ⇔ k ≼ 3. Mas a 3 ≤ a 4 ⇔ k ≤ 3. Logo k = 3 e, portanto, a850 = 32 ⋅ 3 + 31 = 127. Desta forma a1000 ≼ a850 ⇔ a100 ≼ 127(II). De (I) e (II), temos a1000 = 127.

>F p('$ # `_ ! ˜ Sejam A1 A2 A3, B1 B2 B3 e C1 C2 C3 os triângulos. Tome o ponto A1 e trace o diâmetro AA1 que passa por A1. Podemos supor, sem perda de generalidade, que, dentre os pontos B1, B2, B3, C1, C2 e C3, o mais prĂłximo de A ĂŠ B1 e que, dentre os ∊

pontos C1, C2 e C3, o mais prĂłximo de A, contido no arco AA1 que contĂŠm B1 ĂŠ C1.

EUREKA! N°11, 2001

11


Sociedade Brasileira de MatemĂĄtica

C1 B1

A1

O

A

C

O diâmetro C1 C que passa por C1 divide a circunferĂŞncia em dois arcos. Se C1, C2 e C3 estivessem no arco que contĂŠm A1, terĂ­amos que o maior ângulo do 180° = 90° , o que nĂŁo ĂŠ possĂ­vel. Logo triângulo C1C2C3 seria maior ou igual a 2 existe um ponto, Ci no arco que contĂŠm B1. Sendo C1 o mais prĂłximo de A, e ∊

∊

como AC1 que contĂŠm B1, temos que Ci pertence ao arco CA . Temos entĂŁo que o triângulo A1B1Ci satisfaz as condiçþes do enunciado, pois ∊

∊

∊

∊

∊ m(Ci A1 ) m( A1 A)  ∊  m (B A ) m ( AA1 ) = 90°, m( A1 B1 Ci ) = m A1 C i B1  = a 1 1 < a ≤ = 90° 2 2 2 2   ∊

∊

∊

m(C i B1 ) m(CC1 ) < = 90°. Observe que o triângulo Ê retângulo e m(C i A1 B1 ) = 2 2 se, e somente se, Ci = A. Obs. Como o número total de pontos Ê 9, que Ê ímpar, podemos escolher um deles (que serå o A1) cujo antípoda não Ê nenhum dos outros pontos. Nossa solução mostra então que Ê possível obter um triângulo acutângulo AiBjCk.

>F p('$ # `_ ! ™ Suponha, por absurdo, que quaisquer dois pontos do polĂ­gono estejam separados por uma distância menor que S/a.

EUREKA! N°11, 2001

12


Sociedade Brasileira de MatemĂĄtica

y a

x

b

Consideramos o ponto mais Ă esquerda e mais Ă direita x e y do nosso polĂ­gono. Se b ĂŠ a diferença de suas abscissas, o polĂ­gono estĂĄ contido num retângulo de lados b e a. Temos claramente S ≤ ab, donde b ≼ S/a. Como claramente xy ≼ b, temos um absurdo e o resultado estĂĄ provado.

>F p('$ # `_ ! š Temos m + 2001 S(m) = 2m ⇔ 2001 S(m) = m. Assim, m ĂŠ divisĂ­vel por 3 e conseqĂźentemente S(m) tambĂŠm o ĂŠ. Logo S(m) = 3k, para algum k inteiro, e m = 2001 â‹… 3k = 9 â‹… 667k ĂŠ divisĂ­vel por 9. Desta forma, 9 divide S(m). Seja n o nĂşmero de algarismos de m. temos que S(m) ≤ 9n (cada algarismo ĂŠ menor ou igual a 9), assim 2001S(m) ≤ 18009n ⇔ m ≤ 18009n. n n Mas m ≼ 100 , + ...0 = 10 , logo 10 ≤ 18009n. n −1 zeros *

Esta Ăşltima desigualdade sĂł ĂŠ vĂĄlida para n ≤ 6. Assim, S(m) ≤ 9 â‹… 6. Como S(m) ĂŠ divisĂ­vel por 9, temos S(m) = 9, 18, 27, 36, 45 ou 54. Temos S(m) = 9 ⇒ m = 9â‹…2001 = 18009, o que nĂŁo ĂŠ possĂ­vel pois 1+8+0+0+9 = 18. S(m) = 18 ⇒ n = 18â‹…2001 = 36018. S(m) = 27 ⇒ m = 27â‹…2001 = 54027, o que nĂŁo ĂŠ possĂ­vel pois 5+ 4+0+2+7 = 18 S(m) = 36 ⇒ m = 36â‹…2001 = 72036, o que nĂŁo ĂŠ possĂ­vel pois 7+2+0+3+6=18 S(m) = 45 ⇒ m = 45â‹…2001 = 90045, o que nĂŁo ĂŠ possĂ­vel pois 9+0+0+0+4+5=18 S(m) = 54 ⇒ m = 54â‹…2001 = 108054, o que nĂŁo ĂŠ possĂ­vel pois 1+0+8+0+5+4=18 Logo a Ăşnica solução ĂŠ n = 36018.

EUREKA! N°11, 2001

13


>F p('$ # `_ ! &%

Sociedade Brasileira de MatemĂĄtica

Queremos encontrar o menor valor de k tal que g(k) = 2001. Assim, seja ak o menor valor tal que g(ak) = n. Desta forma, devemos calcular a2001. Temos g(1) = 1, logo a1 = 1. Podemos tomar g(2) = 2, logo a2 = 2. Observe que nesse caso g(3) = g(1) = 1 e g(6) = g(2) = 2. Podemos tomar g(4) = 2 e g(5) = 3, logo a3 = 5. Considere g(k) e g(k + 1). Temos g(3k) = g(k) e g(3k + 3 ) = g(k + 1). Assim, g(3k + 3) = g(3k) + 1 ou g(3k + 3) = g(3k) – 1. Desta forma, g(3k +1) e g(3k + 2) sĂŁo no mĂĄximo iguais a g(k) + 1 ou g(k + 1) + 1. Por exemplo, se g(4) = 2 e g(5) = 3, temos g(12) = 2 e g(15) = 3. Tomando g(13) = 3 e g(14) = 4, temos que a4 = 14. Note que, como g(k) ≤ 2 para k ≤ 4, temos g(k) ≤ 3 para k ≤ 12. Assim, podemos encontrar an + 1 em função de an. Temos que g(an – 1) = n – 1 pois g(an – 1 ) < n (se g(an – 1) > n, existiria k < an tal que g(k) = n, o que contradiz a hipĂłtese de an ser mĂ­nimo. Assim, g(3(an – 1) = n – 1 e g(3an) = n. Para k ≤ an – 1, temos g(k) ≤ n – 1, logo g(k) ≤ n para k ≤ 3(an – 1). Podemos tomar g(3an – 2) = n e g(3an – 1) = n + 1. Logo an + 1 = 3an – 1. Note que temos "quase" uma progressĂŁo geomĂŠtrica. Se somarmos um nĂşmero x x −1  de cada lado da igual dado, temos a n +1 + x = 3a n −1 + x = 3 a n +  . Se 3   1 x −1 1 1  1  ⇔ x = − , temos  a n +1 −  = 3 a n −  . Sendo bn = a n − , fizermos x = 3 2 2 2  2  temos bn+1 = 3bn, ou seja , bn ĂŠ uma progressĂŁo geomĂŠtrica de razĂŁo 3. Logo bn = 1 1 b1 â‹… 3n – 1. Como b1 = a1 − = , temos 2 2 n −1 n−1 3 2000 + 1 3 1 3 3n−1 + 1 ⇔ an − = ⇔ an = . . Para n = 2001, temos a n = bn = 2 2 2 2 2 Obs. A função g que construĂ­mos ĂŠ tal que g(k) ĂŠ o maior possĂ­vel, para todo k, e ĂŠ obtida da seguinte forma: Primeiro observamos que todo natural n pode ser escrito de maneira Ăşnica como n = 3 k + Îľ 1 3 k −1 + Îľ 2 3 k − 2 + ... + Îľ k −1 â‹… 3 + Îľ k , com

Îľ j ∈{−1,0,1} para 0 ≤ j < k (onde k ĂŠ tal que g (n) = 1 +

3k + 1 3 k +1 + 1 ≤n< ) . Temos então 2 2

k −1

âˆ‘Îľ

i

,ou seja, g(n) Ê o número de termos não nulos na representação

j =0

acima.

EUREKA! N°11, 2001

14


Sociedade Brasileira de MatemĂĄtica

Ăą vxyÉy`  vAyˆ‰‹Š ÂŒ t t yÂ&#x; z t yÂ&#x; t v w‰ t E ‰nò yˆ t Enunciados e Resultado Brasileiro A XLII OlimpĂ­ada Internacional de MatemĂĄtica foi realizada na cidade de Washington – DC, USA no perĂ­odo de 1 a 14 de julho de 2001 e teve a participação de 85 paĂ­ses. A equipe brasileira foi selecionada atravĂŠs de provas realizadas em março e maio deste ano e foi liderada pelos professores Nicolau C. Saldanha (Rio de Janeiro – RJ) e Antonio Caminha Muniz Neto (Fortaleza - CE).

{ ÂŽ } | Ăľ]ܠ€ƒá ~ ‚¥~ á ‘$ø Ăľ  Ú } Ăş`„Ýá>|  Üg}d  Â„Â…á Ăş ÂŽ Ăź Ă˝ Ăş ÂŽ Ăź Ăş ÂŽ Ăź Ăş ÂŽ Ăź Ăş ÂŽ Ăź 2 Ăş ÂŽ Ăź 4

Ăź  Ü } - `‚ „…„/.>á Ăź10 „…}>Ăľ `á „…ܠ‚ | ›$€ƒ}  Þ¥Â’`á }>| ~]} Ăş`„…  Â€ÂƒÂ‚ | ‚]| Â€Âƒá  á ‚ÿú`á>„Ý„…‚ | ÂŽ ‚ ~ „… Ăľ]} ` Ăľ 0 }>Â„Â…Â€ÂƒÂ‚ĂżÂ›$  Ü >á Â’` á } | ž` á Â? 3 - ¥‚ Ăź Ü } ->á Ăž ~]„Ý gĂž ‚ÿ’` ‚ Ăľ]}  Â„Â…á ž`á Ăž  }>Ăś Â?  5Ăž }  Â„…‚¥›$‚ 0 „…}  Â„Â…á

Â? „…á €ƒá Â? „…á €ƒá Ăş`„Ý‚] Ăž } Â? „…á €ƒá Â? „…á €ƒá Ăş`„Ý‚] Ăž }

Â?$ÂŽ`Â? Â? ‘$Â?gÂŽ {‹ž  Â? Ăź ĂŹ Ă”?°r!Ă— " Âż # a ž* Âś ´:Âł Ç ¡!Ă‘%¡ Âą Âł*ĂŠ _ r ! – Seja ABC um triângulo acutângulo com circuncentro O. Seja PA uma altura do triângulo com P no lado BC. Considere que BCˆ A ≼ ABˆ C + 30° . Prove que CAˆ B + COˆ P < 90° .

_ r ! — Prove que a a + 8bc 2

+

b b + 8ca 2

+

para quaisquer nĂşmeros reais positivos a, b, e c.

EUREKA! N°11, 2001

15

c c + 8ab 2

≼1


_ r ! ˜

Sociedade Brasileira de MatemĂĄtica

Vinte e uma meninas e vinte e um meninos participaram numa competição matemĂĄtica. • Cada participante resolveu no mĂĄximo seis problemas. • Para cada menina e cada menino, existe pelo menos um problema que foi resolvido por ambos. Prove que existe um problema que foi resolvido por pelo menos trĂŞs meninas e pelo menos trĂŞs meninos.

› ‘$ÂœxÂ? Â’ ž`{‹ž Â? ß ĂŹ Ă” °rĂ—! " Âż$# a ž*Âś ´:Âł*Ç ¡%Ă‘!¡*Âą Âł ĂŠ

_ r ! ™ Seja n um inteiro Ă­mpar maior do que 1 e sejam k1 , k 2 ,..., k n inteiros dados. Para cada uma das n! permutaçþes a = (a1 , a 2 ,..., a n ) de {1,2,..., n}, defina S (a) =

n

∑k a . i

i

i =1

Prove que existem duas permutaçþes b e c, b ≠c, tais que n! ĂŠ um divisor de S (b) − S (c).

_ r ! š Num triângulo ABC, seja AP a bissectriz de BAˆ C com P no lado BC, e seja BQ a bissectriz de ABˆ C com Q no lado CA. Sabemos que BAˆ C = 60° e que AB + BP = AQ + QB. Quais sĂŁo os possĂ­veis valores dos ângulos do triângulo ABC?

_ r ! &% Sejam a, b, c, d inteiros com a > b > c > d > 0. Considere que ac + bd = (b + d + a − c)(b + d − a + c). Prove que ab + cd ĂŠ um nĂşmero primo.

EUREKA! N°11, 2001

16


Sociedade Brasileira de MatemĂĄtica

9

t E z z ¢ 76 t 8 y ÂŞ>Žƒ¨ ÂŹ>ÂĽ;:$ÂŚ=< ÂŹ?>A@ ÂŻ B ¤?C 9 ÂŹ>¨ D E ÂŻ ÂŹ?F$Š ÂŞ G>ÂŞ

$ÂŽfŠ ÂŻ E>ÂŹ I>ÂŞdÂĽ*¤ ÂŞ ÂŤ>ÂŹI¤ J ÂŞ ÂŚ5¨ ÂŞ JIÂŻ B ÂŻ ÂĽ Š ÂŽÂƒÂŞ>ÂŤ ÂŞKB>ÂŞML/NO>r¤ J ÂŞPB ÂŞ?Q=¨SR J GdÂŻ T*ÂŞ U(>rÂŞ ¨SVfÂŞdÂŤ>ÂŹ ÂŽ$CĂŒ­WH H

♌ XZY []\ ^`_ a b cdb ePa f \(g A Torre de HanĂłi ĂŠ um dos quebra-cabeças matemĂĄticos mais populares. Ele foi inventado por Edouard Lucas em 1883.

Ă˝ † Â? } h>á |

As peças são n discos de tamanhos diferentes e todos com um furo em seu centro e três pinos onde são colocados os discos. Certamente podem ser encontrados em qualquer loja de brinquedos.

† ÂŽ } „…á | } ‚ 0  }>€ƒ ‚ | ~ ‚ g‚ ‚

Inicialmente os discos formam uma torre onde todos sĂŁo colocados em um dos pinos em ordem decrescente de tamanho.

Devemos transferir toda a torre para um dos outros pinos de modo que cada movimento ĂŠ feito somente com um disco, nunca havendo um disco maior sobre um disco menor.

† Ăź Â? } „ Ăľ Ăž]€ƒá ø Ăľ]} |>} „ 3 >ádÜ á>~ á

Queremos saber qual Ê o menor número de movimentos necessårios para resolver uma torre de Hanói com n discos. Hå uma história (imaginada pelo próprio Edouard Lucas) sobre a torre de Hanói: No começo dos tempos, Deus criou a Torre de Brahma, que contÊm três pinos de diamante e colocou no primeiro pino 64 discos de ouro maciço. Deus então chamou seus saserdotes e ordenou-lhes que transferissem todos os discos para o terceiro pino, seguindo as regras acima. Os sacerdotes então obedeceram e começaram o seu trabalho, dia e noite. Quando eles terminarem, a Torre de Brahma irå ruir e o mundo acabarå. EUREKA! N°11, 2001

17


† ‘ | Â€ÂƒĂľ ~]á Ăž ~ ‚¥‚ÿÚ Â„Â…Â‚ 0  Ü } ¥á

Sociedade Brasileira de MatemĂĄtica

Para resolver um problema (não só este, mas vårios outros problemas na matemåtica) que envolve n coisas, ajuda ver o que acontece para valores pequenos de n. Vejamos alguns casos. • n = 1. Fazemos

1 movimento foi suficiente. • n = 2. Fazemos

3 movimentos deram. • n = 3. Fazemos

7 movimentos deram. Mas Ê claro que não podemos fazer só isso. Não podemos ficar observando o que acontece para todos os valores de n! Então temos que começar a tirar algumas conclusþes.

EUREKA! N°11, 2001

18


Sociedade Brasileira de MatemĂĄtica

2

† ` ‚ ÿ‚¥„Ý}>| ‚ Ăś } „$‚¥Ú „…‚ 0  Ü } á ‚

i

~  | ‚ | j

Vamos olhar o caso n = 3 mais perto. Observe os trĂŞs primeiros movimentos:

Note que o que fizemos foi mesmo para resolver o caso n = 2. O prĂłximo movimento foi

Isto ĂŠ, passamos o disco maior para o pino sem discos. Agora, veja os trĂŞs Ăşltimos movimentos:

Novamente fizemos o mesmo que foi feito para o caso n = 2, só que transferindo agora a "subtorre" para o pino onde estava o disco maior. Agora, imaginemos uma torre com n discos. Imagine tambÊm que sabemos resolver o problema com n – 1 discos. k

n discos

Podemos transferir os n – 1 discos de cima para um pino vazio: vårios movimentos

k

n–1 discos

EUREKA! N°11, 2001

19

k


Sociedade Brasileira de MatemĂĄtica

Depois passamos o disco maior para o outro pino vazio:

l

Por fim, colocamos os n – 1 discos menores sobre o disco maior: vårios movimentos n–1 discos

m

m

Assim, podemos resolver o problema com n discos. Por exemplo, para resolver o problema com 4 discos, transferimos os 4 – 1 = 3 discos de cima para um pino vazio (jå sabemos fazer isso!), depois passamos o disco maior para o outro pino vazio e por fim colocamos os 3 discos sobre o disco maior. Para resolver o problema com 5 discos, transferimos os 5 – 1 = 4 discos de cima para um pino vazio (acabamos de aprender a fazer isso!), e assim por diante.

† ž`á Ăž ~ ‚¥Þ ‚ ÂĄ} á>‚ | 0 ]‚  |

4

Voltemos à pergunta que serå calada: queremos saber o número mínimo de movimentos necessårios para resolver uma torre de Hanói com n discos. Vamos dar um nome para este número, digamos Tn. Assim, o número mínimo de movimentos necessårios para resolver um problema com 1 disco Ê T1, com 2 discos Ê T2, com 2001 discos Ê T2001, com ♼ discos Ê T♼, e, em especial, com n – 1 discos Ê Tn – 1. n

† ”$‚ ܠ€ƒá>Ăž ~ ‚ÿá>‚ÿÚ Â„Â…Â‚ 0  Ü } á

JĂĄ vimos que podemos resolver o problema da seguinte forma: vĂĄrios movimentos

o

n–1 discos

EUREKA! N°11, 2001

20

o


Sociedade Brasileira de Matemática

p

vários movimentos n–1 discos

p

p

Vamos ver quantos movimentos são necessários neste modo de resolver o problema. Precisamos de Tn – 1 movimentos para movimentar os n – 1 primeiros discos, mais um para movimentar o disco maior e mais Tn – 1 para colocar os n – 1 discos sobre o disco maior. Assim, precisamos de Tn – 1 + 1 + Tn – 1 = 2Tn – 1 + 1 movimentos. Mas não sabemos se este modo de resolver o problema usa o menor número de movimentos; poderia haver outro modo que use menos movimentos. Como o menor número de movimentos é Tn, temos: (I) Tn ≤ 2Tn −1 + 1 Provemos que na verdade Tn = 2Tn −1 + 1. Para isso, mostraremos que Tn ≥ 2Tn −1 + 1 (lembre-se de que se a ≤ b e a ≥ b então a = b). Esta aparentemente estranha maneira de se demonstrar que uma coisa é igual a outra é na verdade bem comum em vários problemas. Muitas igualdades podem ser obtidas a partir de desigualdades. Considere agora, então, o disco maior. Ele vai ter que sair da torre inicial uma hora. Mas para ele sair, é preciso que os outros n – 1 discos saiam de cima dele! E mais, se quisermos mudá-lo de lugar ele vai ter que ir para um pino vazio, pois ele não pode ficar sobre nenhum dos outros discos por ser o maior (que trabalho esse disco dá!)! Logo precisamos transferir os n – 1 discos para um pino só, o que requer no mínimo Tn – 1 movimentos. Para mudarmos ele de lugar, precisamos, é claro, de mais um movimento. E depois, para colocarmos os n – 1 discos sobre o disco maior precisamos no mínimo mais Tn – 1 movimentos. Assim, para resolver o problema precisamos na verdade de no mínimo Tn −1 + 1 + Tn −1 = 2Tn −1 + 1 movimentos. Logo

EUREKA! N°11, 2001

21


Sociedade Brasileira de Matemática

Tn ≥ 2Tn −1 + 1

(II)

Assim, de (I) e (II), (*) Tn = 2Tn −1 + 1 Assim, como T1 = 1 (é só ver o caso n = 1), podemos, fazendo n = 2, concluir que T2 = 2T1 + 1 = 2 ⋅ 1 + 1 = 3 (exatamente como achamos antes!!) e, fazendo n = 3, descobriríamos que T3 = 2T2 + 1 = 2 ⋅ 3 + 1 = 7 (que coisa!). Para n = 4, acharíamos T4 = 2T3 + 1 = 2 ⋅ 7 + 1 = 15. Se quiséssemos então Tn para um valor qualquer de n, devemos ter todos os valores de Tk para k = 1, 2, …, n – 1, mas com certeza é possível calcular. Uma seqüência deste tipo (isto é, tal que para calcular um dos valores usamos os valores anteriores) é chamada recorrente e a equação que relaciona os termos da seqüência é chamada de relação de recorrência (no caso, temos que (*) é uma equação de recorrência).1 Poderíamos parar por aqui (pois já sabemos como calcular os valores de Tn ), mas encontraremos uma fórmula para Tn que não depende de seus valores anteriores (tal fórmula é costumeiramente chamada fórmula fechada). Nem sempre se pode (e quando se pode, pode ser bem difícil) fazer isso com uma relação de recorrência, mas com esta em particular pode ser feita. Observe que temos "quase" Tn = 2Tn −1 . Vamos ver se podemos acertar isso. Se somarmos um número x aos dois lados da equação (*), temos 1+ x   Tn + x = 2Tn −1 + 1 + x ⇔ Tn + x = 2 ⋅  Tn −1 +  2   Se fizermos x = (1 + x) / 2 ⇔ x = 1 e sendo An = Tn + 1, temos An = 2 An −1 = 2 ⋅ 2 An − 2 = 2 2 An − 2 = 2 2 ⋅ 2 An −3 = 2 3 An −3 = ... = 2 n −1 A1 Como A1 = T1 + 1 = 1 + 1 = 2, temos An = 2 n . Assim, An = Tn + 1 ⇔ 2 n = Tn + 1 ⇔ Tn = 2 n − 1 Assim, precisamos de 2 n − 1 movimentos para resolver o problema da torre de Hanói com n discos. Ou seja, os sacerdotes precisarão de 2 64 − 1 movimentos. Mesmo se eles fizessem um movimento por segundo, eles precisariam de mais de 500 bilhões de anos!! Podemos ficar tranqüilos por enquanto.

1

Para outros comentários e resultados sobre recorrência veja o artigo "Equações de Recorrência", de Héctor Soza Pollman, publicado na revista Eureka! No. 9

EUREKA! N°11, 2001

22


† { 0 | }>„/ >á h rd‚ÿ ÂĄĂš ‚ „…€ƒá Ăž]€ƒ} q

Sociedade Brasileira de MatemĂĄtica

Os alunos mais observadores devem ter notado de antemĂŁo que Tn = 2 n − 1 bem antes, quando calculamos Tn para valores pequenos de n. Ter essa percepção ĂŠ bom, mas sĂł perceber que Tn = 2 n − 1 nĂŁo ĂŠ suficiente. É preciso provar que esta relação realmente ĂŠ verdadeira. As aparĂŞncias podem enganar!! Por exemplo, considere a seqßência n(n − 1)(n − 2)...(n − 2000) +n 2001! (lembre-se : 2001! = 1â‹… 2 â‹… 3 â‹… ‌⋅ 2001) Temos a1 = 1, a 2 = 2,..., a 2000 = 2000. Isto poderia nos levar a crer que a n = n, nĂŁo? Pois veja quanto vale a 2001 e vocĂŞ terĂĄ uma bela surpresa! an =

‘ - }>„/ s g‚ |

01. Encontre uma fĂłrmula fechada para cada uma das relaçþes de recorrĂŞncia a seguir: a) a n = 3a n −1 + 4, a1 = 0 b) bn = 2bn −1 + 3 , b1 = 5 02. (Prova de Seleção para a IMO e OlimpĂ­ada Iberoamericana 2001, adaptada) Seja f uma função de t em t tal que, para todos x, y, z reais, f ( x + y ) + f ( y + z ) + f ( z + x) ≼ 3 f ( x + 2 y + 3 z ) a) Mostre que f (a) ≼ f (0) para todo a real. u v Mostre que f (a) ≤ f (0) para todo a real e conclua que as funçþes f onde f (a) = f (0) sĂŁo as Ăşnicas soluçþes do problema.

{ 0 | }>„/ >á h rd‚$w A grosso modo, uma função f de um conjunto A em um outro conjunto B, ĂŠ uma relação que toma cada elemento x de A e o transforma em um elemento f(x) de B. As equaçþes de recorrĂŞncia que acabamos de estudar sĂŁo exemplos de funçþes de N em R. 03. Na torre de HanĂłi, suponha que em vez de transferir a torre para um dos pinos, vocĂŞ tenha que transferir a torre para cada um dos outros pinos uma vez. Encontre o nĂşmero mĂ­nimo de movimentos para resolver esse problema.

EUREKA! N°11, 2001

23


Sociedade Brasileira de MatemĂĄtica

z yyx n‰‹ ! z y t w u yzx Ă´ t vx t u ‰ Š z { vx ‰ t u¢ w nvxyÂ&#x;‰ Š ÂŒ t t u £ä(ÂŞ | ÂŞ>¤ ~¨ }ĂŒÂŞÂ€ ÂŽ ÂŞM ĂŒÂŹ B?Š ¤ ¨ ¤ ÂĽ ♌ XZY []\ ^`_ a f \ ‚Sƒ \ „ b Â… ‚ b †(g O que desejamos mostrar com esse texto ĂŠ o potencial significativo da trigonometria para resolver problemas de olimpĂ­adas de matemĂĄtica, principalmente quando combinada com algumas desigualdades. Esse texto, apĂłs a resolução de cada exemplo, apresenta um esquema da mesma, um Guia de Resolução, o qual busca facilitar o entendimento geral do que foi feito. Caso o leitor queira tentar resolver tais exemplos antes de conhecer a resolução descrita aqui, poderĂĄ recorrer a este guia, como instrumento auxiliar. Vamos aos exemplos.

*< ! > C– :

(Seleção para IMO 99 – Brasil ) Para reais positivos satisfazendo 1 1 1 3 + + ≤ , e determine a + b + c = abc, mostre que 2 2 2 2 1+ a 1+ b 1+ c quando a igualdade ocorre.

>F p('$ ‡ A idĂŠia bĂĄsica para resolver esse problema ĂŠ fazer uso da transformação de um nĂşmero real em tangente de outro. Isso vem do simples fato de que qualquer nĂşmero real a pode ser representado pela tangente de outro nĂşmero real Îą pertencente ao intervalo (–π/2, Ď€/2), sendo tal Îą Ăşnico – isso ĂŠ explicado pelo fato da função tangente, nesse intervalo, ser bijetora e ter como imagem todo o conjunto dos nĂşmeros reais. E sendo ainda a um real positivo, podemos fazer a = tg Îą, Îą ∈ (0, Ď€/2). Agora, podemos perguntar: por que essa transformação nos seria Ăştil? Isso ĂŠ respondido se percebermos que a partir da conhecida identidade trigonomĂŠtrica 1+ tg²ι = sec²ι, obtemos o seguinte resultado: 1 / 1 + tg 2Îą = cos Îą , âˆ€Îą ∈ ˆ ,

Ď€ + kĎ€ , com o qual podemos simplificar a desigualdade a ser provada. É 2 claro que se o estudante nĂŁo tem o devido costume com essas fĂłrmulas, ele, provavelmente, nĂŁo as reconheceria e nem pensaria em utilizar a transformação Îąâ‰

EUREKA! N°11, 2001

24


Sociedade Brasileira de MatemĂĄtica

para tangente. Mas ĂŠ aĂ­ que entra a relevância da trigonometria . Agora podemos prosseguir com a resolução. Façamos a = tg Îą, b = tg β e c = tg Îł, onde Îą, β, Îł ∈ (0, Ď€/2). Temos entĂŁo que tg Îą + tg β + tg Îł = tg Îą.tg β.tg Îł (1) . Como 1 / 1 + tg 2Îą = cos Îą , âˆ€Îą ∈ ‰ ,

Îąâ‰

Ď€ + kĎ€ , entĂŁo o que devemos mostrar agora ĂŠ que cos Îą + cos β + cos Îł 2

≤ 3/2 para quaisquer Îą, β, Îł ∈ (0, Ď€/2) que satisfaçam a condição (1). Mas de (1) vem que tg (Îą + β + Îł ) = 0 (verifique! Dica: use a fĂłrmula da tangente da soma de trĂŞs termos). Logo, como Îą , β , Îł ∈ (0, Ď€/2), temos que Îą + β + Îł = Ď€. Para finalizarmos a demonstração, usaremos a seguinte forma especial da desigualdade de Jensen (ver [4]): se uma função f ĂŠ estritamente cĂ´ncava (ver observação abaixo) num dado intervalo (a,b), entĂŁo  a + ... + a n f  1 n 

 f (a1 ) + ... + f (a n ) ,  ≼ n 

para quaisquer ai ∈ (a, b), ocorrendo a igualdade se e somente se os ai ' s forem todos iguais. E caso a função seja estritamente convexa (ver observação abaixo) em um determinado intervalo a desigualdade muda de sinal. Continuando, como a função cosseno ĂŠ estritamente cĂ´ncava no intervalo (0,Ď€/2), temos que: cos Îą + cos β + cos Îł 3 ι + β +Îł  Ď€  1 ≤ cos  = cos  = ⇔ cos Îą + cos β + cos Îł ≤ , 3 3 2 3 2   para quaisquer Îą, β, Îł ∈ (0,Ď€/2), ocorrendo a igualdade se e somente se Îą = β = Îł = Ď€/3 ⇔ a = b = c = tg Ď€/3 = 3 , concluindo a demonstração.

\ F #$ $ F$pŠ' ‡ • • •

A transformação de um número real em tangente de outro e o uso da fórmula 1+ tg²ι =sec²ι . Uso da propriedade dada no enunciado, para encontrar outra de melhor proveito. Uso de uma forma especial da desigualdade de Jensen.

EUREKA! N°11, 2001

25


Sociedade Brasileira de MatemĂĄtica

{ 0 | †z‹

Formalmente, uma função f : I → ÂŒ , onde I ⊂ ÂŒ

ĂŠ um intervalo, ĂŠ

 x + y  f ( x) + f ( y ) estritamente cĂ´ncava se f  , para quaisquer x, y distintos > 2  2   x + y  f ( x) + f ( y ) em I. E ĂŠ estritamente convexa se f  , para quaisquer x, y < 2  2  distintos em I. Uma maneira geomĂŠtrica de identificar funçþes estritamente cĂ´ncavas ou convexas ĂŠ observar a forma do grĂĄfico das mesmas: se o grĂĄfico for uma curva com concavidade voltada para baixo, a função ĂŠ estritamente cĂ´ncava, e se a concavidade for voltada para cima, ĂŠ estritamente convexa. Como exemplos de funçþes estritamente cĂ´ncavas, temos a função f(x) = cos x no domĂ­nio (0,Ď€/2) e as funçþes logarĂ­tmicas cujas bases sĂŁo maiores do que 1. E de funçþes estritamente convexas temos a função f(x) = tg x no domĂ­nio (0,Ď€/2) e a função f(x)=1/sen x, com x em (0,Ď€). (Como exercĂ­cio, classifique outras funçþes conhecidas em estritamente convexas ou cĂ´ncavas). É importante o leitor ver as referĂŞncias [2] e [4], onde encontram-se definiçþes e resultados mais precisos e genĂŠricos, alĂŠm das demonstraçþes.

O próximo exemplo, da IMO de 1996, Ê tido por alguns matemåticos interessados em olimpíadas (ver [1]), como o problema mais difícil jå proposto em IMO’s. É um problema de geometria associado a desigualdade (algo bastante explorado em olimpíadas).

*< ! >  Â— : (IMO 96) Seja ABCDEF um hexĂĄgono convexo tal que AB ĂŠ paralelo a DE, BC ĂŠ paralelo a EF e CD ĂŠ paralelo a FA. Sejam RA, RC, RE os raios das circunferĂŞncias circunscritas aos triângulos FAB, BCD, DEF respectivamente, e seja p o perĂ­metro do hexĂĄgono. Prove que p R A + RC + R E ≼ . 2 >F p('$ ‡ Algo nesse problema jĂĄ nos insinua a usar a trigonometria, vocĂŞ percebe? O fato dele relacionar raio de circunferĂŞncia circunscrita com lado (que tem a ver com o perĂ­metro) faz-nos lembrar da conhecida lei dos senos, que afirma: dado um BC AC AB = = = 2 R , onde R ĂŠ o raio da triângulo ABC, temos que senA senB senC circunferĂŞncia circunscrita ao triângulo dado (como exercĂ­cio, prove-a). DaĂ­, portanto, podemos agora nĂŁo mais trabalhar com os raios dos triângulos citados, mas sim com algumas diagonais do hexĂĄgono. Isso porque, pela lei dos senos, EUREKA! N°11, 2001

26


Sociedade Brasileira de Matemática ∧

obtemos que BF = 2RA.sen F A B , BD = 2RC.sen B C D , FD = 2RE.sen D E F (ou seja, encontramos uma relação entre os raios citados no problema e algumas diagonais do hexágono, o que facilitará o nosso trabalho). A próxima parte da resolução do problema é a que exige uma maior dose de criatividade por parte do estudante. Vejamos. Prolonguemos os lados paralelos BC e EF do hexágono (vide figura 1). Por A e D tracemos perpendiculares aos lados prolongados, obtendo o retângulo de vértices M, N, P e Q, ilustrados na figura 1. Como MN e PQ são as menores distâncias entre pontos das retas paralelas BC e EF (pois esses segmentos são perpendiculares às mesmas), temos que BF ≥ MN e BF ≥ PQ ⇒ 2BF ≥ MN + PQ ⇒ 2BF ≥ AM + NA + DP + DQ ⇒ 2BF ≥ AB.senB + AF.senF + CD.senC + DE.senE (1), onde sen X denota o seno do ângulo interno de vértice X do hexágono, o qual é igual ao seno do respectivo ângulo externo, pois os mesmos são suplementares. M

B

C

P

A

D

N

F

E

Q

figura 1 Pela lei dos senos, nós já sabemos que BF/senA = 2RA . Então dividindo ambos os lados da desigualdade (1) por senA, obtemos: 4 R A ≥ AB.

senB senF senC senE + AF . + CD. + DE. (I) senA senA senA senA

E de forma análoga, seguindo os mesmos passos com as diagonais BD e DF do hexágono, obtemos:

EUREKA! N°11, 2001

27


Sociedade Brasileira de Matemática

4 RC ≥ BC.

senB senD senA senE + CD. + AF . + EF . (ii) senC senC senC senC

4 R E ≥ AB.

senA senC senD senF + BC. + DE. + EF . (iii) senE senE senE senE

E agora, somando (I), (ii) e (iii), obtemos  senA senB   senC senB   senC senD  + + + 4( R A + RC + R E ) ≥ AB.  + BC.  + CD. +  senE senA   senE senC   senA senC   senE senD   senE senF   senA senF  + DE. + + +  + EF .  + FA. .  senA senE   senC senE   senC senA  Agora observe que como os lados opostos do hexágono convexo são paralelos, nós temos que os ângulos opostos do mesmo são congruentes. Assim, nós obtemos: senA = senD; senB = senE; senC = senF. Por conseguinte, nós temos que os fatores que estão multiplicando os lados do hexágono na última desigualdade acima são da forma (z + 1/z), sendo z positivo (pois o seno de um ângulo maior que 0º e menor que 180º é sempre positivo). E é fácil verificar que z + 1/z ≥ 2, para todo z positivo. Assim nós obtemos: 4( RA + RC + RE ) ≥ 2( AB + BC + CD + DE + EF + FA) ⇒ 4( RA + RC + RE ) ≥ 2 p ⇒ p , concluindo a demonstração. $ & W W ~ ~ W 2 ⇒ RA + RC + RE ≥

• • • •

Uso da Lei dos Senos. Uso das construções: prolongamento de dois lados opostos e traçado de perpendiculares pelos dois vértices restantes. Congruência dos ângulos opostos do hexágono convexo. Uso da desigualdade: z + 1/z ≥ 2, z > 0.

d $ ~ ~

(IMO 91) Seja ABC um triângulo e X um ponto interior do mesmo. Prove que pelo menos um dos ângulos ∠XAB, ∠XBC, ∠XCA é menor ou igual a 30º.

EUREKA! N°11, 2001

28


Sociedade Brasileira de MatemĂĄtica

“Š’~” •$– ŽŠ—(˜W• Â&#x;

Geralmente, em questĂľes que envolvem um ponto num interior de um triângulo, ĂŠ Ăştil traçarmos perpendiculares a partir desse ponto aos lados do triângulo. Vamos utilizar isso. Sejam P, Q, R os pĂŠs das perpendiculares traçadas por X aos lados BC, CA e AB, respectivamente. Para facilitar, denotaremos por Îą , β , Îł os ângulos do triângulo (∠BAC, ∠CBA, ∠ACB) e por ι’ , β ’ , γ’ os ângulos ∠XAB, ∠XBC, ∠XCA.

A R

Q

X B

P

C

figura 2 NĂłs temos que PX = BX.senβ ’ = CX.sen(Îł – γ’); QX = CX.senγ’ = AX.sen(Îą – ι’); RX = AX.senι’ = BX.sen(β – β ’). Multiplicando essas trĂŞs igualdades, nĂłs obtemos: sen(Îą − Îą ' ).sen( β − β ' ).sen(Îł − Îł ' ) = senÎą '.senβ '.senÎł ' ⇔ sen(Îą − Îą ' ) sen( β − β ' ) sen(Îł − Îł ' ) = 1. senÎą ' senβ senÎł '

sen( A − x) = senA. cot x − cos A ĂŠ senx estritamente decrescente no intervalo (0, Ď€), visto que a função cotangente ĂŠ estritamente decrescente nesse intervalo. Assim, se ι’ , β ’ , γ’ forem todos maiores que 30Âş, teremos que: Agora

observe

que

a

função

f ( x) =

sen(Îą − Îą ' ) sen( β − β ' ) sen(Îł − Îł ' ) sen(Îą − 30Âş ) sen( β − 30Âş ) sen(Îł − 30Âş ) < ⇔ senÎą ' senβ senÎł ' sen30Âş sen30Âş sen30Âş 1 (1) sen(Îą − 30Âş ) sen( β − 30Âş ) sen(Îł − 30Âş ) > sen30Âş sen30Âş sen30Âş = 8

1=

EUREKA! N°11, 2001

29


Sociedade Brasileira de Matemática

Mas, nós temos que: sen(α − 30º ).sen(β − 30º ) = =

1 (cos(α − β ) − cos(α + β − 60º )) ≤ 1 (1 − cos(α + β − 60º )) = 2 2

1 (1 − sen(γ − 30º )) 2

Observe que essa última igualdade decorre do fato de (γ – 30º) ser complementar a (α + β – 60º). Continuando, nós temos que: sen(α − 30º ) sen( β − 30º ) sen(γ − 30º ) ≤ 11  1 =  −  sen(γ − 30º ) −   24  2

2

1 (1 − sen(γ − 30º )).sen(γ − 30º ) = 2

 1 ≤ .  8 

Mas esta última desigualdade obtida contradiz (1), logo α’ , β ’ , γ’ não podem ser todos maiores do que 30º, o que encerra a nossa demonstração. $ & W W ~ ~ W

• •

Construção das perpendiculares a partir de X aos lados do triângulo e obtenção da igualdade: sen(α − α ' ).sen( β − β ' ).sen(γ − γ ' ) = senα '.senβ '.senγ ' . Esses ângulos são identificados no início da resolução. sen( A − x) Observar que a função f ( x) = é estritamente decrescente. senx Supor, por absurdo, que todos os três ângulos são maiores do que 30º, e chegar a uma contradição.

d $ ~ ~ ¢¡d

Prove que, dentre quaisquer cinco reais y1 , y2 , y3 , y4 , y5 , existem dois, que satisfazem: yi − y j ≤ 1. 0≤ 1 + yi y j

EUREKA! N°11, 2001

30


Sociedade Brasileira de MatemĂĄtica

“Š’~” •$– ŽŠ—(˜W• Â&#x;

Olhando para o termo do meio da desigualdade acima, o que ele nos faz lembrar? Sem muita dificuldade, associamo-lo logo Ă formula da tangente da diferença. EntĂŁo mais uma vez façamos uso da transformação para tangente. Isto ĂŠ, façamos yi = tg xi, i = 1, 2, 3, 4, 5 , xi ∈ (–π/2, Ď€/2). Como tg 0 = 0 e tg Ď€/4 = 1, devemos ter agora: tg 0 ≤

tgx i − tgx j 1 + tgx i .tgx j

≤ tg

Ď€ Ď€ ⇔ tg 0 ≤ tg (x i − x j ) ≤ tg . 4 4

E ainda, como no intervalo (–π/2, Ď€/2) a função tangente ĂŠ sempre crescente, Ď€ obtemos 0 ≤ xi − x j ≤ . Agora o que temos que provar ĂŠ que existem dois 4 dentre os cinco xi’s que satisfazem esta Ăşltima desigualdade. Para isso, usamos o conhecido PrincĂ­pio da Casa dos Pombos. Dividamos o intervalo (–π/2, Ď€/2) de tamanho Ď€ em outros quatro intervalos de tamanho Ď€/4. Assim, pelo princĂ­pio citado, dois dentro os cinco xi’s estarĂŁo no mesmo intervalo, os quais vĂŁo satisfazer a desigualdade pedida. Â?$ŽŠÂ? Â?&‘WÂ’ “WÂ’~”~• – ÂŽW—Š˜Š• Â&#x;

• • •

Uso da transformação para tangente. Aplicação da tangente da diferença. Uso do Princípio da Casa dos Pombos.

Finalizamos esse texto com alguns problemas para o leitor exercitar o que foi mostrado. É lĂłgico que existe mais de uma solução para cada problema, mas pede-se que o leitor tente resolvĂŞ-los utilizando a trigonometria e as desigualdades mostradas ou outras conhecidas. Â’d™$Â’~“Š£Š¤¼£(Â? • ”~Â&#x; Œ §

. (IMO 61) Prove que, para qualquer triângulo de lados a, b, c e ĂĄrea A, temos que: a 2 + b 2 + c 2 ≼ 4 3 A .

EUREKA! N°11, 2001

31


Sociedade Brasileira de Matemática

¦ ¨d©

Prove que, dentre 13 números reais, existem dois, x e y, tais que:

(

)

x − y ≤ 2 − 3 .1 + xy . ¦ ©

(OBM – 85) Um quadrilátero convexo está inscrito em uma circunferência de raio unitário. Demonstre que a diferença entre seu perímetro e a soma de suas diagonais é maior do que zero e menor do que 2. ¦ ¡ ©

(Ibero-Americana 88) As medidas dos ângulos de um triângulo estão em progressão aritmética e as medidas das alturas do mesmo também. Prove que o triângulo é equilátero. ¦ ªd©

(Putnam 78) Encontre a área de um octógono convexo que está inscrito em uma circunferência e que tem que quatro lados consecutivos medindo 3 unidades e os lados restantes medindo 2 unidades. Dê a resposta na forma r + s t , com r, s e t inteiros positivos. ¦ «d©

(Grã-Bretanha 84) O quadrilátero ABCD tem uma circunferência inscrita. ∧

Para o lado AB nós associamos a expressão f(AB) = p1.(sen D A B ) + p2. ∧

(sen A B C ), onde p1 e p2 são as medidas das perpendiculares traçadas de A e B, respectivamente, até o lado oposto CD. Definimos f(BC), f(CD) e f(DA) similarmente, usando para cada um as perpendiculares ao lado oposto. Mostre que f(AB) = f(BC) = f(CD) = f(DA). ¦ ¬d©

(IMO 91) Em um triângulo ABC, as bissetrizes AD, BE, CF encontram-se no ponto I. Mostre que: 1 8 IA IB IC . . . ≤ ≤ 4 AD BE CF 27 ¦ ­d©

Mostre que se um quadrilátero de lados a, b, c, e d é inscritível e circunscritível então sua área é abcd .

EUREKA! N°11, 2001

32


Sociedade Brasileira de Matemática

¦ ®d©

Uma função d(x, y) de dois reais x, y é chamada distância se d(x, y) = d(y, x); d(x, x) = 0 ; e d(x, y) + d(y, z) ≥ d(x, z), para quaisquer reais x, y, z. Prove que a seguinte função é uma distância: d ( x, y ) = ¯ §d¦ ©

x− y 1+ x2 1+ y2

.

Sejam x, y, z reais positivos tais que xy + yz + zx = 1. Prove que:

(

)

(

)

(

)

2x 1 − x 2 2y 1− y2 2z 1 − z 2 x y z + + ≤ + + 2 2 2 2 2 2 2 2 (1 + x ) (1 + y ) (1 + z ) 1+ x 1+ y 1+ z2

Rafael Tajra Fonteles cursa a 2ª. Série do Ensino Médio no Instituto Dom Barreto de Teresina – PI. O Prof. José Nazareno Cardeal Fonteles da Universidade Federal do Piauí e coordenador de matemática do Instituto Dom Barreto colaborou com o artigo, fazendo a revisão do mesmo. ° ° ( ± [1] – ENGEL, Arthur. Problem-Solving Strategies. Springer-Verlag, New York, 1998. [2] – LARSON, Loren C. Problem-Solving Through Problems. Springer-Verlag, New York, 1983. [3] – MEGA, Élio & WATANABE, Renate. Olimpíadas Brasileiras de Matemática – 1ª a 8ª. Comissão de Olimpíadas da SBM. Núcleo, São Paulo, 1988. [4] – MUNIZ NETO, Antônio Caminha. Desigualdade Elementares. Eureka! Nº 5. OBM, 1999. [5] – Página da Web mantida por Jonh Scholes: www.kalva.demon.co.uk/

EUREKA! N°11, 2001

33


Sociedade Brasileira de Matemática

²O³O´&µz¶O²¸·º¹ ´M»½¼¾µy² ¿AÀ(Á= à À ÄMÅÇÆ ÈPÉ Â ÈMÊ5Ë5Ä Ì Í É Î Ä?¿WÏ Â Ð Â

♦ ÑZÒ Ó]Ô Õ ÖWÓd×PØ Ù × ÚPÛ(Ü Um polinômio é uma expressão da forma p( x) = a 0 + a1 x + a 2 x 2 + ... + a n x n . Uma série formal é uma expressão ainda mais simples - basta apagar o último termo: p( x) = a 0 + a1 x + a 2 x 2 + ... Somas e produtos são definidos de maneira análoga às operações correspondentes com polinômios. Assim, por exemplo, (1 − 3x + 3 2 x 2 − 3 3 x 3 + ...)(1 + 3x) = 1 − 3x + 3 2 x 2 − 33 x 3 + ... + 3 x − 3 2 x 2 + 3 3 x 3 − ...

=1 de modo que podemos escrever 1 − 3 x + 3 2 x 2 − 33 x 3 + ... = 1 /(1 + 3 x). De maneira geral, podemos "compactar" uma série formal 1 + ax + a2x2 +… na forma 1/(1 – ax), que certamente ocupa bem menos espaço que uma série infinita… Vejamos uma primeira aplicação das séries formais. Vamos determinar uma "formula fechada" para a seqüência definida por a 0 = 0 , a1 = 1 e a n + 2 = 5a n +1 − 6a n para n ≥ 0. A idéia é considerar a série formal f ( x) = a 0 + a1 x + a 2 x 2 + ... e tentar "compactá-la" e depois "descompactá-la". Para alcançar o primeiro objetivo, observe que f ( x) = a 0 + a1 x + a 2 x 2 + a 3 x 3 + ... − 5 xf ( x) = −5a 0 x − 5a1 x 2 − 5a 2 x 3 + ... 6 x 2 f ( x) =

6a 0 x 2 + 6a1 x 3 + ...

Somando as equações acima, os coeficientes de x n , n ≥ 2, anulam-se e ficamos com x (1 − 5 x + 6 x 2 ) f ( x) = a 0 + ( a1 − 5a 0 ) x ⇔ f ( x) = 1 − 5x + 6 x 2 Agora, como descompactar f(x)? O truque aqui é "quebrá-lo" em pedaços que sabemos como descompactar. Observe que 1 − 5 x + 6 x 2 = (1 − 2 x)(1 − 3x) e que é razoável procurar constantes a e b tais que EUREKA! N°11, 2001

34


Sociedade Brasileira de Matemática

(a + b) − (3a + 2b) x a b x x + = ⇔ = 2 1 − 2 x 1 − 3x 1 − 5 x + 6 x (1 − 2 x)(1 − 3 x) 1 − 5x + 6 x 2

a + b = 0 ⇔ ⇔ a = −1 e b = 1 3a + 2b = −1 Logo f (x) =

1 1 x = − = (1 + 3x + 32 x2 + 33 x3 + ...)− (1 + 2x + 22 x2 + 23 x3 + ...) 2 1 − 5x + 6x 1 − 3x 1 − 2x = (3 0 − 2 0 ) + (31 − 21 ) x + (3 2 − 2 2 ) x 2 + (3 3 − 2 3 ) x 3 + ...

[ ]

Assim, o coeficiente de x n em f(x) (denotado por x n f (x)) é 3 n − 2 n . Mas

[x ]f ( x) = a n

, pela definição de f(x), logo a n = 3 − 2 . n

n

n

~ ° A ~ §

Utilizando séries formais, encontre "formulas fechadas" para as seguintes seqüências: F0 = 0, F1 = 1, Fn + 2 = Fn +1 + Fn para n ≥ 0 (esta é a famosa seqüência de Fibonacci) b) t 0 = t1 = 1, t n + 2 = −2t n +1 − 4t n para n ≥ 0 c) p 0 = p1 = 1, p 2 = 0, p n + 3 = 7 p n +1 − 6 p n para todo n ≥ 0

a)

(As outras seqüências não tem nome. Alguma sugestão?) ~ ° A ~ ¨

Calcule F0 10

0

+

F1 10

1

+

F2 10

2

+

F3 10 3

+ ...

em que Fn denota a seqüência de Fibonacci.

Outra aplicação das séries formais é ajudar a contar. Neste contexto, as séries formais recebem o nome de funções geratrizes. O esquema geral é o seguinte: o exponente de x quantifica alguma propriedade em que estamos interessados, EUREKA! N°11, 2001

35


Sociedade Brasileira de Matemática

como o comprimento de uma seqüência, o número de conjuntos em uma partição, a quantidade de duendes verdes em um jardim, etc. Se para cada objeto associarmos tal potência de x e somarmos estas potências, o coeficiente de x n será, respectivamente, o número de objetos com comprimento n, o número de partições com n conjuntos, o número de jardins com n duendes verdes, etc. Por exemplo, considere o problema de determinar o número de maneiras de se escrever n como soma de termos 1, 2, 3, sem levar em conta a ordem dos termos. A idéia aqui não é tentar obter este número para um valor particular de n. Somos mais ousados: vamos obter todos estes números de uma só vez. Para isto, escrevemos a função geratriz f(x) que é a soma das potências xs para cada soma s: f ( x) = x 0 + x 1 + x 1+1 + x 2 + x 1+1+1 + x 1+ 2 + x 3 + ... = 1 + x + 2 x 2 + 3x 3 + ... ( x 0 corresponde à soma sem nenhum termo), de modo que, por exemplo, o coeficiente de x 3 é o número de maneiras de escrever 3 como soma não ordenada de termos 1, 2, 3. Aparentemente, obter f (x) é uma tarefa mais difícil do que a inicial. Mas observe que cada termo de f (x) é o produto de um termo da forma xsomas de 1's , um termo da forma xsomas de 2's e um termo da forma xsomas de 3's, logo f ( x) = ( x0 + x1 + x1+1 + x1+1+1 + ...)(x0 + x2 + x2+2 + x2+2+2 + ...)(x0 + x3 + x3+3 + x3+3+3 + ...) ⇔ f ( x ) = (1 + x + x 2 + x 3 + ...)( 1 + x 2 + x 4 + x 6 + ...)( 1 + x 3 + x 6 + x 9 + ...) 1 1 1 ⇔ f ( x) = 2 1− x 1 − x 1 − x3

[ ]

O problema agora é encontrar x n f ( x ), o que pode ser feito utilizando-se as técnicas já vistas. ~ ° A ~ &

Mostre que o número de partições não ordenadas de n com exatamente k termos distintos é 1 + x j ( y − 1) xn yk 1− x j j ≥1

[

]∏

EUREKA! N°11, 2001

36


Sociedade Brasileira de Matemática

~ ° A ~ &¡

a) Encontre constantes a, b, c, d, e, f tais que 1 1 1 a b c d e f = + + + + + f ( x) = 2 3 3 2 1− x 1− x 1− x 1 − x 1 + x 1 − ωx 1 − ω 2 x (1 − x ) (1 − x ) em que ω = ( − 1 + i 3 ) / 2 .

[ ]

(n + 3) 2 7 (−1) n ω n + ω 2 n  (n + 3) 2 1  − + + = +  12 72 8 9 2  12 em que x  denota o maior inteiro menor ou igual a x.

b) Mostre que x n f ( x) =

~ ° A ~ ª

a) Determine a função geratriz do número de soluções da equação x1 + x 2 + ... + x k = n , em que x i são inteiros positivos, 1 ≤ x i ≤ k . b) Determine a função geratriz do número de partições ordenadas de n. (por exemplo, 4 = 1 + 3 = 3 + 1 = 2 + 2 = 2 + 1 + 1 = 1 + 2 + 1 = 1 + 1 + 2 = 1 + 1 + 1 + 1, de modo que há 8 partições ordenadas de 4) c) Determine o número de partições ordenadas de n. Os próximos problemas mostram uma técnica muito importante chamada convolução. Ela se baseia no seguinte fato: se f ( x ) = f 0 + f 1 x + f 2 x 2 + f 3 x 3 + ... g ( x ) = g 0 + g 1 x + g 2 x 2 + g 3 x 3 + ... então h ( x ) = f ( x ) g ( x ) é a série formal associada à seqüência h 0 = f 0 g 0 , h1 = f 0 g 1 + f 1 g 0 , h 2 = f 0 g 2 + f 1 g 1 + f 2 g 0 , ..., h k =

∑fg i

j

,...

i+ j=k

~ ° A ~ «

a) Mostre que se f(x) é a função geratriz da seqüência ( a n ) n ≥ 0 , então f ( x ) /(1 − x ) é a função geratriz da seqüência b n = a 0 + a1 + ... + a n . b) Prove que F0 + F1 + ... + Fn = F n + 2 − 1, em que Fn denota a seqüência de Fibonacci. (Este resultado pode também ser provado facilmente sem o uso de séries formais. Tente!) EUREKA! N°11, 2001

37


Sociedade Brasileira de Matemática

~ ° A ~ ¬

Prove que n  n   n  n   F0 +   F1 +   F 2 + ... +   Fn = F2 n n  n − 1  n − 2 0 em que, adivinhe, Fn denota a seqüência de Fibonacci! ~ ° A ~ ­

a) Mostre que o número de triangulações Tn (por diagonais que não se interceptam fora dos vértices) de um polígono convexo de n vértices satisfaz T n +1 = T 2 T n + T3 T n −1 + ... + T n T 2 em que T2 = 1. b) Prove que T ( x ) = T 2 + T3 x + T 4 x 2 + ... =

1 − (1 − 4 x ) 1 / 2 2x

1  2n − 2   . n − 1  n − 1 

c) Mostre que T n =

(T n +1 é o assim chamado n-ésimo número de Catalan). Para isto, lembre-se da fórmula do binômio de Newton generalizada:

(1 + x ) r =

r 

∑  k x k ≥0

k

,

Þ  r  r ( r − 1)( r − 2 )...( r − k + 1)   = ,k ∈ k! k 

e r∈

Ý

Muitas vezes, as funções geratrizes são utilizadas não para calcular o número exato de maneiras de se fazer isto ou aquilo, mas para mostrar que duas quantidade são iguais. Vamos mostrar que o número de partições (não ordenadas) de n em naturais distintos é igual ao número de partições (também não ordenadas) de n em naturais ímpares. Por exemplo, 7 = 5 + 1 + 1 = 3 + 3 + 1 = 3 +1 + 1 + 1 + 1 = 1 + 1 + 1 + 1 + 1 + 1 + 1 e 7 = 6 + 1 = 5 + 2 = 4 + 3 = 4 + 2 + 1, de modo que em ambos os casos o número de partições em naturais distintos é: (1 + x )(1 + x 2 )(1 + x 3 )(1 + x 4 )... enquanto que o número de partições em naturais ímpares é dado por

EUREKA! N°11, 2001

38


Sociedade Brasileira de Matemática

(1 + x + x1+1 + x1+1+1 + ...)(1 + x 3 + x 3+3 + x 3+3+3 + ...)...(1 + x 5 + x 5+5 + x 5+ 5+5 + ...)... = (1 + x + x 2 + x 3 + ...)(1 + x 3 + x 6 + x 9 + ...)...(1 + x 5 + x 10 + x 15 + ...)... 1 1 1 = ... 3 1 − x 1 − x 1 − x5 Observe que as expressões acima são iguais! De fato, para se convencer disto, basta multiplicar as igualdades 1− x2 1− x4 1 − x6 2 = 1 + x, = + = 1 + x 3 ,... 1 , x 2 3 1− x 1− x 1− x

Isto completa a demonstração. ~ ° A ~ ®

a) Escreva a função geratriz do número de maneiras de escrever um número n como soma de potências distintas de 2. b) Verifique que a função acima é igual a 1/(1 – x). c) Utilizando os resultados acima, mostre que todo número pode ser escrito de maneira única em base 2. ~ ° A ~ § ¦

Prove que o número de partições de n em que apenas as partes ímpares podem ser repetidas é igual ao número de partições de n em que nenhuma parte aparece mais do que três vezes. ~ ° A ~ § §

Prove que o número de partições de n com uma única parte menor (ela ocorre uma única vez) e parte maior no máximo duas vezes a parte menor é igual ao número de partições de n em que a maior parte é ímpar e a menor parte é maior do que metade da parte maior. ~ ° A ~ § ¨

Mostre que o número total de 1's nas partições de n é igual à soma dos números de partes distintas em cada partição de n.

EUREKA! N°11, 2001

39


Sociedade Brasileira de MatemĂĄtica

š Ă&#x;à ¾åÂžâO㼟žä Âź ²üŸžšÌ´MÂśOäMš ´çäMšÌÂžèMĂŠ&äMš ĂŤĂŞ

O comitĂŞ editorial de ĂŹAĂ­ZĂŽ ĂŹAĂŻ Ă– ! agradece aos admiradores da seção OlimpĂ­adas ao redor do Mundo o envio de crĂ­ticas, soluçþes e possĂ­veis erros nos enunciados dos problemas. Diversos leitores apontaram um possĂ­vel erro no enunciado de: Â? Â? Š Âœ š Ăą ò Ăł ôÜþ`á ødĂš §dÂŽÂ ÂŽdÂŽdĂş Ă° Seja n um nĂşmero natural tal que 2n 2 possui 28 divisores

distintos e o número 3n 2 possui 30 divisores distintos. Qual o número de divisores do número 6n 2 ? Na realidade o problema estå errado mesmo, apesar de corresponder à versão publicada em : http://www.olsedim.com/olympiad/99.html De fato, se 2n2 tem 28 divisores, 3n2 só pode ter 24, 42 ou 54 divisores (prove!). O problema Ê parecido com a 29ª. questão da ÝPß ý Þ7ÿ onde os números que possuíam 28 e 30 divisores eram respectivamente 2n e 3n, e pedese o número de divisores de 6n (Esse tem solução!). Continuamos salientando que estamos à disposição na OBM para aqueles que estiverem interessados na solução de algum problema particular. Para tanto, basta contactar a OBM, atravÊs de carta ou e-mail.

Ăż 5Ă˝ 5Ă˝

Primeiramente vamos aos problemas propostos deste número Ž §dŠ

Ž ¨dŠ

Ž � Š

!#" $%$

á ø Ú ¨ ŒdŒdŒ ú

!#" $%$

á ø Ú ¨ ŒdŒdŒ ú

!#" $%$

á ø Ú ¨ ŒdŒdŒ ú

Os coeficientes a e b da equação x 2 + ax + b = 0 e suas raĂ­zes sĂŁo quatro nĂşmeros distintos. É possĂ­vel determinar a equação usando estes quatro nĂşmeros? Um inteiro positivo n ĂŠ chamado perfeito se a soma de todos os seus divisores, excluindo n, ĂŠ igual a n. Prove que se um nĂşmero perfeito maior do que 28 ĂŠ divisĂ­vel por 7 entĂŁo ele ĂŠ divisĂ­vel por 49. Prove a existĂŞncia de nĂşmeros reais distintos a1 , a 2 , ... , a10 tais que a equação: (x − a1 )â‹… (x − a 2 )â‹… ... â‹… (x − a10 ) = (x + a1 )â‹… (x + a 2 )â‹… ... â‹… (x + a10 ) possua exatamente 5 raĂ­zes reais distintas.

EUREKA! N°11, 2001

40


Sociedade Brasileira de Matemática

® ¡ ©

® ªd©

¨ ¦d¦d¦%*

!#" $%$'& (%)

O círculo inscrito no triângulo ABC possui centro O e tangencia o lado AC no ponto K. Um segundo círculo S com centro no mesmo ponto O intersecta todos os lados do triângulo ABC. Sejam E e F os pontos de interseção de S com os lados AB e BC que estão mais próximos do vértice B; B1 e B2 são os pontos de interseção de S com o lado AC com B1 mais próximo de A. Se P é o ponto de interseção dos segmentos B 2 E e B1 F , mostre que os pontos B, K e P são colineares. ¨ ¦d¦d¦%*

+!,#- .%. & (%)

(a1 , a 2 ,..., a 2000 )

A seqüência de números reais

satisfaz a

condição: + + ⋅ ⋅ ⋅ + = (a1 + a 2 + ⋅ ⋅ ⋅ + a n ) para todo n , 1 ≤ n ≤ 2000 . Mostre que todo elemento da seqüência é um número inteiro. a13

® «d©

® ¬d© ® ­d©

® ®d©

§ ¦d¦d©

a 23

2

a n3

¨ ¦d¦d¦%*

+!,#- .%. & (%)

Cada um dos números 1, 2, 3, ..., N é preto ou branco. É permitido mudar simultaneamente as cores de quaisquer três dos números se um deles é a média aritmética dos outros dois. Determine os valores de N para os quais é possível fazer com que todos os números fiquem brancos.

+!/0.2143 5 & (')

¨d¦d¦ ¦'*

Determine todos os restos possíveis da divisão do quadrado de um número primo com 120 por 120.

+!/0.2143 5 & (')

¨d¦d¦ ¦'*

Em um inteiro positivo M, de três algarismos, o algarismo das centenas é menor do que o algarismo das dezenas e o algarismo das dezenas é menor que o algarismo da unidades simples. A média aritmética de M com todos os números de três algarismos obtidos pela reordenação dos algarismos de M termina em 5. Determine tais números M.

+!/0.2143 5 & (')

¨d¦d¦ ¦'*

Gustavo convidou um número ímpar de pessoas para a festa de seu aniversário e os dispôs em torno de uma mesa circular de modo que os vizinhos de cada menina fossem meninos, os vizinhos de cada menino, exceto Gustavo cujos vizinhos eram ambas meninas, fossem uma menina e um menino. Mostre que : a) o número de meninos convidados para a festa é divisível por 4. b) na direção diametralmente oposta a Gustavo está sentada uma menina .

+!/0.'6 7980:%5 & (%)

¨ ¦d¦d¦%*

Seja H o ortocentro de um triângulo acutângulo ABC com AC ≠ BC . A reta que passa pelos pontos médios de AB e HC intersecta a bissetriz do ângulo ∠ACB no ponto D. A reta HD passa pelo circuncentro do triângulo ABC. Determine a medida do ângulo ∠ACB .

EUREKA! N°11, 2001

41


Sociedade Brasileira de Matemática

§ ¦d§d©

+!/0.'6 7980:%5 & (%)

¨ ¦d¦d¦%*

Sobre uma mesa existem pilhas de moedas. Um movimento consiste em escolher uma das pilhas com pelo menos três moedas, pegar uma moeda desta pilha, retirá-la da mesa e finalmente dividir as moedas restantes desta pilha em duas outras pilhas (não necessariamente do mesmo tamanho). É possível obtermos somente pilhas com três moedas após vários movimentos se começarmos com uma única pilha com 2000 moedas? ¨ ¦d¦d¦%*

§ ¦d¨d©

Determine todas as funções f : ; → ; condição f (x − f ( y )) = 1 − x − y para todos x, y ∈ ; .

§ ¦ ©

+!/0.'6 7980:%5 & (%)

+!/0.'6 7980:%5 & (%)

que satisfazem a

¨ ¦d¦d¦%*

Sobre uma mesa estão três caixas com pelo menos uma bola em cada uma delas. Um movimento consiste em duplicar o número de bolas de uma das caixas de modo que o número de bolas necessário para tal seja retirado de uma das outras duas caixas. É possível que após vários movimentos uma das caixas esteja vazia?

§ ¦ ¡ © °

¨ ¦d¦ ¦'*

§ ¦dªd© °

¨ ¦d¦ ¦'*

+ & < 6 7 =>- .'. & (')

Determine todos os pares de números inteiros (x, y) que satisfazem a equação: y x 2 + 36 + x y 2 − 36 + y 2 ( y − 12 ) = 0

+ & < 6 7 =>- .'. & (')

(

) (

)

Seja M o ponto de interseção das diagonais AC e BD de um quadrilátero convexo ABCD. Seja K o ponto de interseção do prolongamento do lado AB, no sentido de B para A, com a bissetriz do ângulo ∠ACD . Sabendo que MA ⋅ MC + MA ⋅ CD = MB ⋅ MD , prove que ∠BKC = ∠CDB .

§ ¦d«d©

¨ ¦d¦ ¦'*

§ ¦d¬d©

¨ ¦d¦ ¦'*

§ ¦d­d©

¨ ¦d¦ ¦'*

+ 7 6 ? @28 & (')

Para cada subconjunto não vazio X do conjunto M = {1,2,...,2000}, seja a X a soma do menor com o maior elemento de X. Determine a média aritmética de todos tais números a X assim obtidos.

+ 7 6 ? @28 & (')

Determine o valor máximo do produto xy se os números reais x e y satisfazem a relação: y (1 + x 2 ) = x 1 − 4 y 2 − 1 .  

+ 7 6 ? @28 & (')

Dois círculos se intersectam nos pontos M e N. A reta que passa por M intersecta os círculos nos pontos A e B de modo que M ∈ (AB ) . Os pontos C e D são os pontos médios dos arcos AN e BN respectivamente e que não contém o ponto M e os pontos K e L são os pontos médios dos segmentos AB e CD respectivamente. Prove que CL = KL.

EUREKA! N°11, 2001

42


Sociedade Brasileira de MatemĂĄtica

§ ŒdŽdŠ

§ §dŒdŠ

+!A0B C D (')

¨dŒdŒ Œ'*

Seja q(n) a soma dos algarismos de n. Qual o valor de ? q q q 2000

(((

+!E F>G'H & (')

2000

)))

¨dŒdŒ Œ'*

Determine o nĂşmero primo p para o qual o nĂşmero 1 + p + p + p 3 + p 4 ĂŠ um quadrado perfeito. 2

¨ ŒdŒ Œ'*

§ §d§dŠ

Sejam P (x ) e Q(x ) dois trinĂ´mios quadrĂĄticos tais que trĂŞs das raĂ­zes da equação P (Q (x )) = 0 sĂŁo os nĂşmeros − 22 , 7 e 13 . Determine a quarta raiz desta equação.

§ §d¨dŠ

+!,#I'J - K 6 & H ( .RLNH'O I'H ( IS/0.%6 7Q8 ( H (')

§ § ¥ Š

+!T .'F ( I%6 )

+!,#I'J - K 6 & H ( .MLNH'O I'H ( IP/0.%6 7Q8 ( H (')

¨ ŒdŒ Œ'*

Sabendo que a , b e c sĂŁo nĂşmeros reais positivos, resolva o sistema no conjunto dos nĂşmeros reais positivos :  xy + xz − x = a   yz + yx − y = b  zx + zy − z = c  § § Â? Š › ¨dÂŚdÂŚÂ ÂŚ'* ( 7 6 3 5 & (') Uma seqßência ( p1 , p 2 ,..., p n ) de nĂşmeros primos satisfaz Ă seguinte condição: para n ≼ 3 , p n ĂŠ o maior divisor primo de p n −1 + p n − 2 + 2000. Mostre que a seqßência ( p n ) ĂŠ limitada. ¨ ŒdÂŚÂ ÂŚ'*

ABC Ê um triângulo do Plano Cartesiano cujos vÊrtices são pontos de coordenadas inteiras. As medidas de dois dos lados AB, BC e CA pertencem ao conjunto 17 , 1999 , 2000 . Qual o valor måximo possível da årea de ABC ?

{

§ §dªdŠ

§ §ddŠ

+!T 5 U 6 ( 14I'F>F (')

¨dŒ ŒdŒ'*

Sejam P e Q os pontos de tangência da tangente comum a dois círculos C1 e C 2 que se intersectam nos pontos M e N sendo N mais próximo de PQ do que M. Mostre que os triângulos MNP e MNQ possuem åreas iguais.

+!T 5 U 6 ( 14I'F>F (')

¨dŒ ŒdŒ'*

Quais os inteiros positivos a e b tais que

( a+ 3

§ §ddŠ

}

+!V#6 I'W ( 5 O (')

3

)

2

b − 1 = 49 + 203 6 .

¨dŒdŒ Œ'*

Determine os nĂşmeros reais x tais que:  x 2 − x − 1 − 3 − 5 − 7 − 9 − 11 − 13 = x 2 − 2 x − 48

EUREKA! N°11, 2001

43


Sociedade Brasileira de Matemática

§ §d­d©

§ §d®d©

+!V#6 I'W ( 5 O (')

¨d¦d¦ ¦'*

Um quadrilátero convexo ABCD está inscrito em um semicírculo de diâmetro AB. Sejam S o ponto de interseção de AC e BD e T o pé da perpendicular baixada de S a AB. Mostre que ST divide o ângulo ∠CTD ao meio.

+!T 5 U 6 ( 14I'F>F (')

§d® ¬d¦'*

As medidas dos ângulos ∠B e ∠C de um triângulo isósceles ABC são iguais a 50º . Sejam D e E pontos sobre os lados BC e AC respectivamente de modo que ∠BAD = 50º e ∠ABE = 30º . Determine a medida do ângulo ∠BED .

§ ¨d¦d©

+ 7 6 3 5 & (')

¨d¦d¦ ¦

o conjunto de todos os inteiros positivos. Prove ou ) Seja X disprove a seguinte afirmação : “Existe uma função f : X → X tal que a igualdade f ( f (n )) = 2n é verdadeira para todo n ∈ Y ”.

Z[Z\Z Agora vamos aos comentários e soluções dos leitores para alguns dos problemas apresentados em nossa seção nos números anteriores da ] ^ _ ]a`cbed f § ©

°

§ ®d® ­'*

Seja f (x ) = x 3 − 3 x + 1 . Determine o número de soluções reais e distintas da equação f ( f (x )) = 0 .

+ B 6 U g%F & (')

h i j k l m i n oqp o rts j n ivu o rtj w x i y{z x w icr | h m ivu s k j i } h u ~2 A resposta é 7. Utilizando cálculo, a derivada de f (x ) é 3x 2 − 3 , e então seus pontos críticos ocorrem em (− 1,3) e (1,−1) , respectivamente máximo e mínimo. Mas f (− 2 ) = −1 , f (0) = 1 e f (2 ) = 3 assim, as raízes de f pertencem aos intervalos (− 2,−1) , (0,1) e (1,2) . Se contarmos o número de vezes que f atravessa completamente cada intervalo obteremos a nossa resposta. Com efeito, f atravessa (− 2,−1) uma vez (quando x < −2 ), atravessa o intervalo (0,1) três vezes (quando − 2 < x < −1 , 0 < x < 1 e 1 < x < 2 ); atravessa o intervalo (1,2) três vezes também e portanto a equação f ( f (x )) = 0 possui 7 raízes.

EUREKA! N°11, 2001

44


Sociedade Brasileira de MatemĂĄtica

¨ Š

+!,#I'J - K 6 & H ( . L#H%O I%H ( IR/0.%6 798 ( H (') x x x x  = 88 .

§dŽdŽ ­

) Determine todos os nĂşmeros reais x tais que

€  Â‚ ƒ „ Â…  Â† ‡ ˆŠ‰ ‹tÂŒ ‡{‚ Â? ‹tÂŒ ÂŽ Â? ‚ Â? ÂŽ ‘  cƒ ‹tÂŒ †  Â’ ÂŒ ‹t‹“Œ•” Â?  Â‹t– ÂŒ ‚—‡ ˜'ÂŒ ™ Â’ š ›2Âœ 22 . Com efeito, 7 88 88 3 x x x  = ⇒ x ĂŠ da forma com k ≤ x . x k 1) Se x ≼ 0 entĂŁo x ≼ 3 pois, se x < 3 terĂ­amos A resposta ĂŠ

x x x x  < 3 4 = 81 Como

88 1 = 29 tem-se que 29 ≼ x x x  ≼ 27 e daĂ­ : 3 3

88

x x x  = 27 ⇒ x = 27 (nĂŁo serve) 88 22 = 28 7 88 x x x  = 29 ⇒ x = (nĂŁo serve) 29 2) Se x ≤ 0 entĂŁo

x x x  = 28 ⇒ x =

x < −3 ⇒ x  ≤ −4 ⇒ x x x x  ≼ 3 3 â‹… 4 > 88 por outro lado,

x ≼ −3 ⇒ x  ≼ −3 ⇒ x x x x  ≤ 3 4 = 81 e portanto, nĂŁo hĂĄ soluçþes negativas. ¨ dŠ

+!Â?07 6 3 5 & (')

¨dŒdŒ Œ

) Prove ou disprove a seguinte afirmativa :

Todo nĂşmero racional positivo pode ser escrito sob a forma

a 2 + b3 onde a , b , c5 + d 7

c e d sĂŁo inteiros positivos.

€  Â‚ ƒ „ Â…  Â† ‡ ž Â&#x; ‹t Â?Š vž•Â? ‹tÂŒ ÂŽ †0Œ•† ‡•’ ÂŒ ‹tÂĄ ÂŒ ‚Š¢  v”¤£ ‡ ‹“‡ ‰ Â? ÂŽcÂŒ ™ ÂĽ ˆŠ›2Âœ A afirmativa ĂŠ verdadeira. Fazendo a = x 3 y 2 , b = x 5 y 2 , c = xy e d = x 2 y para quaisquer inteiros positivos x e y , temos: EUREKA! N°11, 2001

45


Sociedade Brasileira de Matemática

(

) ( ) ( ) 2

3

a 2 + b3 x3 y 2 + x5 y 2 = 7 c5 + d 7 (xy )5 + x 2 y

=

x 6 y 4 + x 15 y 6 x = x 5 y 5 + x 14 y 7 y

Como todo número racional pode ser obtido pelo quociente de dois números inteiros positivos, a afirmação está provada.

¦%§

¨!©#ª «%«'¬ ­%®°¯%±'±

¦

) Um número de 10 algarismos é dito interessante se todos os seus algarismos são distintos e ele é um múltiplo de 11111. Quantos números interessantes existem ?

t ² v t { c t¡ ¢ v £c t q c¥ 2 A resposta é 3456 . Seja I um número interessante então I ≡ 0 + 1 + 2 + ⋅ ⋅ ⋅ + 9 ≡ 0(mod 9 )

(

)

Logo, I = 99999 ⋅ N = 10 5 − 1 ⋅ N para algum número natural N de 5 algarismos. Digamos, N = a1 a 2 a 3 a 4 a 5 logo, I = 10 9 a1 + ⋅ ⋅ ⋅ + 10 5 a5 − 10 4 a1 − ⋅ ⋅ ⋅ − 10a 4 − a5 I = 10 9 a1 + ⋅ ⋅ ⋅ + 10 6 a 4 + 10 5 (a 5 − 1) + 10 4 (9 − a1 ) + ⋅ ⋅ ⋅ + 10(9 − a 4 ) + 10 − a 5 . Sejam d1 , d 2 ,..., d 9 , d10 os dígitos de I, nesta ordem, então d1 + d 6 = 9 , d 2 + d 7 = 9 , d 3 + d 8 = 9 , d 4 + d 9 = 9 e d 5 + d10 = 9 . Como os únicos pares de dígitos cuja soma é 9 são (0,9), (1,8), (2,7), (3,6) e (4,5) o número de possibilidades para d1 , d 2 ,..., d 9 , d10 é 9 ⋅ 8 ⋅ 6 ⋅ 4 ⋅ 2 ⋅ 1 ⋅ 1 ⋅ 1 ⋅ 1 ⋅ 1 = 3456

¯

¦'§

¨!³#´>µ·¶'¸ ¬ ­'®!¯'±'±%±'¹ Resolva a equação

1 1 + 2 x 4 − 3x

(

)

2

=1

º » ¼ ½ ¾ ¿ » À Áq Á ÃtÄ ¼ À »v¥ Á Ãt¼ Å Æ » Ç º ¿ »v¥#Ä ½ ¼ »vÈcº ¥ É2Ê As soluções da equação são

2 3 2 3 (1 − 2 cos 20º ) , 2 3 (1 + 2 cos 40º ) e , 3 3 3

2 3 (1 + 2 cos 80º ) . 3

EUREKA! N°11, 2001

46


Sociedade Brasileira de Matemática

Com efeito fazendo-se 4 − 3 x = y + 2 temos que x = x≠

4

2− y 3

para x ≠ 0 e

. Fazendo as devidas substituições e simplificando chegamos a

3

(

)

2 3 e y 3 − 12 y − 8 = 0 . 3 Aplicando-se a fórmula da equação do terceiro grau nesta última temos: y y 3 − 12 y − 8 = 0 o que implica em y = 0 ⇒ x =

2

y=3 −

3

2

−8 −8  − 8   − 12   − 8   − 12  +  −   +  +3 −  +  2 2 3 2      2   3 

3

ou y = 3 4  3 1 + 3i + 3 1 − 3i  = 3  

onde θ = 2kπ +

 θ  θ  4  3 2cis + 3 2cis −   3  3  

π π (1 + 6k ) . Daí, y = 4 cos para k = 0,1,2 e assim obtemos as 3 9

outras raízes. Obs: Também é possível chegar às soluções fazendo

1 1 3 1 = cosθ , = senθ ⇔ + =4⇔ x cosθ senθ 4 − 3x 3 1 senθ + cos θ = 2 senθ cos θ ⇔ sen(θ + 30°) = sen(2θ ) . Podemos fazer 2 2 as mesmas θ = 30°,θ = 50° , θ =170° ou θ = 290° , e obtemos ⇔

soluções

(ainda que escritas numa outra forma:

2 3 1 2 3 1 1 = = , (1 + 2 cos 40°) = , 3 cos 30° 3 cos 290° cos 70° 2 3 1 2 3 1 1 e ). =− (1 − 2 cos 20°) = (1 + 2 cos 80°) = 3 cos 50° 3 cos170° cos10°

EUREKA! N°11, 2001

47


Sociedade Brasileira de MatemĂĄtica

Ă‹%ĂŒ'§

¨!Ă?0ÂŤ'ĂŽ ­'¸ Ă? ­%ÂŽ

Ă‹'ĂŒ%ĂŒ'ĂŒ

š Mostre que existe uma seqßência de inteiros positivos

(a 1 , a 2 ,..., a n ,...) tal que

a12 + a 22 + ... + a n2 ĂŠ um quadrado perfeito para todo

inteiro positivo n .

Âş Âť Âź ½ ž Âż Âť Ă€ Ă Ă? Ă„ ĂƒtĂ‘ ²ÂŠÂź Ă… ÂťvĂ?•Å ĂƒtĂ„{Æ Ă€cĂ„ Ă€ à •Ò Ă„ ĂƒtĂ“ Ă„ Ÿ—Ô ÂťvÇ Ă•cĂ ĂƒtĂ Ă– Ă… Æ Ă„qĂˆcĂ— Ă˜ É2ĂŠ Consideremos a seguinte seqßência: n −1

∑ (a )

2

k

a1 = 3 e a n =

k =1

2

−1

, ∀n ∈ {2,3,4,...}

Desta forma teremos : n

∑ (a )

2

k

k =1

 n −1 2 2 2 2 =  (a k )  + (a n ) = (2a n + 1) + (a n ) = (a n + 1)  k =1 

∑

o que conclui a demonstração. (note que an ĂŠ sempre par, e tambĂŠm ĂŠ igual a a n −1 (a n −1 + 2 ) , para todo n ≼ 3). 2

Ă‹%Ă™'§

¨!Ă?0ÂŤ'ĂŽ ­'¸ Ă? ­%ÂŽ

Ă‹'ĂŒ%ĂŒ'ĂŒ

š Determine o maior número inteiro N que satisfaz as seguintes

condiçþes : N  (a)   possui seus três algarismos iguais. 3 N  (b)   Ê igual à soma de n números naturais consecutivos a partir de 1. 3

Âş Âť Âź ½ ž Âż Âť Ă€ à ÚÛÄ Âź Ÿ—Ä Ă‘ Ă Â•Ăœ ÂťcĂ€ ĂƒtĂ… Ă? ½ Ă Ă–qĂ€ à •Þ Âť Âź Ă„ ÆcĂ€ Ă„qĂ?•Å ĂƒtĂ„ Æ Ă€cĂ„ Ç Ă•0Ă ĂƒÂ“Ă Ă– Ă… ÆcĂ„ Ăˆ Ă— Ă˜ÂŠĂ‰2ĂŠ De acordo com as condiçþes (a) e (b), tem-se que

Ă&#x;  N  n(n + 1) = 111 â‹… k , k ∈ ; 1 ≤ k ≤ 9 3= 2   ou seja, n=

− 1 + 1 + 8¡111¡k 2

EUREKA! N°11, 2001

48


Sociedade Brasileira de MatemĂĄtica

Como n ĂŠ natural, o radicando deve ser um quadrado perfeito o que ocorre somente para k = 6 que substituĂ­do na expressĂŁo anterior nos fornece n = 36 e N N  daĂ­,   = 111 â‹… 6 = 666 ⇒ 667 > ≼ 666 ⇒ 2001 > N ≼ 1998 3 3 e portanto o maior N que satisfaz Ă s condiçþes dadas ĂŠ 2000. Atendendo a um pedido especial do leitor JosĂŠ Renato Carneiro e Carneiro antecipamos a solução de:

Ă™%Ă™'§

¨!�0'à å9â0œ%¸  ­%Ž°¯%¹'¹'¹%š Determine todos os inteiros x e y que satisfazem à equação x 3 + 9 xy + 127 = y 3 .

Âş Âť Âź ½ ž Âż Âť Ă€ Ă ĂŁ Ă„ Æ Ă… Ă Âź Ă— Ă… Æ Ă” Ă Ă… ĂƒtÂťvÂş Âť ä ĂƒtĂ Ă… ĂƒtĂ„ Ç ĂĽ ÂťcĂƒÂ“ĂŚ Ă„ Âź Ă {ç'Ă„ ĂˆcĂ’ è É2ĂŠ As soluçþes da equação sĂŁo (3,7 ) e (− 7,−3) . Fazendo-se y = x + a , substituindo-se e simplificando a equação proposta

(

)

chegamos a: (9 − 3a )x 2 + 9a − 3a 2 x + 127 − a 3 = 0 (*) Esta equação deve possuir soluçþes reais para possuir raĂ­zes inteiras. Seu discriminante ĂŠ:

(

)

(

D = (9 − 3a ) − 4 ⋅ (9 − 3a ) 127 − a 3 = (9 − 3a ) a 3 + 9 a 2 − 508 2

)

Se a ≼ 6 entĂŁo a 3 + 9a 2 ≼ 540 e D < 0 . Para − 2 ≤ a ≤ 2 temos 9 − 3a > 0 e a 3 + 9a 2 = a 2 (a + 9) ≤ 4 â‹… 11 < 508 . Para − 8 ≤ a ≤ −3, a 2 (a + 9) ≤ 64 â‹… 6 < 508, e

para a ≤ −9, a 2 (a + 9) ≤ 0 < 508. Assim, D ≼ 0 somente para a ∈ {3,4,5}. Para

a = 3 obtemos uma contradição em (*) . Para a = 4 , a equação x 2 + 4 x − 21 = 0 ĂŠ satisfeita com x = 3 e x = −7 . Para a = 5 a equação 3x 2 + 15 x − 1 = 0 nĂŁo tem nenhuma solução inteira. As Ăşnicas soluçþes da equação proposta sĂŁo portanto (3,7 ) e (− 7,−3) .

ĂŠ ĂŠ ĂŠ

EUREKA! N°11, 2001

49


Sociedade Brasileira de Matemática

Enviaram soluções de problemas os seguintes leitores da êìë·í êìî ï !

ðcñ òôó%õ öø÷úùû÷ýü4÷#öþ÷ ÿþó ò ÷ ðcò>ö ñ ÷ $ò #ý÷ ñ ñ / 0ñ øò>÷#ö % ÷

1 ó ó ÷ ð 5 ÷ ó%ñ õ ÷ 4 ýó ò õ°ö 4% ÷ 71 ó ó ÷ 94ö ÷ ò °ö ð ò ö ñ ó4ö 4%: õ ;ûó <ô= ó ý?ñ > A÷ @ ó ó 5 ñ ó%õ öøA ÷ ñ õ ò ÷ 5 ñ ó%õ öøA ÷ ñ õ ò ÷N=ù Døò ÷{ñ 5 øñ ó E ò @ ö ñ ÷ ö % ÷

1 ó F òøñ HG ó % ó ò ó ò ó I ÷ 1 õ @ Jó ÷ ÷{ñ ñ K4ó =ù Døò ÷ ñ ùû÷ L5E õ M ñ @'÷{ñ ñ Nó ò ÷ @ ó%ñ õ ÷ °ö % ÷

1 ó @ ó%ñ õ ÷

O òô÷Nö P õ ; ñ ó @ ó%ñ > õ A ÷ @ ñ ó%ò>öýó ö ó%ñ ;ûó'õ Mô÷ @6R%ñ & ÷ @ ñ ó ò öýó ö ó%ñ ;4ó%õ M ÷ @ = ó øñ & ÷ < õ 0ö 4% 6÷

1ôó @ øñ õ E ÷ SQó ÷{ò õ ÷ ðcò>ö{ñ ó4ö ÷ @ õ õ ÷ %$V ÷{7ò óûöý÷ P$ JU ó%õ öý& 4= ó øõ ÷#ðcõ 4ó òôö ñ ðcñ =ó D W ÷ % 6÷

1 ó ð 36 ó%=ñ ò M ñ ÷ ÷þö{ñ 3ø÷N ÷ Hó ó ÷ ò X ó%ñ õ ÷ % 6 Y ó%õ õ ó J Z °ö @4 @ ñ ó òôöþó

ö ó ó 4ü ÷ tó øõ ÷ ó%õ ò þ÷ ù ÿýó ó'õ ÿ ÷#ö ù4ó ò ñ ÷ <ô÷ ñ ó%õ 1ôó 2 9 <ô÷ ñ ó%õ 1ôó 2 9 ÷ t ó øõ ÷ '% % ü4& ÷ t ó øõ ÷ '% % ü4& ÿý ó ó'õ 6 ÿ % ó%õ ;ûó4öø÷{ñ /þð % ó%õ ;ûó4öø÷{ñ /þð ù 1 ö < ÷ ñ ó @ 5 ÷#ö ù4ó ò ñ ÷ / õ Q ûð # ñ òôó =I # ñ òôó =I ÷#ö ù4ó ò ñ ÷ @ ó TE ð G P õ > EV ó 2% # ñ òôó =I <ô÷ ñ ó%õ 1ôó 2 9 <ô÷ ñ ó%õ 1ôó 2 9 # ñ òôó =I %

&

'% 2 3 6

ù

ù

ù ù

ñ ÷ !ö ö ÷ ÿ ñ÷ ñ÷ ñ÷ ñ÷ ñ÷ ñ÷ ñ÷ ñ ÷ !ö ö ÷ ÿ ñ÷ ñ÷ ñ÷ ñ÷ ñ÷ ñ÷ ñ÷ ñ÷ ñ÷ ñ÷ ñ÷ ñ÷ ñ÷ ñ÷ ñ÷

'ó 0÷ ÿ

! " ( ) * + , * + - * . + ( . * + " * + , * . " * . *(. + ( . * + * + . * . ) ( * , * + * . ) * . 8 *(! " ( ) * * + * + " * + , *(. * . 8 * . * ! " (. . B C. * + " = (! " ( ) * + (, * - * ! * + " * + *(+ . * . " * .= * . ) * . 8 * . . (8 ) * ) * += J* + - * + *(+ . (- ( * + , ( * ) * , * - * + 8 *(+ * . ) * . - * . . * . ! ( ) * ! * + * + ! * . " *(. ) ( ) * - * . * + ) ( * + ( ) * , B C) * ) 8 * , ) * - , * ) *( . * + " * + 8 * . ) * . , (, , * - * - + * 8= * 8 ) *(8 ( * + * . * + " * + , *(. ) * . ( ) * . * + ( * ) * , * - * . *( ! * + ) * + * + . * . " * . ) * . +

ó 0÷ ÿ

[&\$]_^a` bEcZd bfe gih$jlk mEn m_k o:p_qEmlrsout vxw_j y m$vzj r

{4|Z|&}~jLp_k$m:p$ y o4r m$v oEn 4h$r o_v

: L _ : _ _ $ L _ : : L

f f f f Q Z _ f f f Z ¢¡:£ ¤ ¥$¦§£¨ ©¦ £Eª ¦:« f f f f Z f & Q f _ f Z f f f ¬ B ¡:£ ¤ ¥$¦§£ ­¦$£uª ¦$« ® ¯f¯f¯f¯f¯Z¯f¯f¯&°²± ³f´_µZµfµfµfµ µfµZµ ¶ ·J¸­¹:º » ¼$½§º¾¸¿½$ºuÀ ½$Á ® ¯f¯f¯f¯f¯Z¯f¯f¯f¯f¯ °Q± ³Z´_µfµfµ µfµZµfµfµfµfµE¶¬·2°J¯¬¹_º » ¼$½§º­°'¯i½$ºuÀ ½:Á

Âuà Ã$Ä:çÅLÆ_ÇÈÄ É Ê$ÃÌË ÊLÇÍÇÈÎ Ï Ð Ê$ÃÌÑ­Ã Ò:ÊÓÐ Ê_ÔEÊ ÃÓÕ_É_Ï Ç Ê$Ã_Ö ×fØLÙ ØÌÚ$Û Ü ÝÌÞLß:Û Û_Ú Ú àÓÚ$Û:Þ4á â_ã ä å à¿ä$Ø$ã:æ_å ãLß à_ç

EUREKA! N°11, 2001

50


Sociedade Brasileira de MatemĂĄtica èĂˆĂŠiĂŞĂŹĂŤĂ“Ă­Ă“ĂŽiĂŻĂˆèùð™ïóòĂˆĂ´ĂŒĂŠiþ™êÏïá܊ø¢èóòĂˆĂ´Ă“ĂŠiòáÊièQÚúÊiè Ăť ĂźLĂ˝ Ăž Ăż Ă˝ Ăż Ă˝ HĂż

48) Doze pintores vivem em doze casas construídas ao longo de uma rua circular e são pintadas ou de branco ou de azul. Cada mês um dos pintores, pegando consigo bastante tinta branca e azul, deixa sua casa e caminha ao longo da rua no sentido anti-horårio. Desta forma, ele repinta cada casa (iniciando na sua) com a cor oposta. Finaliza o trabalho tão longo repinte alguma casa branca de azul. Em um ano, cada casa estarå pintada com a sua cor original sabendo que, no começo do ano, ao menos uma casa estava pintada de azul.

Âş Âť Âź ½ ž Âż Âť Ă€ Ă c Âť ĂƒtÂť Ă„ Ă– ĂŚ ĂƒÂ“Âť"! ç Ă„ # ä ½%$ Ă„'& Ă ĂŚ Âť Ç Ăœ Ŋ Ă€ Ă '( Ă„ ÆcĂ {Ă… ĂƒtÂťvĂˆ Ăœ)( É2ĂŠ Dizemos que uma pintura de uma casa ĂŠ induzida se nĂŁo ĂŠ feita pelo dono da casa. Isso sĂł acontece quando o pintor acabou de mudar a cor da casa imediatamente anterior de azul para branca. Vamos mostrar que cada casa muda de cor exatamente duas vezes, sendo uma induzida e outra nĂŁo, o que claramente implica o resultado. Primeiro vamos ver que uma casa nĂŁo pode mudar de cor trĂŞs vezes. Para isso, supomos por absurdo que alguma casa muda de cor pelo menos 3 vezes, e consideramos o primeiro momento em que uma casa muda de cor pela terceira vez. Nesse caso, pelo menos duas das 3 pinturas sĂŁo induzidas, e portanto a casa imediatamente anterior muda duas vezes de azul para branca antes disso, e portanto muda de cor pelo menos 3 vezes antes desse momento, o que ĂŠ uma contradição. Para terminar, vamos agora ver que cada casa muda de cor duas vezes, o que equivale a dizer que muda (pelo menos) uma vez de forma induzida. Para isso, basta ver que toda casa muda alguma vez de azul para branca (pois uma casa muda de forma induzida quando a casa anterior muda de azul para branca ela muda de forma induzida). Isso ĂŠ obvio para casas azuis. Vamos provar isso por indução no nĂşmero de casas imediatamente anteriores que sĂŁo inicialmente brancas. Para a casa imediatamente anterior esse nĂşmero ĂŠ menor (lembre que hĂĄ pelo menos uma casa azul no inĂ­cio), e portanto em algum momento ela muda de azul para branca, e nossa casa muda de forma induzida, donde muda de cor pelo menos duas vezes, e portanto em algum momento muda de azul para branca â??

EUREKA! N°11, 2001

51


Sociedade Brasileira de MatemĂĄtica

Obs. Note que essa solução não depende da ordem em que os pintores executam seu trabalho (depois de um pintor acabar de pintar, o próximo não precisa ser seu vizinho, mas Ê importante que cada pintor faça o processo uma vez). 52) Quatro retas se interceptam formando quatro triângulos conforme a figura abaixo. A D

E

F

C

B

a) Prove que as circunferências circunscritas aos quatro triângulos possuem um ponto em comum. b) Prove que os centros dessas quatro circunferências são concíclicos (i.e. existe uma circunferência que passa por todos eles).

Âş Âť Âź ½ ž Âż Âť Ă€ Ă * ĂƒtÆ Ă„ Âź Ă€ Âť+( Âť0Âż Âť Ă€ Âť+& Ă„ Ă– Ă‘ Ă…-, à Æ ĂŚ Âť Ç ĂŁ ½/. ½ à •À0Ă Ă’ Ă„ 0 Ă… Ă„ Ă–Â•Ăˆ Ăœ)( É2ĂŠ A P

D

E

F

C

B

Seja P o outro ponto de interseção dos circuncĂ­rculos aos triângulos AED e FEB. Temos que #FBEP ĂŠ inscritĂ­vel (# denota quadrilĂĄtero nesta solução), logo EFˆB = BPˆ E . Da mesma forma EPˆ D ≥ Aˆ EUREKA! N°11, 2001

52


Sociedade Brasileira de Matemática

(#ADEP é inscritível) e APˆ D ≡ AEˆ D ≡ FEˆ B , mas no triângulo FBE temos FBˆ E ≡ Aˆ + Cˆ e FEˆ B ≡ 180° − ( Aˆ + Cˆ + EFˆB ) ⇒ FEˆ B + EFˆB ≡ 180° − ( Aˆ + Cˆ ), mas como BPˆA ≡ APˆD + EPˆD + BPˆE ≡ FEˆB + EFˆB + Aˆ ≡ 180° − Aˆ − Cˆ + Aˆ ≡ 180° − Cˆ , portanto #APBC é inscritível o que implica que o circuncírculo ao triângulo ABC passa por P. Temos ainda que FPˆ B ≡ FEˆ B (# FBEP é inscritível), logo FPˆ D ≡ BPˆ A ≡ 180° − Cˆ , o que nos dá que o #FPDC também é inscritível, com isso os circuncírculos aos triângulos ABC, FDC, FEB e AED possuem o ponto P comum. b) A P O2 I1

O1

D

I2

O3 E

F

C

B O4

Sejam: O1: Centro do circuncírculo a ∆BEF ; O2: centro do circuncírculo a ∆ADE ; O3: centro do circuncírculo a ∆ABC ; O4: centro do circuncírculo a ∆CDF ; Lema: O segmento determinado pelos pontos de intersecção de dois círculos é perpendicular ao segmento determinado pelos centros dos círculos:

M

C1

C2 N

EUREKA! N°11, 2001

53


Sociedade Brasileira de Matemática

∆C1 MC 2 ≡ ∆C1 NC 2 (L.L.L), logo MCˆ 2 C1 ≡ NCˆ 2 C1 o que significa que C1C 2 é

bissetriz de ∆MC 2 N , mas como este triângulo é isósceles então C1C 2 também é altura e C1C 2 ⊥ MN (c.q.d). ∧

Do lema acima temos PB ⊥ O1O3 e PE ⊥ O1O2 , e portanto B P E = O2 O1 O3 e BPˆ E ≡ BFˆE (subentendem o mesmo arco BE no círculo de centro O1). Analogamente DPˆ C ≡ O Oˆ O , mas DPˆ C ≡ DFˆC (subtendem o mesmo arco 2

4

3

DC no círculo de centro O4). Como DFˆC ≡ BFˆE temos DPˆ C ≡ BFˆE ≡ BPˆ E ≡ O2 Oˆ 1O3 ⇒ O2 Oˆ 4 O3 ≡ O2 Oˆ 1O3 . Com isso provamos que #O1O2O3O4 é inscritível.

53) Prove que num círculo convexo dado e para o mesmo número de lados, o polígono regular inscrito é aquele cuja superfície é máxima.

º » ¼ ½ ¾ ¿ » À Á Ò Ä Ã ¼ » Ö * ¼ äcÁ Ãtæ »vÀ Ä º Å ¼ Ó Ä 1 Å Ñ æ » Ã Ç32 Å5456%7/8%4 Å Ö È Ü)9 É2Ê a) Observe inicialmente o polígono convexo A1 A2 A3…An no círculo de centro O. B2 A2 B1

B3

A1 A3

Bn

O A4

An : ;

Tracemos os segmentos OA1 , OA 2 , OA 3 ,..., OA n que encontram a circunferência nos pontos B1, B2, B3,…,Bn; respectivamente. Observe que o polígono B1B2B3…Bn tem superfície maior que o polígono A1A2…An; isto nos garante que o polígono de área máxima deve ser inscrito na circunferência.

EUREKA! N°11, 2001

54


Sociedade Brasileira de Matemática

b) Considere agora, o polígono convexo inscrito de n lados como na figura abaixo: B2 B3

B1

θ2

θ1 θn Bn

θ3

O

B4 = <

A área S do polígono é dado por S = n

onde

R2 (senθ 1 + senθ 2 + ... + senθ n ) 2

∑θ i = 2π . S será máximo quando i =1

n

∑ senθ

i

for máximo.

i =1

Para este problema, vamos utilizar a "desigualdade de Jensen" que diz: suponha que f com concavidade para baixo (côncava) no intervalo I; então para todo x1 , x 2 ,..., x n ∈ I teremos: f ( x1 ) + f ( x 2 ) + ... + f ( x n )  x + x 2 + ... + x n  ≤ f 1  n n   e a igualdade ocorrerá se e somente se x1 = x 2 = ... = x n . Para o nosso problema temos f ( x) = senx e I = ]0, π [ onde θ 1 ,θ 2 ,...,θ n ∈ I ; portanto senθ 1 + senθ 2 + ... + senθ n  θ + θ 2 + ... + θ n   2π  ≤ sen 1  = sen  n n  n    2π e para o caso máximo: θ 1 = θ 2 = θ 3 = ... = θ n = ; ou seja, o polígono é regular n inscrito.

EUREKA! N°11, 2001

55


Sociedade Brasileira de Matemática x 54) Sejam ( x n ) a seqüência definida por x1 = 2, x n +1 = 2 n , ∀n ≥ 1, e ( y n ) a ( y n2001 )

seqüência definida por y1 = 2001, y n +1 = 2001

, ∀n ≥ 1. Prove que

existe c natural tal que y n ≤ x n + c para todo n ∈ > com essa propriedade.

e determine o menor c

?@8%45A%BDC 8"E%F *)45FD0 Ò)8%GHGHI J'* ä%GHF A"K32 L æ3F GH6%L McÜ 9DNPO

Provaremos não que x n + c ≥ y n mas sim x n + c ≥ 44022 ⋅ y n2001 = 22 ⋅ 2001 ⋅ y n pois é fácil ver que só com o fato xn + c ≥ yn não se pode fazer indução. 2001

Escolheremos c de tal modo que x1+ c ≥ 22 ⋅ 2001 ⋅ y12001 . Suponha válida a desigualdade para um certo k então xk +c ≥ 22 ⋅ 2001 ⋅ yk2001 ⇒ xk +1+c = 2 xk +c > 2 22⋅2001⋅ yk , 2001

2001 mas 211 ≥ 2001 ⇒ xk +1+ c ≥ (2001y k ) 2⋅ 2001 = yk2+⋅2001 = yk2001 1 +1 ⋅ yk +1 , 2001

mas y1 = 2001 ⇒ y k2001 ≥ 20012001 > 22 ⋅ 2001 ⇒ x k +1+ c ≥ 22 ⋅ 2001 ⋅ y k2001 +1 o que completa a indução. Vemos que c = 4 satisfaz a condição pois x5 = 265536 > 22 ⋅ 20012002 mas 2001

x5 = 2 65536 < y2 = 20012001 já que 65536 < 20012001 < 2 < 2001, portanto c > 3 e para c = 4 funciona ⇒ c = 4 é o menor valor.

55) Seja S o conjunto de pontos interiores de uma esfera de raio 1 e C o conjunto de pontos interiores de um círculo também de raio 1. Existe alguma função f : S → C tal que d ( A, B) ≤ d ( f ( A), f ( B)) para quaisquer pontos A, B ∈ S? (d (A, B) denota a distância euclidiana entre A e B). ?@8%45A%BDC 8"E%FRQSA%L54UT%F GH,+F'V%AXW5LUYZJDGHJ K3?@CD8\[@J A%4U8+M%?@[@NPO

Seja O o centro da esfera e C o da circunferência. Tome A, B ∈ S tal que A, B e O sejam colineares, EUREKA! N°11, 2001

56


Sociedade Brasileira de Matemática

d ( A; O) = d ( B; O), d ( A; B ) = 2 − ε , ε → 0, ε > 0. Tome também P, Q ∈ S tal que P, Q e O sejam colineares, d ( P; O) = d (Q; O), PQ⊥ AB e d ( P; Q) = 2 − ε . ∧

Agora note que d ( f ( A); f ( B)) ≥ 2 − ε ⇒ ∠f ( A) C f ( B ) → 0, analogamente ∧

d ( f ( P ); f (Q)) ≥ 2 − ε ⇒ ∠f ( P ) C f (Q) → 0.  ε Como d ( A; P ) = d ( B; P) = 2 1 −  então 2  ∧  ε d ( f ( A); f ( P)), d ( f ( B ); f ( P )) ≥ 2 1 −  ⇒ f ( P) C f ( A) → 90°. 2 

Agora vamos tomar P' e Q' do mesmo modo que tomamos P e Q, porém em um plano perpendicular ao plano de A, B e P. Temos que  ε d ( P ' ; P) = d ( P' ; Q) = 2 1 − .  2 ∧

Mas como vimos antes, f ( P ' ) C f (Q' ) → 0 e f ( P ' ) C f ( A) → 90°, logo f ( P ' ) → f ( P ) ou f ( P ' ) → f (Q) ⇒ d ( f ( P ' ); f ( P )) → 0 ou d ( f ( P' ); f (Q)) → 0, o que é um absurdo, pois  ε d ( f ( P ' ); f ( P )), d ( f ( P ' ); f (Q)) ≥ 2 1 −  → 2 ❏ 2  *)]%^H_ `%aDb a ,"8%c'd3_ ,+eXfD,g8"a h%iDj-8+`%_Dc c 8%45k%lDm%a c a _'b 8%4-_De%8%^H_ lDn 8"`Xapo q r s t u v wyx zD{ | } z |

~y{D v {

u v u v u s z

© ©

©

z

© r s Dz ys Xu } zR

z X y

© X

r z Du x |Du r v z

©

r z Du x |Du r v z

} r u | w z

z v r Dz r s s v

©

u { zDt z w { X y

©

x

©

x

| w v r z r }Pw

EUREKA! N°11, 2001

57


Sociedade Brasileira de MatemĂĄtica òĂˆĂ´Â™ĂŠiþ™êÏïá܊ø¢èóòĂˆĂ´Ă“ĂŠiòáÊièQÚúÊiè

š

›yÂœ Â?Â&#x;ž  ¥ ¢ ÂŁÂ•Âœ ¤"ÂœÂŚÂĽ §  ¨ Âœ Šª¢Œ§ Â?Xž– ¢ Šª¤ Âœ ÂĽ ÂŤ ÂŹ ­ § ¤"ÂĄ Âœ ¤¯ŽXŠ Âœ ° ÂĽ § £•¢ ¤Â?ÂŽÂ&#x;Š ÂœPÂŽÂ&#x;Âœ ¤ ¨ Âœ ¤+§¹¤ ÂŤ ² § ¤ ¨ ­ § ¤³¥ §´Â? ÂœXž Âœ ¤ ÂŽÂ&#x;Š Âœ ° ÂĽ § ÂŁ ¢ ¤%ÂŽX¢ Š ¢Â•Âœ ¤\ÂŽÂ&#x;Š ¾¡œ¸ £Â•Âœ ¤ Â? š ÂŁ § Š Âœ ¤ .

56) Para cada nĂşmero n, seja f(n) a quantidade de maneiras que se pode expressar n como a soma de nĂşmeros iguais a 1, 3 ou 4. Por exemplo, f(4) = 4, pois todas as maneiras possĂ­veis sĂŁo 4 = 1 + 1 + 1 + 1, 4 = 1 + 3, 4 = 3 + 1, 4 = 4. Demonstrar que se n ĂŠ par, f(n) ĂŠ um quadrado perfeito. 57) Dado n nĂşmeros reais x1, x2, ‌, xn satisfazendo as condiçþes x1 + ... + x n = 0 e x12 + ... + x n2 = 1 , prove que existem i e j tais que 1 xi x j ≤ − . n 2 p −1 − 1 ĂŠ o quadrado 58) Determine todos os primos p para os quais o nĂşmero p de um inteiro. 59) Um pedestal de altura a sustenta uma coluna de altura b (b > a). A que distância do monumento se deve colocar um observador para ver o pedestal e a coluna sob ângulos iguais? 60) Se num triângulo ABC , A = 2B, provar que a 2 = b(b + c). Obs.: a, b e c sĂŁo, respectivamente, os lados opostos aos ângulos A, B e C. 61) Na figura abaixo um quadrado EFGH foi colocado no interior do quadrado ABCD, determinando 4 quadrilĂĄteros. Se a, b, c, e d denotam os medidas das ĂĄreas dos quadrilĂĄteros, mostre que a + b = c + d .

a c d b

EUREKA! N°11, 2001

58


Sociedade Brasileira de Matemática

62) Se ABCD é um quadrilátero convexo tal que os lados AB, BC , CD e DA medem respectivamente a, b, c e d e que α, β, σ e γ são as medidas dos seus ângulos internos, mostre que a medida da área desse quadrilátero, denotada por (ABCD), é dada por: D

γ

d

A α

c

d1

σ

a

C

b

β B

(ABCD) =

( p − a )( p − b)( p − c)( p − d ) − abcd cos 2 δ onde:

a+b+c+d 2 β +γ α +σ δ= (a fórmula também vale se fizermos δ = ) 2 2 p=

v z tDs u }\º »\u\º ¼\½ v z ½ zD}·w z }+½ z v¾~y¿ u v zÁÀp{ r z s {D u }+Ã Ä z v-w s u ~y XÅ Æ ½ v zDt s u º Ç ½ v zD½ zD}·w z±½Dz v v u s z È r |Dz± Du"É s r Xu r v à u s Ê Ë y Å Æ)½Dv z tDs u }̺ ÍÌuÌ» Î̽Dv z ½Dz } w z }̽Dz v É }P s z u s s z ½Dz |DÏ © r ¸zÐÃ É s ¿ ½ r X ŠƯ½Dv z tDs u }Ñ» ÒÑuÑ» ÓѽDv z ½Dz }PwHzD}ѽ¸z v'~ v s zD}Ñ q zD u } à w s yÅ

©

EUREKA! N°11, 2001

59


Sociedade Brasileira de Matemática øÕÔiï×ÖÓð÷ø

é­êSØ ö

ò×Ù Ú÷ø

Û@Û@ÜUÜ5Ü ÝSÞ%Ü5ß [@à5*)á)*Ñâ)ã *)ä Ü5Þ/å@Ü ã)*Ñá å±ß *)æ å@ß'ç æ Ü5è * 2)à51 å@Ü ä¦éDê ë'a ì [@^Hj5,+aDj5^H_ V/_ c a í'î@ï¸ð ñDò ópôöõRòD÷ªø ùDú ûDó ä@aD]%k%h%`/_ V%_Dc a í î@ïDð ñ òDóüô ý ò ÷ þ ÷Dÿ5÷ 'ð 3ó

æ%a ^Hb aDj5^H_ V/_ c a í î@ïDð ñDò óüô 'ò ÷RóDù ÿ5ùDð ó 3ú ÷ þÁý ô ÷

ó ú Dópô yýªòD÷Ró ùDÿ5ù ð 3ó ú ÷ þ R÷ /þ ÷ Dù úDò¸ó ò ñ ò ÷ 3ó ñ 2)à51 å@Þ 2 Ü 1 å ã)ä Ü æ ç ã Ü5Ý

[@^Hj5,+aDj5^H_ V/_ c a í'î@ï¸ð ñDò ópôDý¾òD÷ þ ÷ ÿ5÷ Rð 3ó ä@a ]/k%h%`%_'V%_ cDa í'î@ï ðDñ ò ópô '÷ ó ú ¸óyô pýªò ÷'ó ùDÿ5ù¸ð 3ó

1 Ü5ÜDÝSÞ%Ü5ß [@à5*)á *Ñá å¦ß * Ü5Ý î@ï ð ñDò óyô ý ò ÷ Rñ ó

Û@Ü5ÜDÝZÞ/Ü5ß [@àU* á)*Ñá å¦ß *)æ å@ß ç æ ÜUè *Ñá Ý

è)Ý 2 å

ñ 'ò ÷¾ø ù û¸ó

ä )Þ

î@ñ úDÿ ñ ópô )û ÷ Û@Þ%Ü5Ü ÝSÞ%ÜUß [@à5*)á)*

Ü 2)æ å ã)2)* è ÜUÝ 2)* Þ á å¦ß *)æ å@ß'ç æ Ü5è *

ýªñ ý ' ! ò ÷ªø ù û ó " ñ þ û ú ÿ5óDúpô # þ ÿ5ñ òDó¸þ $)ú òDó¸þ Û 1 Ü ÝSÞ%Ü5ß [@à5*)á)*

Ü â å ã Ý * ß'å ã Ü5è * 2)*Ñá å¦ß *)æ å@ß ç æ ÜUè *

'ñ ò ÷ þ ÷ ÿ-÷ ð 3ó % úDñ þ ô $& 3ù ùDñ

Ü1

ÝZÞ/Ü5ß [@àU* á)*

Ü â å ã Ý * ß å ã Ü5è *)2)*Ñá å±ß *)æ å@ß'ç æ Ü5è * î@ï ð ñDò óyô 'ò¸÷ óDù ÿ5ù ð 3ó

♦♦♦ ♦♦ ♦

EUREKA! N°11, 2001

60

2 Ü 1 å ã)ä Ü æ ç ã Ü *


Sociedade Brasileira de Matemática ÚÓéiéiô ð ï×Ö÷øiðÓéiô ïÈèóô ï ÔÑÙ éÑÖ÷øÕÙ7è

')( *,+ - .0/ 1 23*)45/ 6 78*9')+ *0:<; 1 ')6 OQP,+ R 1TS3*8.U.8PEV W3*U*82 ')VQ_ P06 *L`a*0( *0+ _b1 pbPEV<PU2A/ R 1 qb*U2 PEr s *8.UtU1 VHt8P06 1 .vu + P0/ + P `A*0+ 6 1b.yu + P82bP0+ / tU1Tpb1 + _bPU.]zh*06 ( P0/ + * `)6 *Er<2A/ 1 ')+ tU1 VHtE~HP,+ `)6 *Er<.yS3*UP8R?/ V<_bP0+ `)6 PU1 VQ1 +h`)+ PU. tU EVHt,/ 1 2b*8.y 3PU78PU. 6 / 1T ,P8_b* Wb13. EVQ_ P06 *L401ar< H* u 6 1A+ EVQt0/ 1TuHP0+ + P,/ + *L r / ( *0+ UP8.vu / 6 ~<1 / . P06 PL23P)')+ *E:<; 1&z0+ *8R P *U2b1 rQ.0( 81 78*EV / 6 2 PvuHP,+ VH*EV<2bPU.L4 *8*U2 8*Ut8 ArQP,6 / VHPvu<*EO / 1 6 *yW31b; *U.)')+ *EVQt,/ Ob/ * 81b 81&pbPEV - t0/ 1 2bPy ,P06 1T 3P8R 1 81b 81&u + *EV<t0/ .8tU1T ,P06 1 / OQ1 VH*8R / + P0V<Pv b*E ?*U1A ?* 81b.8 9`A*0+ 6 1b.yzE/ V<R 1& <P0/ 78*U. 81b.8 9`)6 137 PU.L4 *0+ *,/ 7 * 81b.8 9 *U.,¡Q*0+EW,r<*U.yu / 6 ~H1 81b.8 v rb/ W31b.U*U.]zE/ Vb~H1 81b.8 9s5/ P0/ + *9')6 7UP8. a*0+ tUP,6 1&WAr<¢ / V<1 2 PL£ 6 / 78P,/ + * / t0/ 1&SbP,+ VH*EV<2bPU.yp3P8 HP,+ + * rH / VQ*,6 7U*y L/ + *EV<2b*L23P)'9( 1 + / ( a*0+ tU1AV<23P .9`A*U78*06 t8*EV<R Pvu + *EV<¥U* zU*EO 6 1TW3132 + / _ 1 *EV<*U.U.0/ ( zU*Er 6 1TS3PEVb+f/ ArHPL`)+ r< bP,/ 7 *92bPv / ( *9 0+ ¨ WbP0/ V<*06 2b1 PEV t,~b/ + 1ª')+ *E *E 8/ WA/ tU*0+ 2 1 ')( 1 + / ( Wb1aO<P0+ R 1 s5/ <P0rªpb*,+ + 1b. 4E 0+ _A/ 1 `)6 «0rH2A/ 1 Wb*E( 13. 45/ 6 70/ 1 2 Pyp3*0+ + 1b.y ,P06 1 q *U2bPEr uHP0+ + P,/ + *L 1 ( PU. q 1a( «U.y ,P0VH 0VH23P Wb1b2A+f/ _ar<PU. sE*06 2 PEV OQP,+ _ª')+ *E:H; 1 2b*L45/ 6 7U* ®v*U_ VHP0+ zUP,+ P0/ + *y H1a¡<PU. ®v*06 2bP0( *0+ L¨}`a*0VH*06 6 /

=?>A@<B0C =?>A@<X @HC = cA\Ud ef^ GL[U\]gh[8Y8i<J8FQj8G9[8\9k0[UY8l e G I KLcLghk0C =?>A@ waxbC =?{0>bcA|}wbD G8C = cAG l U 8D Gv ?\UGUd8J0^ [8G9[8J9BED d8iHD C =?>AxA BEkE AgUoEC =?>b k M)C = cAG l U 8D GvgUe JU 8J8C = cAG l U 8D G9oED dU Y8l J0^ C =?>A@ gho0C =?>A@<N C =?>bc) U 3G8n mhG8IHi<G8C =?>A@ { m8C =?>A@ {0 C = N ^ Y8 UGygh[8Y iQJ8iHD G8dUJ8l [U\8J8l C =?>Ag8MLC =?>A@ w3N5C =?>A@<M9k0C =? cAM)cA|?>3o {8C =?>A@<o cAC =?>A@ { m8C = o0D I<e \8nLJ9 D e Y8l J0^ [ \vgUd8I<D dUG8C =?>A@<o cAC =?>A@ mhk0C =?>A@<cAC =? D iH\ YL 3\,^ ^ J8I [UG]ghd8 U\Ud8¦UG8C =?g8IHi<G8l JL b 8i<d8D i<JygU¤Q\,^ J0^ [UGy{hJ IHIHG IHC =? xA{0gUC = cA\Ud ef^ GygU[8Y8iHJ iHD G8dUJ lb }G8 UG8I<C = cAG l U 8D G9kEiHJU\ C =? M)|?>A@bwbN o0C =?>A@ { g8C =?>Ag0mhkEC =?>A U@<\ [8\,^ J8l<[U\ywbGUdU[8­Ud8D J8C =?>A U@<\U[8\,^ J8l<[U\LoE\0^ 8D U\ C =?g8IHi<G8l JL b 8i<d8D i<Jy@Q\U[ \0^ J8l<[8\9N5G8D 8I<C =?{0^ \ ¬ \8D e Y0^ JLM9Y8d8D iHD UJ8lH[U\Lo© }XHG8jUG)[U\LM9\0^ D e D C

EUREKA! N°11, 2001

61

BED FHG8I<JLK9MLN XHY8D Z [8\]@QG0^ J9KLMLN m l Y8n9\Ud8JUYLK9o0c xbJ8e J8lHK]w,x wbD GL[U\)XHJUd8\8D ^ G9K]wbX XHY8d [8D J0 }KLo5{ ?J8 \ JU[8G9Kyw3o MLJUd8JUY8I KLk M oEjUGy{UJ8Y8l GLK)o { oEJUd8e G)k dU[0^ LK9o5{ BED e 0^ D JLKvgUo N G D Ud8D J)KLN cAJUnL UGLN ^ JUdU[ \UKLM9o XHG8j Gv{U\8I<IHGUJLKv{0m \0^ \8I<D d8J9K]{E mU\8l 8n Kv{Uk MLJ0^ D dU 8 9K]{ w wbD GLN ^ J dU[8\9Kyw3o oEjUGy }Y D I K)MLk oEjUGLcaJ0^ l G I KLo { @<l G0^ D JUd8 U UG l D I KLoEc cAJ nL 8D d JLN ^ J8dU[U\LKy{Em mU\8l 8n Kv{Uk @<l G0^ D JUd8 U UG l D I KLoEc oEJ8l ¤HJU[8G,^ Kym8k @<G0^ e J l \8ZHJ)K9c)g {UD ^ J8i<D i<J8§UJLKLo { oEX [8G8I caJ8n9 G8I K9o©{ oEX [8G8I caJ8n9 G8I K9o©{ xbG8¤HJ] YUJ8FHYLKvwbX BEG8l e Jvwb\U[UG8dU[8J9KywbX {UG0^ e G9kEl \U 0^ \LKvwbo wb\8iHD ¬ \LKv{Eg XHY8J ZH\8D ^ G9KymUk {UG0^ e G9BE\ l ¦8G9Kywb oEjUGLc)^ D I<e G8¤HjUG)KLo g XHJ8e J, }KLN oEX [8\LM9\0^ D e D<KywbX


CONTEÚDO

AOS LEITORES

2

XLII OLIMPÍADA INTERNACIONAL DE MATEMÁTICA Problemas e Soluções

3

XVI OLIMPÍADA IBEROAMERICANA DE MATEMÁTICA Problemas e Soluções

13

ARTIGOS COMO FERMAT E BÉZOUT PODEM SALVAR O DIA Antonio Caminha Muniz Neto

25

GRAFOS E CONTAGEM DUPLA Carlos Yuzo Shine

31

OLIMPÍADAS AO REDOR DO MUNDO

40

SOLUÇÕES DE PROBLEMAS PROPOSTOS

49

PROBLEMAS PROPOSTOS

59

AGENDA OLÍMPICA

60

COORDENADORES REGIONAIS

61


Sociedade Brasileira de Matemática

AOS LEITORES Nesta última edição de 2001 publicamos soluções dos problemas da Olimpíada Internacional e da Olimpíada Iberoamericana deste ano, nas quais o Brasil obteve ótimos resultados, que deixaram toda a nossa comunidade olímpica muito contente. Por falar na comunidade olímpica, lembramos que neste ano foi criado o nível universitário da OBM, cujas provas e soluções serão publicadas no próximo número da Eureka!. Aproveitamos para saudar os novos (e antigos) olímpicos universitários. Mais uma vez agradecemos o grande número de soluções e problemas propostos que são enviados pelos leitores, e que nos ajudam a fazer a Eureka!. Como um estímulo adicional a nossos colaboradores, vamos dar um reconhecimento especial aos leitores que resolverem mais problemas propostos,ou aqueles problemas propostos que considerarmos mais difíceis, ou que estiverem há bastante tempo sem solução. Aguardem e continuem colaborando! Finalmente, agradecemos a colaboração de Carlos Yuzo Shine, Eduardo Tengan e Pablo Ganassim, que ajudaram na revisão deste número, e desejamos um ótimo 2002 a todos.

Os editores.

EUREKA! N°12, 2001

2


Sociedade Brasileira de Matemática

XLII OLIMPÍADA INTERNACIONAL DE MATEMÁTICA Problemas e Soluções PROBLEMA 1

Seja ABC um triângulo acutângulo com circuncentro O. Seja PA uma altura do triângulo com P no lado BC. Considere que BCˆ A ≥ ABˆ C + 30° . Prove que CAˆ B + COˆ P < 90° . SOLUÇÃO DE ALEX CORRÊA ABREU (NITERÓI – RJ): A

90 – Cˆ

90 – B

O α A–α

C

P

B

M

Sejam ACˆ B = C , ABˆ C = B, CAˆ O = A e COˆ P = α ⇒ CP = b cos C . Seja M o ponto médio de BC. CM a Como COˆ M = Aˆ e COˆ P = α ⇒ POˆ M = A − α ⇒ tg Aˆ = e = OM 2OM PM onde BC = a , AC = b , AB = c e R = OA = OB = OC ⇒ tg ( Aˆ − α ) = OM CM − PM ˆ ˆ tg A − tg A − α CP ⋅ OM OM = = = tgα = tg Aˆ − Aˆ − α = ˆ ˆ CM ⋅ PM OM 2 + CM ⋅ PM 1 + tgA ⋅ tg A − α 1+ 2 OM b cos Cˆ ⋅ R cos Aˆ já que OM = R cos Aˆ , = CP = b cos Cˆ a 2 ab 2 2 ˆ − R cos A + cos Cˆ 4 2

( (

))

(

(

)

EUREKA! N°12, 2001

3

)


Sociedade Brasileira de Matemática

a a a − CP = − b cos Cˆ e CM = e, pela lei dos senos no triângulo 2 2 2 ABC ⇒ b = 2 RsenBˆ e a = 2RsenAˆ ⇒ 2R 2 senBˆ cos Cˆ cos Aˆ 2senBˆ cos Cˆ cos Aˆ ⇒ tgα = 2 = , mas R cos 2 Aˆ + R 2 sen 2 Aˆ − 2 R 2 senAˆ senBˆ cos Cˆ 1 − 2senAˆ senBˆ cos Cˆ 2senBˆ cos Cˆ cos Aˆ senAˆ ⋅ = como tgα > 0 ⇒ 1 − 2senAˆ senBˆ cos Cˆ > 0 ⇒ tgα ⋅ tgAˆ = 1 − 2senAˆ senBˆ cos Cˆ cos Aˆ 2 senBˆ senAˆ cos Cˆ  1  = e como 2x 2 − x −1 < 0, ∀x ∈ − ,1 ⇒ 2 sen 2 Aˆ − sen Aˆ < 1  2  1 − 2 senBˆ senAˆ cos Cˆ 1 ∀Aˆ ∈ (0,90°), mas sen Cˆ − Bˆ ≥ pois 90° > Cˆ − Bˆ ≥ 30° e a função seno é 2 ˆ (0,90°) ⇒ senAˆ ⋅ sen Cˆ − Bˆ ≥ senA ⇒ crescente no intervalo 2 2 ˆ ˆ ˆ ˆ ˆ ˆ ˆ ˆ ˆ ˆ ⇒ − senA ≥ −2 senAsen C − B ⇒ 2 senA − 2senAsen C − B ≤ 2 sen A − senA < 1 ⇒ PM =

(

)

(

(

)

)

( ) ⇒ 2 senAˆ (senAˆ − sen(Cˆ − Bˆ ))< 1 ⇒ 2 senAˆ (sen(Bˆ + Cˆ ) + sen(Bˆ − Cˆ ))< 1 ⇒

2 senAˆ senBˆ cos Cˆ pois 1 − 2 senAˆ senBˆ cos Cˆ 1 − 2 senAˆ senBˆ cos Cˆ > 0 ⇒ tgAˆ ⋅ tgα < 1 ⇒ tg Aˆ + α = tgα + tgAˆ > 0 ⇒ Aˆ + α < 90°. ⇒ 4 senAˆ senBˆ cos Cˆ < 1 ⇒

(

)

PROBLEMA 2

Prove que a a 2 + 8bc

+

b b 2 + 8ca

+

c c 2 + 8ab

≥1

para quaisquer números reais positivos a, b, e c. SOLUÇÃO DE ALEX CORRÊA ABREU (NITERÓI – RJ):

a a 2 + 8bc

+

b b 2 + 8ac

+

c c 2 + 8ba

≥ 1 ⇔ a 2 (b 2 + 8ca)(c 2 + 8ba) + b 2 (a 2 + 8cb)(c 2 + 8ba) +

c 2 (b 2 + 8ca)(a 2 + 8bc) ≥ 513a 2b 2 c 2 + 8(a 3b 3 + a 3c 3 + c 3b 3 ) + 64abc(a 3 + b 3 + c 3 ) Elevando ao quadrado, obtemos EUREKA! N°12, 2001

4


Sociedade Brasileira de Matemática

(I) 2 513a 2 b 2 c 2 + 8(a 3 b 3 + a 3 c 3 + c 3 b 3 ) + 64abc(a 3 + b 3 + c 3 ) ⋅ ⋅ (ab c 2 + 8ab + ac b 2 + 8ac + bc a 2 + 8bc ) ≥ 510a 2 b 2 c 2 − 8(a 3 b 3 + a 3 c 3 + b 3 c 3 ) mas pela desigualdade das médias vale:

(II) 8(a 3b3 + a 3c 3 + c 3b3 ) ≥ 24a 2b 2c 2 e donde:

64 abc ( a 3 + b 3 + c 3 ) ≥ 192 a 2 b 2 c 2

(III) 2 513a 2 b 2 c 2 + 8( a 3 b 3 + a 3 c 3 + c 3 b 3 ) + 64abc(a 3 + b 3 + c 3 ) ≥ 27abc portanto basta mostrar que: (IV) (ab c 2 + 8ab + ac b 2 + 8ac + bc a 2 + 8bc ) ≥ 9abc . É fácil ver que (II), (III) e (IV) implicam (I). Dividindo (IV) por abc, obtemos 8 ab2 + 1 + 8 ac2 + 1 + 8 bc2 + 1 ≥ 9 , e fazendo c

b

a

ab ac bc = u , 2 = v, 2 = w temos que provar que: 8u + 1 + 8v + 1 + 8 w + 1 ≥ 9 , 2 c b a dado que uvw = 1. Usando várias vezes a desigualdade entre as médias aritmética e geométrica, obtemos 8u + 1 + 8v + 1 + 8w + 1 ≥ 3 ⋅ 6 512uvw + 64(uv + uw + vw) + 8(u + v + w) + 1 ≥

≥ 3 ⋅ 6 512 uvw + 192 3 u 2 v 2 w 2 + 24 3 uvw + 1 = 3 ⋅ 6 729 = 9 , o que prova (IV), e de (II), (III) e (IV) obtemos (I), c.q.d.. PROBLEMA 3

Vinte e uma meninas e vinte e um meninos participaram numa competição matemática. • •

Cada participante resolveu no máximo seis problemas. Para cada menina e cada menino, existe pelo menos um problema que foi resolvido por ambos.

Prove que existe um problema que foi resolvido por pelo menos três meninas e pelo menos três meninos.

EUREKA! N°12, 2001

5


Sociedade Brasileira de Matemática

SOLUÇÃO DE CARLOS STEIN NAVES DE BRITO (GOIÂNIA – GO):

Considere a tabela: O

...

A

O1

O2

O3

O21

A1

X1,1

X1,2

X1,3

X1,21

A2

X2,1

X2,2

X2,3

X2,21

A3

X3,1

X3,2

X3,3

X3,21 . . .

. . . A 21 X 21,1 X21,2 X21,3

...

X 21,21

Cada intersecção Xi,j é a questão que a menina Ai e o menino Oj fizeram em comum (se fizeram mais de uma em comum, escolha uma delas qualquer) Lema: em cada linha i da tabela temos pelo menos 11 Xi,k's com a propriedade (I):

esse Xi,k aparece pelo menos 3 vezes nesta linha (contando ele mesmo). Prova: Se por absurdo há menos de 11, então há pelo menos 11 que não têm essa

propriedade (são 21 no total). Então cada um desses se refere a uma questão que aparece no máximo 2 vezes na linha. Mas se há pelo menos 11 Xi,k’s e para cada questão no máximo dois Xi,k’s se referem a ela, então há no mínimo 6 questões referidas nesse grupo. Mas uma linha só tem no máximo 6 questões referidas, pois a menina i fez no máximo 6 questões. Assim todas 6 questões feitas pela menina aparecem no máximo 2 vezes. Absurdo, pois 6 × 2 = 12, mas na linha aparecem 21 (21 > 12) questões. Assim o lema é verdadeiro. Há o lema análogo de que em cada coluna j há pelo menos 11 Xkj’s com a propriedade (II): cada um se refere a uma questão que aparece pelo menos 3 vezes na coluna. A prova é análoga. Contando os Xi,j’s com a propriedade (I) temos pelo menos 11 × 21 (21 linhas). Analogamente pelo menos 11 × 21 com a propriedade (II). Se não houvesse nenhum Xi,j que tem ambas propriedades, teríamos pelo menos 11 × 21 com propriedade (I), outros 11 × 21 com propriedade (II), somando 22 × 21 Xi,j’s, absurdo, pois há só 21 × 21 Xi,j’s. Assim há um Xi,j que tem ambas propriedades. Assim ele se refere a uma questão y que aparece 3 vezes em sua coluna e 3 vezes em sua linha. Como aparece 3 vezes em sua coluna (em 3 linhas distintas), as 3 meninas relativas a essas 3 linhas fizeram a questão y (pois tem ela em comum). EUREKA! N°12, 2001

6


Sociedade Brasileira de Matemática

Analogamente 3 meninos fizeram a questão y, analisando as 3 colunas distintas. Assim 3 meninos e 3 meninas fizeram a questão y, o que sempre ocorre para certa questão y. SEGUNDO DIA DURAÇÃO: 4 horas e meia. PROBLEMA 4

Seja n um inteiro ímpar maior do que 1 e sejam k1 , k 2 ,..., k n inteiros dados. Para cada uma das n! permutações a = (a1 , a 2 ,..., a n ) de {1,2,..., n}, defina S (a) =

n

∑k a . i

i

i =1

Prove que existem duas permutações b e c, b ≠ c, tais que n! é um divisor de S (b) − S (c). SOLUÇÃO DE CARLOS STEIN NAVES DE BRITO (GOIÂNIA – GO):

Se não há S(b) – S(c) tal que n! S (b) − S (c) , e há n! permutações, cada permutação é de uma classe módulo n!. Se não,existem b e c tais que S (b) ≡ S (c)(mod n) ⇒ S (b) − S (c) ≡ 0(mod n). Absurdo.

∑ S (a ), o somatório de todas n! permutações possíveis. Cálculo de ∑ S ( a ) : Seja

i

i

Cada ki aparece em (n – 1)! permutações multiplicando cada coeficiente (de 1 a n). Logo no total temos (n − 1)!(1 + 2 + ... + n ) de cada ki. Assim, *

∑ S (a ) = (n − 1)! i

(n + 1)n ⋅ 2

∑k

i

=

(n + 1) ⋅ n! ⋅ 2

∑k

i

≡ 0(mod n!)

(pois n é ímpar).Temos também que cada permutação é de uma classe, logo

∑ S (a ) ≡ 1 + 2 + ... + n!≡ i

Como n > 1, 2 n! , e

n!(n! + 1) (mod n!). 2

n!(n!+1)  n!  n! n! ≡   ⋅ n! + ≡ (mod n!) . 2 2 2 2 n! De * e ** temos que 0 ≡ (mod n!) , absurdo. 2

**

∑ S (a ) ≡ i

EUREKA! N°12, 2001

7


Sociedade Brasileira de Matemática

Logo há S(b) e S(c) tais que S(b) ≡ S(c) (mod n!) ⇒ n! S (b) − S (c). PROBLEMA 5

Num triângulo ABC, seja AP a bissectriz de BAˆ C com P no lado BC, e seja BQ a bissectriz de ABˆ C com Q no lado CA. Sabemos que BAˆ C = 60° e que AB + BP = AQ + QB. Quais são os possíveis valores dos ângulos do triângulo ABC? SOLUÇÃO DE THIAGO BARROS RODRIGUES COSTA (FORTALEZA – CE): A

Q c

B

b

α α P

c

a

Nomenclatura: a = BC b = AC c = AB α = ABˆ Q = CBˆ Q 2ac cos α Lema 1: BQ = (a + c) Prova: note que a área do triângulo ABC é igual a soma das áreas dos triângulos ABQ e BQC. Logo a ⋅ c ⋅ sen2α c ⋅ BQ ⋅ senα a ⋅ BQ ⋅ senα = + ⇒ 2 2 2 a ⋅ c ⋅ 2 senα ⋅ cos α = (a + c )BQ ⋅ senα ⇒ 2ac cos α BQ = (a + c)

EUREKA! N°12, 2001

8


Sociedade Brasileira de Matemática

Lema 2: AQ =

bc (a + c)

Prova: Pelo teorema das bissetrizes internas temos que:

(a + c) CQ a CQ + AQ (a + c) b bc . = ⇒ = ⇒ = ⇒ AQ = (a + c) AQ c AQ c AQ c Lema 3: BP =

ac (b + c )

Prova: análogo ao lema 2.

Do problema, temos que AB + BP = AQ + QB. Então, pelos lemas: 2ac cos α ac bc a + b + c b + 2a cos α +c= + ⇒ = ⇒ b+c a+c a+c b+c a+c ⇒ (a + c )(a + b + c ) = (b + c )(b + 2a cos α ) ⇒ a 2 + c 2 + 2ac + ab + bc = b 2 + bc + 2a (b + c )cos α ⇒

a 2 + c 2 − b 2 + 2ac + ab = 2a(b + c )cos α . (I) Agora, pela lei dos cossenos no ∆ABC : a 2 + c 2 − 2ac cos 2α = b 2 ⇒ a 2 + c 2 − b 2 = 2ac cos 2α Assim, de (I): 2ac cos 2α + 2ac + ab = 2a(b + c )cos α ⇒ 2c(cos 2α + 1) + b = 2(b + c )cos α ⇒

(

)

2c cos 2 α − sen 2α + cos 2 α + sen 2α + b = 2(b + c )cos α ⇒

4c cos α − 2(b + c )cos α + b = 0. Resolvendo a equação: 1 b cos α = ou cos α = . 2 2c 1 Se cos α = , o único valor que ele poderia assumir era 60°, nesse caso 2 ˆ ABC = 2α = 120° = 180° − 60° = 180° = BAˆ C , absurdo. b Então cos α = . Pela lei dos senos no triângulo 2c 2

EUREKA! N°12, 2001

9


Sociedade Brasileira de Matemática

2senα cos α b sen2α sen2α = ⇒ cos α = = ⇒ 2 sen(120° − 2α ) 2 sen(120° − 2α ) c sen(120° − 2α ) sen(120° − 2α ) = senα (note que cosα ≠ 0) ⇒ Veja que α deve pertencer ao intervalo (0, 60°) para que o triângulo possa existir. Assim, o único valor possível para α seria fazendo α = 120° − 2α ⇒ α = 40° ⇒ ABC = 80° ⇒ BCA = 40° Logo os ângulos do triângulo são: ABˆ C = 80°, BCˆ A = 40° e BAˆ C = 60° . ABC :

PROBLEMA 6

Sejam a, b, c, d inteiros com a > b > c > d > 0. Considere que ac + bd = (b + d + a − c)(b + d − a + c). Prove que ab + cd é um número primo. SOLUÇÃO DE ANTONIO CAMINHA MUNIZ NETO (FORTALEZA – CE):

Simplificando a condição do enunciado obtemos a2 + c2 – ac = b2 + d2 + bd, ou ainda (2a – c)2 + 3c2 = (b + 2d)2 + 3b2 (*). Suponha, por contradição, que ab + cd = p, p primo. A condição a > b > c > d > 0 garante que p = ab + cd ≥ 4×3 + 2×1 = 14, de modo que p ≥ 17. Por outro lado, 2p = 2ab + 2cd = (2a – c)b + (b + 2d)c, de modo que mdc(2a – c, b + 2d) divide 2p. Para mdc(2a – c, b + 2d) ser par, deveríamos ter b e c pares, donde p = ab + cd seria par, o que é um absurdo. Logo, mdc(2a – c, b + 2d) = 1 ou p Afirmação: mdc(2a – c, b + 2d) = 1: Suponha que mdc(2a – c, b + 2d) = p. Então (*) nos daria que p23(b2 – c ), e daí p2(b2 – c2), uma vez que p ≠ 3. Porém, p = ab + cd > b, de modo que 2

0 < b2 – c2 < p2, um absurdo.

EUREKA! N°12, 2001

10


Sociedade Brasileira de Matemática

Agora, sejam x = 2a – c, y = b + 2d. Então mdc(x, y) = 1 e segue de (*) que x2 – y2 = 3(b2 – c2), ou ainda (x – y)(x + y) = 3(b – c)(b + c) Consideraremos dois casos separadamente: i. b ≡/ c (mod 2): o fato de ser p = ab + cd garante que mdc(b, c) = 1. Como b + c e b – c são ímpares, isto implica em mdc(b + c, b – c) = 1. Um argumento análogo implica em mdc(x + y, x – y) = 1 também. Se 3  (x + y) então x – y = mdc(x – y, (b + c)(b – c)) = mdc(x – y, b + c) mdc(x – y, b – c) e x + y = 3mdc(x + y, (b + c)(b – c)) = 3mdc(x + y, b + c) mdc(x + y, b – c) Escrevendo α = mdc(x – y, b – c), β = mdc(x – y, b + c), γ = mdc(x + y, b – c) e finalmente δ = mdc(x + y, b + c), temos x – y = αβ e x + y = 3γδ. Por outro lado, b – c = mdc(b – c, x – y)mdc(b – c, x + y) = αγ e b + c = βδ, analogamente. Resolvendo para a, b, c e d obtemos 4a = αβ + βδ + 3γδ – αγ, 2b = αγ + βδ, 2c = – αγ + βδ e 4d = – αβ – βδ + 3γδ – αγ. Daí 4p = 4(ab + cd) = βγ (α2 + 3δ2) Mas b ≡/ c (mod 2) implica em b + c e b – c ímpares, de modo que α, β, γ e δ são ímpares. Assim, temos os seguintes casos: a) βγ = p: nesse caso, α 2 + 3δ 2 = 4 e daí α = δ = 1. Portanto, a = b, um absurdo. b) βγ = 1: neste caso, β = γ = 1 e daí c = d, um novo absurdo. ii. b ≡ c (mod 2): Nesse caso b e c devem ser ímpares, pois do contrário 2 dividiria ab + cd = p. Assim, nas notações acima, temos x e y também ímpares. Segue que mdc(b + c, b – c) = mdc(x + y, x – y) = 2

EUREKA! N°12, 2001

11


Sociedade Brasileira de Matemática

(pois já temos mdc(x, y) = 1). Se 3(x + y) (o outro caso é novamente análogo), escrevendo  x − y  x + y   b − c  b + c      = 3  2 2     2  2 

e pondo  x− y b−c x− y b+c , ,  , β = mdc  , 2  2   2  2

α = mdc 

 x+ y b−c x+ y b+c , ,  , δ = mdc   2  2   2  2

γ = mdc 

chegamos, como acima, a 2a = αβ + βδ + 3γδ – αγ, b = αγ + βδ, c = – αγ + βδ e 2d = – αβ – βδ + 3γδ – αγ. Daí, p = ab + cd = βγ(α 2 + 3δ 2) Nem β nem γ são iguais a p, pois do contrário teríamos b > p, o que contradiz ab + cd = p. Logo β = γ = 1 e daí c = d, um novo absurdo.

EUREKA! N°12, 2001

12


Sociedade Brasileira de MatemĂĄtica

XVI OLIMP�ADA IBEROAMERICANA DE MATEMà TICA Enunciados e Resultado Brasileiro A XVI Olimpíada Iberoamericana de Matemåtica foi realizada na cidade de Minas, Uruguai no período de 24 a 29 de setembro de 2001. A equipe brasileira foi liderada pelos professores Carlos Gustavo Tamm de Araujo Moreira, do Rio de Janeiro – RJ e Pablo Rodrigo Ganassim, de São Paulo – SP. O Resultado da Equipe Brasileira BRA 1 BRA 2 BRA 3 BRA 4

Carlos Stein Naves de Brito Daniel Massaki Yamamoto Daniel Pinheiro Sobreira Thiago Barros Rodrigues Costa

Ouro Prata Ouro Prata

PRIMEIRO DIA DURAĂ‡ĂƒO: 4 horas e meia. PROBLEMA 1

Dizemos que um nĂşmero natural n ĂŠ "charrua" se satisfaz simultaneamente as seguintes condiçþes: - Todos os algarismos de n sĂŁo maiores que 1 - Sempre que se multiplicam quatro algarismos de n, obtĂŠm-se um divisor de n. Demonstrar que para cada nĂşmero natural k existe um nĂşmero "charrua" com mais de k algarismos. SOLUĂ‡ĂƒO DE DANIEL MASSAKI YAMAMOTO (SĂƒO PAULO – SP):

Vou construir nĂşmeros “charruasâ€? (o que, afinal, significa isso?), utilizando os algarismos 2 e 3. Multiplicando-se 4 algarismos, obtemos um nĂşmero da forma d = 3. . 2 Âą., onde 0 ^ . ^ Vou fixar o final do nĂşmero em “3232â€? e completar o resto apenas com 2’s. EntĂŁo 0 ^ . ^ 3 2 n (I) Para que dGn, basta garantirmos que:  4 (II) 2 n

EUREKA! N°12, 2001

13


Sociedade Brasileira de MatemĂĄtica

É fĂĄcil ver que 2 4 3232 ⇒ 2 4 M â‹… 10 4 + 3232, ∀M ∈ ⇒ 2 4 n , logo (II) estĂĄ satisfeito. Para garantirmos a divisibilidade por 9 , basta que a soma dos algarismos seja divisĂ­vel por 9. Pegue n0 = 22223232 . Note que 9G Podemos colocar blocos de nove 2’s Ă esquerda de n0 que nĂŁo afetarĂĄ a divisibilidade por 9. k  k  Adicionando   blocos, o nĂşmero terĂĄ mais de 9 â‹…   + 8 > k dĂ­gitos. 9 9 PROBLEMA 2

A circunferĂŞncia inscrita no triângulo ABC tem o centro O e ĂŠ tangente aos lados BC, AC e AB nos pontos X, Y e Z, respectivamente. As rectas BO e CO intersectam a recta YZ nos pontos P e Q, respectivamente. Demonstrar que se os segmentos XP e XQ tĂŞm o mesmo comprimento, entĂŁo o triângulo ABC ĂŠ isĂłsceles. SOLUĂ‡ĂƒO DE THIAGO BARROS RODRIGUES COSTA (FORTALEZA – CE): A

Z

Y

βQ O .

Îą B

Ďƒâ€™ Ďƒ

Îą X

C

Pelo teorema do bico, o triângulo BXZ ĂŠ isĂłsceles. Como OB ĂŠ bissetriz do ângulo B , OB ĂŠ mediatriz do segmento XZ . Considere o triângulo ZXP. O ponto P pertence a mediatriz de XZ . Assim, a altura relativa ao vĂŠrtice P e a mediana relativa ao mesmo sĂŁo iguais ⇒ ∆XZP ĂŠ isĂłsceles ⇒ XP = PZ . (*) De maneira anĂĄloga, XQ = QY .(**) . Suponha agora que XP = XQ. EUREKA! N°12, 2001

14


Sociedade Brasileira de Matemática

Então, por (*) e (**), QY = PZ (* * *) . Seja β = AZˆY = AYˆZ (O ∆AZY é isósceles, pelo teorema do bico). Como O é incentro OZ ⊥ AZ ⇒ YZˆO = 90 − β . Da mesma forma, OY ⊥ AY ⇒ OYˆZ = 90 − β ⇒ OYˆZ = YZˆO . (1) Note que OZ e OY são raios ⇒ OZ = OY . (2) Por (***), (1) e (2) temos que os triângulos OZP e OYQ são congruentes ⇒ OQˆ Y = OPˆ Z . (3) Se α = OBˆ Z e σ = OCˆ Y , temos que (no ∆ZPB )

α + ZPˆ O = β = σ + OQˆ Y mas, por (3), OQˆ Y = OPˆ Z ⇒ α = σ , mas ABˆ C = 2α = 2σ = ACˆ B ⇒ ∆ABC é isosceles. PROBLEMA 3

Sejam S um conjunto de n elementos e S1, S2,…,Sk subconjuntos de S (k ≥ 2), cada um deles com pelo menos r elementos. Demonstrar que existem i e j, com 1 ≤ i < j ≤ k , tais que o número de elementos nk comuns a Si e Sj é maior ou igual a r − . 4(k − 1) SOLUÇÃO DE CARLOS STEIN NAVES DE BRITO (GOIÂNIA – GO):

Chamaremos os n elementos de S de b1, b2, ...,bn .Se o número de subconjuntos Si  ai  que contêm {bi} é ai,   é o número de pares de subconjuntos que 2  compartilham bi. n

Temos que

∑a

i

≥ kr , pois é a soma de quantos elementos há em cada um dos

i =1

subconjuntos Si.  x + k   x   x + k − 1  x + 1 , pois:  +   ≥   +  Temos que, para k ≥ 1,   2  2  2   2  x 2 + 2xk + k 2 − k − x + x 2 − x ≥ x 2 + k 2 +1+ 2kx − 2x − 2k − x − k +1 + x 2 + x ⇔ k ≥ 1.

(

)(

) (

)(

EUREKA! N°12, 2001

15

)


Sociedade Brasileira de Matemática n

Logo em

 ai 

∑  2  você pode ir reduzindo a diferença entre os a ’s que a soma só i

i =1

diminui até que você tenha número com diferença máxima, dois a dois, de um. Seja kr = yn + q, 0 ≤ q ≤ n − 1. Assim teremos no fim q ai’s sendo (y + 1) e  ai 

n

(n – q) ai’s sendo y ( para minimizar

∑  2  ). i =1

Temos então:

 ai 

 y + 1  y  y  + (n − q )  = qy + n . 2  2 2 

∑  2  ≥ q

Vamos mostrar que  kr  (yn + q ) y + q − 1 kr  − 1 2  y n  n  ⇔ 2qy + ny − ny ≥  qy + n  ≥  ⇔ 2 2 2 2   ⇔ 2qy + ny 2 − ny ≥ ny 2 +

q2 q2 , o que segue de n ≥ q. + 2 yq − ny − q ⇔ q ≥ n n

Logo temos:  ai    ≥ i =1  2  n

 kr  kr  − 1 n   . 2

Note agora que

 ai 

∑  2  conta com multiplicidade o número de subconjuntos S

i

 k  k (k − 1) com algum elemento em comum. Dividindo por   = , que é o número 2 2 de pares de subconjuntos Si , obtemos o tamanho médio das interseções de dois  kr  kr  − 1 n  ⋅2⋅ 1 ⋅ 1 . desses subconjuntos, que é portanto maior ou igual a  2 k k −1 Basta mostrar que esse mínimo para esse certo subconjunto é maior ou igual a nk e acaba o problema.Para isso,note que r− 4(k − 1)  kr  r  − 1 n   ≥ r − nk ⇔ 4(k − 1) k −1

EUREKA! N°12, 2001

16


Sociedade Brasileira de Matemática

 kr  r  − 1 n  ≥ 4r (k − 1) − nk ⇔ 4(k − 1) k −1 4r 2 k − 4n ≥ 4nrk − 4rn − n 2 k ⇔ 4r 2 k ≥ 4nrk − n 2 k ⇔ 4r 2 ≥ 4nr − n 2 ⇔ n 2 − 4nr + 4r 2 ≥ 0 ⇔ ⇔ (n − 2r ) ≥ 0 (sempre verdade). 2

SEGUNDO DIA DURAÇÃO: 4 horas e meia. PROBLEMA 4

Determinar o número máximo de progressões aritméticas crescente de três termos que pode ter uma sucessão a1 < a 2 < ... < a n de n ≥ 3 números reais. Nota: três termos a i , a j , a k de uma sucessão de números reais formam uma progressão aritmética crescente se a i < a j < a k e a j − a i = a k − a j . SOLUÇÃO DE DANIEL PINHEIRO SOBREIRA (FORTALEZA – CE):

Primeiro vejamos que cada P.A. tem um termo central, pois são formadas de três termos. Vamos estimar o número máximo de posibilidades para o termo central. Lema: Se em uma seqüência temos um termo ak, e temos x números menores que ak e y números maiores que ak , vamos ver que ak é termo central de no máximo min{x, y} P.A.’s. Para cada P.A. onde ak é o termo central, um dos outros termos é maior e o outro menor, logo devo ter a mesma quantidade de termos maiores e menores que ak que participam das P.A.’s de que ak é termo central. Então se ak for termo central de j > min{x, y} P.A.’s teríamos que ter no mínimo j números menores que ak e no mínimo j números maiores que ak, ou seja, x _ j e y _ j, logo min{x, y}_ j. Então o número de P.A.’s com ak como termo central é no máximo min{x, y}. Agora vamos dividir em 2 casos: Caso 1: n ímpar. Temos: a1 < a 2 < a 3 < ... < a n +1 < a n + 3 < ... < a n −1 < a n 2

2

É lógico que a1 e a n não podem ser termos centrais. EUREKA! N°12, 2001

17


Sociedade Brasileira de Matemática

Se pegarmos o número a j , temos ( j − 1) números menores que ele (a1 , a 2 ,..., a j −1 ) e (n − j ) números maiores que ele (a j +1 , a j + 2 ,...a n ). Logo a j é termo central de no máximo min{ j − 1, n − j} P.A.’s. n −1

Então temos que o número de P.A.’s é menor ou igual a

∑ min{ j − 1, n − j}. j =2

n +1 temos j − 1 < n − j ⇒ 2 j < n + 1. Mas para 2 ≤ j < 2 n +1 n +1 < j ≤ n − 1 temos j − 1 > n − j. temos j − 1 = n − j e para Para j = 2 2 Portanto, o número de P.A.’s é menor ou igual a n −1 2

n −1 ( j − 1) + + ∑ 2 j =2 n −3 2

n −1

∑ (n − j ) = j=

= ∑i + i =1

n +3 2

n −3 2

n −1 n − 3  n −1  + ∑ i = 21 + 2 + ... + = + 2 2  2  i =1

 n − 3  n − 3    1 +   2  2   n − 1  2 + n − 3  n − 3  n − 1   = 2 = +   + 2 =  2 2 2  2     

 n − 1  n − 3   n − 1   n − 1  n − 3   n − 1  n − 1   n − 1  + 1 =  =   + =  =  .  2  2   2   2  2   2  2   2  2

2

 n −1  P.A.’s, pois acontece  2 

É fácil ver que a seqüência (1, 2, 3,..., n) tem 

justamente o que queremos: 2 é termo central de uma P.A. o 3 de duas P.A.’s e assim é exatamente como ocorre a igualdade. Caso 2: n par. Temos: a1 < a 2 < a3 < ... < a n < a n 2

2

+1

< ... < a n −1 < a n

Da mesma forma a1 e a n não podem ser termos centrais.

EUREKA! N°12, 2001

18


Sociedade Brasileira de Matemática

Se pegarmos o número a j , temos ( j − 1) números menores que ele (a1 , a 2 ,..., a j −1 ) e (n − j ) números maiores que ele (a j +1 , a j + 2 ,...a n ). Logo a j é termo central de no máximo min{ j − 1, n − j} P.A.’s. n n + 1 ≤ j ≤ n − 1, temos , temos j − 1 < n − j e para 2 2 j − 1 > n − j , logo o número de P.A.’s é menor ou igual a

Mas para

2≤ j≤

n −1

n 2

j =2

j =2

∑ min{ j − 1, n − j} = ∑ ( j − 1) + n −1 2

n −1

∑ (n − j ) =

n j = +1 2

n −1 2

  n  = ∑ i + ∑ (i ) = 21 + 2 + ... +  − 1  =  2  i =1 i =1  n  n     − 1 + 1 − 1  2  2   =  n  n − 1 = n(n − 2) . = 2     2  2  2 4     Mas é fácil ver que a seqüência (1, 2, ..., n) tem

n(n − 2 ) P.A.’s, porque ela 4

satisfaz todas as igualdades. Logo o número máximo de P.A.’s é:  n −1 Para n ímpar:    2  n(n − 2) . Para n par: 4

2

PROBLEMA 5

Num tabuleiro de 2000 × 2001 as casas têm coordenadas (x, y) com x, y inteiros, 0 ≤ x ≤ 1999 e 0 ≤ y ≤ 2000. Uma nave no tabuleiro move-se da seguinte maneira: antes de cada movimento, a nave está numa posição (x, y) e tem uma velocidade (h, v) onde h e v são inteiros. A nave escolhe uma nova velocidade (h', v') de forma que h' – h seja igual a –1, 0 ou 1 e v'– v seja igual a –1, 0 ou 1. A nova posição da nave será (x', y') onde x' é o resto da divisão de x + h' por 2000 e y' é o resto da divisão de y + v' por 2001. EUREKA! N°12, 2001

19


Sociedade Brasileira de Matemática

Há duas naves no tabuleiro: a marciana e a terrestre que quer capturar a marciana. Inicialmente cada nave está numa casa do tabuleiro e tem velocidade (0, 0). Move-se primeiro a nave terrestre e continuam movendo-se alternadamente. Existe uma estratégia que permita sempre à nave terrestre capturar a nave marciana, quaisquer que sejam as posições inicias? Nota: a nave terrestre, que sempre vê a marciana, captura a marciana se depois de um movimento seu cai na mesma posição da marciana. SOLUÇÃO DE CARLOS STEIN NAVES DE BRITO (GOIÂNIA – GO):

A estratégia existe! A nave terrestre deve fazer uma estratégia para ficar na mesma coordenada x, independente do movimento da nave marciana e também na mesma coordenada y, até que isso ocorra simultaneamente e a terrestre pegue a nave marciana. Para ficar no mesmo x : x mi , y mi é a coordenada depois de i rodadas da nave

(

(

)

)

marciana e xti , y ti é a coordenada da terrestre. 0 0 Seja 0 ≤ p ≤ 1999, p ≡ x m − xt (mod 2000).

Se o primeiro h’ escolhido for 1, xt0 aumenta 1 (mod 2000) . ' A partir daí se a nave marciana fizer sua h’ tal que hm − hm = a,

a ≤ 1, a nave

' terrestre fará o mesmo: ht − ht = a.

A partir daí temos ht − hm = 1 ( ht e hm são as velocidades momentâneas da nave terrestre e marciana, respectivamente), isso porque a terrestre já tem ht = 1 por causa da primeira jogada. Esse ht − hm = 1 , se manterá sempre depois da jogada terrestre, pois se nm' − nm = a e

ht' − ht = a; ht − hm = 1 ⇒ (ht' − a) − (hm' − a) = 1 ⇒ ht' − hm' = 1, logo a indução é óbvia. Assim a cada rodada a nave marciana “anda” hm e a terrestre hm + 1. Logo a cada rodada da nave terrestre temos (tudo mod 2000):

      0  0 1  ' x − x = p;  x m + hm  −  xt + hm + 1  ≡ x m0 − x10 − 1 ≡ p − 1;

 

  x1t  x1m    0 m

(

0 t

(

) (

(

))

) (

)

x m2 − xt2 ≡ x m' + n m2 − xt' + n m2 + 1 ≡ x m' − xt' − 1 ≡ ( p − 1) − 1 ≡ p − 2,

( )− (x

Assim por indução finita:

(

x −x ≡ x k m

k t

k −1 m

+h

k m

se x mk −1 − k −1 k t m

) )− 1 ≡ p − k.

xtk −1 ≡ p − (k − 1)

) (

+ h +1 ≡ x

EUREKA! N°12, 2001

20

k −1 m

−x

k −1 t


Sociedade Brasileira de Matemática

Logo depois k jogadas terrestres a diferença entre as coordenadas x m e xt é p − k (mod 2000), assim para k x = 2000a + p, para qualquer inteiro a ≥ 0, p − k ≡ 0(mod 2000), assim estarão na mesma coordenada x. O estudo para as coordenadas y m e y t é análogo, como a nave terráquea tendo v1 = 1 , e depois alternando sua velocidade igual à marciana, mas agora tudo 0 0 módulo 2001, logo se a diferença inicial x m − y t = q temos que depois de k’

jogadas a diferença será. q − k ' (mod 2001), logo, para k y = 2001b + q, estarão no mesmo y. Então a terrestre capturará a marciana quando está no mesmo x e no mesmo y, o que acontece quando o número de rodadas k satisfaz k x e k y :

k = k x = k y ⇒ 2000a + p = 2001b + q, a, b ∈ 1 ⇒ 2001b − 2000a = p − q. Achar solução para isso é fácil: é só adotar a = b = p – q, que é obviamente solução da equação e se p – q < 0, é só pegar b = p – q + 2000 ≥ 0 e a = p − q + 2001 ≥ 0, que também são soluções e agora positivas. Logo na jogada terrestre de número k = 2001 p − 2000q + 2000 ⋅ 2001, ela terá o mesmo x e y da marciana, capturando-a. PROBLEMA 6

Demonstrar que é impossível cobrir um quadrado de lado 1 com cinco quadrados 1 iguais de lado menor que . 2 SOLUÇÃO OFICIAL: D

C

v A

B

u

EUREKA! N°12, 2001

21


Sociedade Brasileira de Matemática

Seja ABCD um quadrado unitário e suponhamos que é possível cobri-lo 1 utilizando cinco quadrados iguais de lado a < . Então, como o diâmetro de cada 2 2 um dos quadrados pequenos é a 2 < , ou seja, menor que a metade da 2 diagonal de ABCD, os vértices A, B, C, D e o centro O de ABCD devem pertencer a quadrados distintos. Chamaremos de QA, QB, QC, QD e QO estes quadrados. Para obter uma contradição, demonstraremos primeiro que a parte do perímetro que cobre cada um dos quadrados QA, QB, QC, QD tem comprimento menor ou igual a 2a. Consideremos, por exemplo, QA; como contém a A, o quadrado intersecta ABCD tal como mostra a figura. Sejam u, v as porções dos lados AB, AD, contidas em QA. Então u + v ≤ 2(u 2 + v 2 ) e

u 2 + v 2 é a hipotenusa de um triângulo retângulo contido

em QA. Logo, u 2 + v 2 ≤ a 2 e, em conseqüência, u + v ≤ 2 .a 2 = 2a. Segue que QA, QB, QC, QD em conjunto cobrem uma porção do perímetro de ABCD de comprimento total menor ou igual a 8a < 4. Então, o quinto quadrado, QO, deve ter interseção não vazia com o perímetro de ABCD. Digamos, por exemplo, que intersecte AB. D

E

C

O

F

B

A

Vamos ver que os cinco quadrados em conjunto não podem cobrir simultaneamente os segmentos AB, CD, e EF, onde E e F são os pontos médios de AD e BC. A demonstração se apoia no lema a seguir: 1 1 Dadas duas retas paralelas l e m a distância e um quadrado de lado a < 2 2 que tem interseção não vazia com cada uma das retas l e m, a soma dos EUREKA! N°12, 2001

22


Sociedade Brasileira de Matemática

comprimentos dos segmentos da interseção do quadrado com as retas é menor que a. Antes de demonstrar o lema, vejamos que é suficiente para completar a solução. De fato, do lema se conclui que ao menos k ≥ 2 dos quadrados QA, QB, QC, QD devem ter interseção com EF (um destes quadrados mais QO cobrirão em conjunto uma porção menor que 2a < 1 do comprimento do segmento EF). Então, novamente pelo lema, esses k quadrados e QO cobrem uma porção do comprimento menor que (k + 1)a dos segmentos AB, CD, e EF. Os restantes (4 – k) quadrados intersectam exatamente um dos segmentos AB e CD, e cada um deles pode cobrir uma porção de comprimento no máximo a 2 do segmento correspondente. Assim, a porção de AB, CD e EF coberta pelos cinco quadrados é menor que: (k + 1)a + (4 − k )a 2 = ka(1 − 2 ) + a (1 + 4 2 ). 1 Como 1 − 2 < 0, k ≥ 2 e a < , observamos que 2 3+ 2 2 <3 ka 1 − 2 + a 1 + 4 2 ≤ 2a 1 − 2 + a 1 + 4 2 = a 3 + 2 2 < 2 o que contradiz que AB, CD e EF estão cobertos.

(

) (

)

(

) (

) (

M h1

)

l

N

L

K

h2

m

Falta demonstrar o lema. Se o quadrado KLMN de lado a <

1 intersecta as retas 2

1 , então dois vértices de KLMN devem estar 2 em lados distintos da banda determinada por l e m (também podem estar em l e 1 implica que esses dois vértices de KLMN são opostos, m). A condição a < 2 digamos K e M. Então, pela mesma condição, os restantes dois vértices do quadrado devem estar contidos na faixa comprendida entre l e m. Sejam h1 e h2 as alturas paralelas l e m que estão a distância

EUREKA! N°12, 2001

23


Sociedade Brasileira de Matemática

correspondentes às hipotenusas dos triângulos retângulos semelhantes que são as duas porções do quadrado que estão fora da faixa comprendida entre l e m, e denotamos ϕ ao menor dos ângulos agudos dos triângulos. Então, o ângulo comprendido entre KM e qualquer reta perpendicular a l e m é 45° − ϕ e a projeção de KLMN sobre uma reta mede a 2 cos(45° − ϕ ). Logo, 1 1 h1 + h2 = a 2 cos(45° − ϕ ) − = a (senϕ + cos ϕ ) − . 2 2 A soma das hipotenusas consideradas é:

h1 + h2 S = (h1tgϕ + h1 cot ϕ ) + (h2 tgϕ + h2 cot ϕ ) = = senϕ cos ϕ

a (senϕ + cos ϕ ) − senϕ cos ϕ

Como senϕ + cos ϕ < 1 + senϕ cos ϕ equivale à desigualdade evidente (1 − senϕ )(1 − cos ϕ ) > 0 e a < 1 , obtemos 2 1 1 1 a(senϕ + cos ϕ ) − a(1 + senϕ cos ϕ ) − a− 2< 2 =a+ 2 < a. S= senϕ cos ϕ senϕ cos ϕ senϕ cos ϕ A demonstração está completa.

EUREKA! N°12, 2001

24

1 2.


Sociedade Brasileira de Matemática

COMO FERMAT E BÉZOUT PODEM SALVAR O DIA Antonio Caminha Muniz Neto, Fortaleza - CE ♦ Nível Avançado. Certamente você, leitor, tem alguma familiaridade com os fatos mais básicos da teoria elementar dos números. Portanto, não objetivo desenvolvê-los aqui de modo sistemático. Para isso você pode consultar [1], [2] ou [3] nas referências. O que vou fazer é mostrar como dois resultados particulares, os teoremas de Bézout e Fermat, podem ser usados para abordar com sucesso alguns problemas interessantes. Para facilitar a leitura, vamos relembrar alguns conceitos e provar os resultados mais centrais para nós. Definição 1: Dados dois inteiros não nulos a e b, definimos o máximo divisor

comum (mdc) como o maior inteiro d que divide ambos a e b. A definição acima certamente faz sentido, uma vez que a e b têm divisores comuns (1, por exemplo) e qualquer inteiro que divida a e b deve ser, em particular, menor ou igual a a. Assim, realmente existe um maior inteiro que divide a e b. Caso esse maior inteiro seja igual a 1, dizemos que a e b são primos entre si, ou ainda relativamente primos. Para nossos propósitos, o seguinte resultado sobre o mdc de dois inteiros será suficiente: Teorema 1 (Bézout): Sejam a e b inteiros não nulos dados e d seu mdc. Então existem inteiros x e y tais que d = ax0 + by0. Mais ainda, se a e b são positivos, podemos escolher x0 > 0 e y0 < 0, ou vice-versa. Prova: Seja S o conjunto dos inteiros da forma ax + by, com x e y inteiros.

Escrevendo a= a×(±1) + b×0, concluímos quea∈ S, e portanto S contém inteiros positivos. Podemos então escolher o menor inteiro positivo pertencente a S, o qual vamos denotar por d. Afirmamos que tal menor elemento positivo d é o mdc de a e b.

Desde que d está em S, devem existir inteiros x0 e y0 tais que d = ax0 + by0. Para provar que d divide a, dividamos a por d: a = dq + r, com 0 ≤ r < d. r = a – dq = a – (ax0 + by0)q = a(1 – x0q) + b(– y0q), que por definição está em S. O fato de ser d o menor inteiro positivo em S, juntamente com 0 ≤ r < d, implica que r = 0, e assim d divide a. Analogamente mostramos que d divide b. Por outro lado, se d’ for qualquer outro divisor comum de a e b, segue que d’ divide ax + by, quaisquer que sejam os inteiros x e y. Em EUREKA! N°12, 2001

25


Sociedade Brasileira de Matemática

particular, d’ divide d = ax0 + by0, de modo que d’ ≤ d. Isso prova ser d o mdc de a e b. Para o que falta, analisemos somente o caso em que a, b > 0 (a análise dos demais casos é totalmente análoga). Como d = ax0 + by0 = a(x0 + tb) + b(y0 – ta), escolhendo t > −

x0 y 0 , obtemos x0 + tb > 0 > y0 – ta. b a

Corolário 1.1: Dois inteiros não nulos a e b são primos entre si se e só se existirem inteiros x e y tais que ax + by = 1.

O outro resultado que usaremos nos problemas a seguir é o pequeno teorema de Fermat. Recordemos seu enunciado e prova: Teorema 2 (Fermat): Dados inteiros a > 1 e p primo, tem-se a ≡ a (mod p). p

Prova: Se a for múltiplo de p nada há a fazer. Senão, como p é primo temos que a

e p são primos entre si. Considere agora os números a, 2a, 3a, ...., (p – 1)a. Dados 1 ≤ i < j ≤ p – 1, como ja – ia = (j – i)a é um produto de dois números não divisíveis por p, temos que ja – ia não é divisível por p. Em linguagem de congruências isso é o mesmo que ja ≡/ ia (mod p ) . Também, nenhum dos números ja é múltiplo de p, e portanto os restos dos números a, 2a, 3a, ...., (p – 1)a na divisão por p formam uma permutação de 1, 2, 3, ..., p – 1. Voltando às congruências, isso implica que a×2a×3a× .... ×(p – 1)a ≡ 1×2×3× .... ×(p – 1) (mod p), ou ainda (p – 1)!ap – 1 ≡ (p – 1)! (mod p) Mas (p – 1)! é relativamente primo com p, e portanto pode ser cancelado em ambos os membros da última congruência acima, dando o resultado desejado. Após essa breve revisão(?), vamos aos problemas! PROBLEMAS RESOLVIDOS Problema 1: Sejam a e b inteiros positivos primos entre si. Então todo inteiro c maior ou igual que o número (a – 1)(b – 1) pode ser escrito da forma c = ar + bs, com r, s ≥ 0. Mais ainda, o menor inteiro com essa propriedade é (a – 1)(b – 1). Solução: Dado c inteiro, o fato de serem a e b primos entre si garante que existem

inteiros x e y tais que c = ax + by (você entendeu por quê?). Seja agora y = da + EUREKA! N°12, 2001

26


Sociedade Brasileira de Matemática

s, onde 0 ≤ s < a. Temos c = ax + b(da + s) = a(x + bd) + bs. Seja r = x + bd. Se c ≥ (a – 1)(b – 1) então (a – 1)(b – 1) ≤ c = ar + bs ≤ ar + b(a – 1), de modo que ar ≥ – (a – 1) , e portanto r ≥ 0. Resta mostrarmos que (a – 1)(b – 1) – 1 = ab – a – b não pode ser escrito da forma ar + bs, com r, s ≥ 0. Supondo o contrário, seja ab – a – b = ar + bs, onde r, s ≥ 0. Então temos a (b – 1 – r) = b(s + 1). Como a e b são primos entre si, segue que a divide s + 1 e b divide b – 1 – r. Como b – 1 – r < b, deve ser b – 1 – r ≤ 0, ou ainda r ≥ b – 1. Também, como s + 1 > 0 e a divide s + 1, deve ser s + 1 ≥ a, ou s ≥ a – 1. Mas aí, ar + bs ≥ a(b – 1) + b(a – 1) = 2ab – a – b > ab – a – b = (a – 1)(b – 1) – 1, uma contradição. Problema 2: (Seleção da Romênia para IMO) Prove que não existe um inteiro n >

1 tal que n divida 3n – 2n. n

Prova: Suponha o contrário, isto é, que para algum inteiro n > 1 tenhamos 3 – 2

n

≡ 0 (mod n). Obviamente 2 e 3 não dividem n. Seja agora p o menor fator primo de n e n = pm (aqui é que usamos ser n > 1, para garantir que n tem fator primo). Nossa hipótese, juntamente com o pequeno teorema de Fermat, nos dão: 3n ≡ 2n (mod n) ⇒ 3mp ≡ 2mp (mod p) ⇒ 3m ≡ 2m (mod p) (*) Se d = mdc(m, p – 1), temos em particular que d divide n. Portanto, o fato de ser p o menor divisor primo de n implica que d = 1. Tome então inteiros positivos x e y satisfazendo mx = (p – 1)y + 1. O pequeno teorema de Fermat de novo, juntamente com (*), nos dão 3 ≡ 3(p – 1)y + 1 = 3mx ≡ 2mx = 2(p – 1)y + 1 ≡ 2 (mod p), o que é um absurdo. Problema 3: Sejam m e n inteiros positivos. Determine o polinômio mônico p, de maior grau possível, que divide simultaneamente os polinômios xm – 1 e xn – 1 . d

Solução: Primeiro, não é difícil vermos que, sendo d o mdc de m e n, então x – 1 m

n

divide ambos x – 1 e x – 1. De fato, seja por exemplo m = dk, com k > 0 inteiro. Então xm – 1 = (xd)k – 1 = (xd – 1)(xd(k – 1) + xd(k – 2) + .... + xd + 1) Mostrar que xd – 1 divide xn – 1 é análogo. A parte mais difícil é mostrar que xd – 1 é o polinômio mônico p de maior grau que divide ambos xm – 1 e xn – 1, e é para isso que precisamos de Bézout. Seja p um polinômio mônico que divide ambos xm – 1 e xn – 1 e z uma raiz complexa de p. Como p divide xm – 1 e xn – 1, temos que z é raiz de ambos esses polinômios. Em outras palavras, zm = zn = 1. Mas o EUREKA! N°12, 2001

27


Sociedade Brasileira de Matemática

teorema de Bézout garante que existem inteiros u e v tais que mu + nv = 1. Isso nos dá zd = zmu + nv = (zm)u×(zn)v = 1u×1v = 1, e assim z é raiz de xd – 1. Como toda raiz de p é também raiz de xd – 1 e como xd – 1 só tem raízes simples, segue que p divide xd – 1. Portanto, xd – 1 é o polinômio mônico de maior grau que divide ambos xm – 1 e xn – 1. Problema 4: (The William Lowell Putnam Competition) Sejam m e n inteiros mdc (m, n)  m    é inteiro. positivos, com m ≥ n. Prove que m n Prova: Para esse problema usamos o teorema de Bézout de um modo bastante x m elegante. Seja S o conjunto dos inteiros x tais que   seja inteiro. Veja que m mn

está em S já que números binomiais são inteiros. Também n está em S, pois  m − 1 n  m n m!   =  =  n m   m (m − n)! n!  n − 1 

Por outro lado note que, se x e y estiverem em S, então ux + vy também estará em S, quaisquer que sejam u e v inteiros. De fato, ux + vy  m  x  m y  m   = u ×   + v ×   , m n mn mn

que é um inteiro. Como o mdc de m e n pode ser escrito da forma mu + nv, para algum par de inteiros u e v, segue que mdc(m, n) está em S. Problema 5: (Seleção do Brasil para a IMO) Determine todas as funções

f : 4 + →= satisfazendo as seguintes condições: *

i. f (1999) = 1. ii. f (ab) = f (a) + f (b), para todos os racionais positivos a, b. iii. f (a + b) ≥ min {f (a), f (b)}, para todos os racionais positivos a, b. Solução: Fazendo a = b = 1 em ii. Vem que f (1) = 2f (1), donde f (1) = 0. Daí,

dados inteiros positivos m e n, temos m 1 f   = f (m) + f   n n

e

donde EUREKA! N°12, 2001

28

1 f   + f (n) = f (1) = 0, n


Sociedade Brasileira de Matemática

m f   = f (m) – f (n) n

Assim, basta calcularmos os valores de f (n), com n inteiro positivo. Seja n = a a p1 1 .... p k k a decomposição de n em fatores primos. Usando ii. várias vezes, vem que f (n) = a1 f (p1) + .... + ak f (pk), de modo que basta calcularmos f (p), com p primo. Afirmamos que f (n) ≥ 0 para todo inteiro positivo n. Para provar esse fato, façamos indução sobre n. Já vimos que f (1) = 0. Suponhamos agora que f (n) ≥ 0 para algum inteiro positivo n. Então f (n + 1) ≥ min {f (n), f (1)} ≥ 0, e isso termina nossa indução. Afirmamos agora que f (2) = 0 ou f (3) = 0. De fato, como 1999 = 3×9 + 493×4, temos que 1 = f (1999) ≥ min {f (3) + f (9), f (493) + f (4)} ≥ ≥ min {3f (3), 2f (2)}, e daí segue o afirmado. Suponhamos que f (2) = 0 ( f (3) = 0 é análogo), e seja p um primo diferente de 2 e de 1999. Tomando k inteiro suficientemente grande, o problema 1 garante a existência de inteiros positivos x e y tais que px + 1999y = 2k. Então 0 = f (2k) ≥ min { f (p) + f (x), f (1999) + f (y)} Como f (1999) + f (y) ≥ 1, segue que f (p) = 0. Então, sendo n = 1999am, com 1999 e m primos entre si, segue que f (n) = a. Mas é imediato verificar que tal função satisfaz as condições impostas no enunciado.

PROBLEMAS PROPOSTOS Problema 6: Generalizando a teoria desenvolvida acima e o problema 1, sejam a1,

a2, ...., an (n ≥ 2) inteiros positivos tais que mdc(a1, a2, ...., an) = d (a definição de mdc que demos no início se aplica nesse caso ipsis literis). Prove que:

i. Existem inteiros x1, x2, ...., xn tais que a1x1 + a2x2 + .... + anxn = d. ii. Se d = 1, mostre que existe um inteiro positivo m0 tal que todo inteiro m ≥ m0 pode ser escrito da forma a1y1 + a2y2 + .... + anyn, com y1, y2, ...., yn ≥ 0. (sugestão: para o item i. imite a prova do texto acima. Para ii., que tal usar indução sobre n > 1? Para uma cota mais precisa para m0, veja [2]). Problema 7: Considere duas progressões aritméticas infinitas e não constantes de

inteiros positivos. Prove que existem infinitos naturais termos de ambas as EUREKA! N°12, 2001

29


Sociedade Brasileira de Matemática

seqüências se e só se o mdc de suas razões dividir a diferença entre seus termos iniciais. (sugestão: use Bézout. É fácil!). Problema 8: (The William Lowell Putnam Competition) Prove que não existe inteiro n > 1 tal que n divida 2n – 1.

(sugestão: use as idéias que apareceram na prova do problema 2). Problema 9: (Olimpíada Búlgara) Determine todos os primos p, q tais que pq

divida o número (5p – 2p)(5q – 2q) (sugestão: se q dividir 5p – 2p e p ≥ q, então mdc(p, q – 1) = 1. Como antes, use Bézout e Fermat). Problema 10: (Seleção da Romênia para IMO) Sejam p, q números primos. Se q

dividir 2p + 3p, prove que q > p ou q = 5. (sugestão: se p > 3 e q ≤ p, então q – 1 < p, donde q – 1 e p são primos entre si. Mais uma vez use Bézout e Fermat).

REFERÊNCIAS [1] Introdução à Teoria dos Números. Antônio Plínio dos Santos. IMPA. Rio de Janeiro, 1998. [2] Olimpíadas de Matemática, um Curso de Introdução, vol. 2. Antonio Caminha. Editora Ipiranga. Fortaleza, 2001 (a ser publicado). [3] Divisibilidade,Congrências e Aritmética módulo n. Carlos Gustavo Moreira, Eureka! no.2 (1998),pp. 41-52. EUREKA! N°12, 2001

30


Sociedade Brasileira de Matemática

GRAFOS E CONTAGEM DUPLA Carlos Yuzo Shine, Colégio Etapa ♦ Nível Intermediário. 1. GRAFOS 1.1 O que são e para que servem grafos?

Define-se grafo como o par (V, A) onde V = {v1, v2,...,vn} é um conjunto de vértices e A ⊂ {{vi, vj}t.q. vi, vj ∈ V, i≠ j} é um conjunto de arestas (na verdade, uma aresta é um par não-orientado de vértices). A representação mais comum de grafos é associar os vértices a pontos e as arestas a linhas que ligam os pares de vértices que as formam. Mas o mais importante é que os grafos podem representar inúmeras situações. Por exemplo, quando você brinca de ligar os pontos, no fundo você está traçando arestas em um grafo onde os vértices são dados (em vez de “ligue os pontos”, poderíamos escrever “areste o grafo”...). Embora pareçam simples, os grafos têm muito mais utilidades, como veremos. Na verdade, a Teoria dos Grafos é uma das partes mais importantes da Matemática, e é muito utilizada principalmente em computação. Exemplo 1.1

Podemos construir um grafo que represente pessoas apertando mãos. Os vértices seriam as pessoas. Ligamos dois vértices (formando assim uma aresta) se duas pessoas se cumprimentaram. Edmilson Gustavo

Eduardo

Carlos

Onofre

Paulo Emanuel

EUREKA! N°12, 2001

31


Sociedade Brasileira de Matemática

Exemplo 1.2

É possível que os cavalos do tabuleiro (I) fiquem na posição do tabuleiro (II) ?

(II) (I) Observação: um cavalo, no xadrez, se movimenta da seguinte forma: ele se move duas casas na vertical ou horizontal e depois se move uma casa na direção perpendicular à direção em que havia se movimentado antes.

Resolução

Vamos numerar as casas do tabuleiro da seguinte forma: 1

2

3

4

5

6

7

8

9

Vamos construir um grafo onde os vértices são as casas do tabuleiro. Ligaremos dois vértices i e j se é possível um cavalo ir da casa i à casa j. Temos então o seguinte grafo (verifique!). EUREKA! N°12, 2001

32


Sociedade Brasileira de Matemática 1 6

8

5

7

3

4

2 9

Coloquemos agora os cavalos nas situações inicial e final, respectivamente: 1

1

6

6

8

5

7

3

5

7

4

2

8

3

4

2

9

9

Inicial

Final

Observe que não podemos ter dois cavalos na mesma casa, assim os cavalos devem sempre estar na mesma ordem no ciclo. Logo não é possível chegar na posição final. Exercícios

01. (IMO) Considere um inteiro positivo r e um retângulo de dimensões AB = 20, BC = 12. O retângulo é dividido em uma grade de 20 × 12 quadrados unitarios. Uma moeda pode ser movida de um quadrado a outro se, e somente se, a distância entre os centros dos quadrados é r . A tarefa é encontrar uma seqüência de movimentos que levem uma moeda do quadrado que tem A como vértice ao quadrado que tem B como vértice. a) Mostre que a tarefa não pode ser feita se r é divisível por 2 ou 3. b) Prove que a tarefa pode ser feita se r = 73. c) Pode a tarefa ser feita quando r = 97? Dicas: Para o item a), use o fato de que um quadrado perfeiro pode deixar somente os restos 0 ou 1 quando divididos por 3 ou 4. Para os itens b) e c), EUREKA! N°12, 2001

33


Sociedade Brasileira de Matemática

construa dois grafos: um que considera a posição da moeda na horizontal e outro na vertical. 1.2. Grau de vértice

Definimos grau de vértice vi como o número de arestas que contêm vi e denotamos d(vi). No último exemplo, o grau de um vértice seria o número de apertos de mão que a pessoa correspondente deu. Exemplo 1.3.

Na cidade de Micrópolis, há 7 telefones. Um candidato a prefeito prometeu que ampliaria a rede de telefonia de modo que cada um dos 7 telefones esteja conectado diretamente a exatamente 5 outros telefones. É possível que ele cumpra sua promessa? Resolução

Se imaginarmos um grafo onde os vértices são os telefones e as arestas, as conexões, teríamos que o grau de cada vértice seria 5. Vamos contar o número de conexões entre dois telefones (ou seja, o número de arestas do grafo). Como de cada telefone sairiam 5 conexões, teríamos a princípio 5.7 = 35 conexões; mas contamos cada conexão duas vezes, uma vez em cada um dos dois telefones a que ele está conectado. Assim, deveríamos ter na verdade 35/2 conexões, o que seria um absurdo. Assim, o candidato a prefeito não pode cumprir sua promessa (não votem nele!!). Este exemplo mostra 1.3. Um teorema importante Teorema. Em um grafo, a soma dos graus de todos os vértices é igual ao dobro do

número de arestas. Em símbolos: no grafo (V, A), d (v i ) = 2 A

vi ∈V

( X denota o número de elementos do conjunto X.) Demonstração

De cada vértice v saem d(v) arestas. Assim, se somarmos os graus de todos os vértices, obtemos o número de arestas multiplicado por dois, pois contamos cada aresta duas vezes (lembre-se de que cada aresta está associada a dois vértices).

EUREKA! N°12, 2001

34


Sociedade Brasileira de Matemática

2. Contagem Dupla

O que acabamos de fazer foi contar algo de duas maneiras diferentes, no caso o número de arestas (na verdade, o seu dobro). Esta idéia de contar duas vezes é às vezes muito útil para demonstrar algumas relações. Exemplo 2.1.

(Combinações) De quantos modos podemos escolher k elementos dentre n disponíveis? Importante: Tal número é representado por  n  – lê-se n escolhe k ou   k 

combinação de n k a k. Resolução

Vamos contar de duas maneiras o número de filas com os k elementos escolhidos. Podemos (i) primeiro escolher os k elementos e colocá-los em filas ou (ii) escolher diretamente os elementos e irmos colocando na fila. Fazendo como em (i), temos  n  maneiras de escolhermos os elementos;   k 

podemos escolher o primeiro da fila de k maneiras, o segundo de k – 1 maneiras, e assim por diante. Assim temos n n   ⋅ k ⋅ (k − 1) ⋅ ... ⋅ 1 =   ⋅ k! k k  maneiras de formar a fila. (lembrete: k ⋅ (k − 1) ⋅ ... ⋅ 1 = k! – lê-se k fatorial). Por outro lado, fazendo como em (ii), temos n maneiras de escolher o primeiro da fila, n – 1 maneiras de escolher o segundo e assim por diante, até o último, que pode ser escolhido de n – k + 1 maneiras. Assim, temos n ⋅ (n − 1) ⋅ ... ⋅ (n − k + 1) maneiras de formar a fila. Logo, de (i) e (ii), concluímos que n   ⋅ k! = número de filas = n ⋅ (n − 1) ⋅ ... ⋅ (n − k + 1) k   n  n ⋅ (n − 1) ⋅ ... ⋅ (n − k + 1) (n − k )! n! ⇒   = ⋅ = (n − k )! k!(n − k )! k! k 

Exemplo 2.2.

(Lema de Sperner) Dividimos um triângulo grande 123 em triângulos menores de modo que quaisquer dois dentre os triângulos menores ou não têm ponto em comum, ou têm um vértice em comum ou tem um lado (completo) em comum. EUREKA! N°12, 2001

35


Sociedade Brasileira de Matemática

Os vértices dos triângulos são numerados: 1, 2 ou 3. A numeração é arbitrária, exceto que os vértices sobre os vértices do triângulo maior oposto ao vértice i não podem receber o número i. Mostre que entre os triângulos menores existe um com os vértices 1, 2, 3. 1 1 1

1

3 1

1

1

2

1

2 2

2

3

2

3

Resolução

Contaremos o número de segmentos 12 (com algumas repetições). Eles aparecem nos triângulos 1

1

1

3 2

2

2

1

2

Digamos que há x triângulos 123, y triângulos 122 e z triângulos 112. Observe que os segmentos 12 internos ao triângulo grande são contados duas vezes (eles são comuns a dois triângulos) e os segmentos do lado do triângulo grande, somente uma vez. Notemos também que os segmentos 12 aparecem duas vezes nos triângulos 122 e 112 e uma vez nos triângulos 123. Assim, 2 segmentos interiores + segmentos nos lados = número de segmentos = x + 2 y + 2 z Mostraremos um fato mais forte que o lema: provaremos que x é ímpar e portanto não pode ser zero. Observando a equação acima, vemos que basta provarmos que o número de segmentos 12 sobre os lados do triângulo grande é ímpar. Como não podemos ter pontos 1 no lado 23 nem pontos 2 no lado 13 , todos os segmentos 12 estão sobre o lado 12 do triângulo grande. Provemos que o número de segmentos sobre o lado é ímpar. Para isso, vamos “colocar” vértices 1 ou 2 no lado 12 . Assim, no começo, temos somente o lado 12 : EUREKA! N°12, 2001

36


Sociedade Brasileira de Matemática 1

2

Na hora de colocar vértices, considere o menor segmento em cujo interior colocaremos o vértice. Poderemos estar em uma das seguintes situações: • Este segmento é do tipo 11 : 1 ou 2 ↓

1

1

2

Se colocarmos 1, o número de segmentos 12 não muda; se colocarmos 2, aumenta de 2. De qualquer forma, a paridade do número de segmentos 12 não muda. • Este segmento é do tipo 22 : 1

2

1 ou 2 ↓

2

Se colocarmos 1, o número de segmentos 12 aumenta de 2; se colocarmos 2, não muda. De qualquer forma a paridade do número de segmentos 12 não muda. • Este segmento é do tipo 12 : 1

1

1 ou 2 ↓

2

Se colocarmos 1 ou 2, o número de segmentos 12 não muda e é claro que a paridade desse número não muda também. Logo a paridade do número de segmentos 12 nunca muda (ou seja, é invariante). Como no começo temos um segmento 12 (o próprio lado 12 ), temos que o número de segmentos 12 no lado do triângulo grande é sempre ímpar, o que completa nossa demonstração. Contar algo de duas maneiras também nos ajuda a demonstrar desigualdades. Exemplo 2.3.

Na terra de Oz há n castelos e várias estradas, sendo que cada uma liga dois castelos e não há mais do que uma estrada ligando diretamente dois castelos. Diz a lenda que se houver quatro castelos ligados em ciclo (ou seja, se existirem quatro castelos A, B, C e D tais que A e B, B e C, C e D e D e A estão ligados), um dragão aparecerá do centro dos castelos e destruirá a Terra de Oz. Mostre que para esta desgraça não acontecer o número de estradas deve ser menor ou igual a 1 + 4n − 3 n / 4.

(

)

EUREKA! N°12, 2001

37


Sociedade Brasileira de Matemática

Resolução

Considere um castelo ligado a outros dois.

 d (v)   pares de estradas. Para cada castelo v do conjunto V dos castelos temos   2  Para a desgraça não ocorrer, observemos que devemos Ter no máximo um par de estradas asociado a um mesmo par de castelos. Assim, a quantidade de pares de estradas é menor ou igual à quantidade de pares de castelos. Logo

 d (v )  n  = pares de estradas ≤ pares de castelos =   v∈V  2  2

∑  ⇒

∑ ((d (v ))

2

)

− d (v ) ≤ n 2 − n

v∈V

⇔ ∑ (d (v )) − ∑ d (v ) ≤ n 2 − n 2

v∈V

v∈V

Sabemos que a soma

v∈V

(*)

d (v) é igual ao dobro do número de estradas

2 A . Além disso, pode-se mostrar (usando a desigualdade entre as médias quadrática e aritmética, ou mesmo Cauchy-Schwarz) que

∑ (d (v))

2

(∑ ≥

v∈V

v∈V

d (v)

)

2

n

=

4A

2

n

Assim (*) ⇒

4A

2

(

2

)

− 2 A ≤ n 2 − n ⇔ 4 A − 2n A − n n 2 − n ≤ 0 (**)

n Resolvendo (**) em A , obtemos

(

)

n n − n 4n − 3 n + n 4n − 3 ≤ A≤ ⇒ A ≤ 1 + 4n − 3 . 4 4 4

EUREKA! N°12, 2001

38


Sociedade Brasileira de Matemática

Exercícios

02. Dizemos que dois poliedros P e Q são equidecomponíveis se é possível cortar o poliedro P em vários poliedros menores e montar, sem deixar espaços vazios e sem sobrar poliedros, o poliedro Q. Sejam α 1 , α 2 ,..., α m os ângulos diédricos (ângulos entre faces adjacentes) de P e β 1 , β 2 ,..., β m os ângulos diédricos de Q. Mostre que se P e Q são equidecomponíveis então existem números inteiros positivos a1 , a 2 ,..., a m , b1 , b2 ,..., bn e um número inteiro k tais que a1α 1 + a 2α 2 + ... + a mα m − (b1 β 1 + b2 β 2 + ... + bn β m ) = kπ A partir desta relação podemos mostrar (isto é um pouco mais difícil!!) que um cubo e um tetraedro de mesmo volume não são equidecomponíveis. 03. Dado n inteiro, seja d(n) o número de divisores de n. Seja d (n) o número médio de divisores dos números entre 1 e n, ou seja, 1 n d (n) = d ( j) n j =1

Mostre que n

1 ≤ d ( n) ≤ i=2 i

n

∑i

1

i =1

Esta desigualdade nos mostra que d (n) ≅ ln n, e que a diferença d (n) − ln n é no máximo 1. 04. (IMO) Num concurso, há m candidatos e n juízes, onde n ≥ 3 é ímpar. Cada candidato é avaliado por cada juiz, podendo ser aprovado ou reprovado. Sabe-se que os julgamentos de cada par de juízes coincidem em no máximo k candidatos. Prove que k n −1 ≥ 2n m

Referências Bibliográficas A parte de grafos foi baseada em um dos capítulos do livro Mathematical Circles – A Russian Experience, que aborda o treinamento russo para as olimpíadas. Muitos dos exercícios de contagem dupla foram extraídos do livro Proofs From The Book, que contém as demonstrações consideradas pelos autores (e também por muitos leitores!) as mais elegantes.

EUREKA! N°12, 2001

39


Sociedade Brasileira de Matemática

OLIMPÍADAS AO REDOR DO MUNDO

Þ O comitê editorial de EUREKA! agradece aos leitores o crescente envio de soluções e aos inúmeros elogios para a seção OLIMPÍADAS AO REDOR DO MUNDO. Continuamos salientando que estamos à disposição na OBM para aqueles que estiverem interessados na solução de algum problema particular. Para tanto, basta contactar a OBM, através de carta ou e-mail. Antonio Luiz Santos

Primeiramente vamos aos problemas propostos deste número 121. (Rússia-2001) Os valores da função quadrática f (x ) = x + ax + b para dois 2

inteiros consecutivos são os quadrados de dois inteiros também consecutivos. Mostre que os valores da função quadrática são quadrados perfeitos para todos os inteiros. 122. (Rússia-2001) Determine todos os números inteiros positivos que podem ser

representados de maneira única sob a forma

x2 + y onde x e y são xy + 1

inteiros positivos. 123. (Rússia-2001) Os

senos dos ângulos de um triângulo são números racionais.

Mostre que os seus cossenos são tambem racionais.

s1 e s 2 de centros O1 e O2 intersectam-se nos pontos A e B . Seja M um ponto qualquer do círculo s1 tal que MA intersecta s 2 no ponto P e MB intersecta s 2 no ponto Q . Mostre que se o quadrilátero AO1 BO2 é cíclico (inscritível) então AQ e BP intersectamse em s1 .

124. (Rússia-2001) Dois círculos

o

125. (Rússia-2001) Eliminando-se o 2000 . algarismo da expansão decimal da

1 (onde p é um número primo maior que 5) obtemos a fração p a irredutível . Mostre que b é divisível por p . b

fração

EUREKA! N°12, 2001

40


Sociedade Brasileira de Matemática

126. (Rússia-2001) Cinco números, um dos quais é 2000, são escritos em um

quadro negro. É permitido apagar qualquer um destes números e o substituirmos pelo número a + b − c , onde a , b e c são três quaisquer dos números restantes. É possível com estas operações obter cinco números iguais a 2000? 127. (Estônia-2001) Quantos números inteiros positivos menores que 20002001

não contém outros algarismos distintos de 0 e 2? 128. (Estônia-2001) Em um triângulo ABC , as medidas dos seus lados são inteiros

consecutivos e a mediana relativa ao lado BC é perpendicular à bissetriz interna do ângulo ∠ABC . Determine as medidas dos lados do triângulo ABC . 129. (Estônia-2001) Considere todos os produtos por 2, 4, 6,...,2000 dos elementos

1 1  1 1 1 do conjunto A =  , , ,..., ,  . Determine a soma de todos 2000 2001 2 3 4 estes produtos. 130. (Eslovênia-2001) Para os inteiros positivos

x e y é verdadeira a igualdade

3x + x = 4 y + y . Mostre que x − y é um quadrado perfeito. 2

2

131. (Eslovênia-2001) Determine todos os números primos da forma 101010...101 . 132. (Eslovênia-2001) Gustavo tentou escrever alguns números utilizando somente

o algarismo 1 e o sinal de “mais”. Ele percebeu , por exemplo, que existem apenas dois inteiros positivos n (13 e 4) para os quais o número 13 pode ser escrito utilizando n 1’s e o sinal de “mais”. De fato, o número 13 pode ser escrito como a soma de 13 1’s ou 11 + 1 + 1 onde 4 1’s foram utilizados. Determine quntos são os inteiros positivos n tais que o número 125 seja escrito utilizando-se n 1’s e o sinal de “mais”. 133. (Eslovênia-2001) Seja ABC um triângulo retângulo de hipotenusa AC .

Sabendo que sobre o lado BC existem pontos D e E tais que ∠BAD = ∠DAE = ∠EAC e EC = 2 ⋅ BD . Determine os ângulos do triângulo.

EUREKA! N°12, 2001

41


Sociedade Brasileira de Matemática

Determine todas as funções satisfazem à equação :  x −3 3+ x f =x + f  x +1   1− x 

134. (Croácia-2001)

f : 5\{-1, 1} → 5 que

135. (Croácia-2001) Se x + y + z = 0 , simplifique

x7 + y7 + z 7 xyz x 4 + y 4 + z 4

(

Sugestão : calcule (x + y ) e (x + y ) 4

)

6

136. (Croácia-2001). Dado o número n = p1 p 2 p 3 p 4 onde p1 , p 2 , p 3 e p 4 são

quatro números primos distintos. Sejam

d1 = 1 < d 2 < d 3 < ⋅ ⋅ ⋅ < d15 < d16 = n os divisores positivos de n . Determine todos os n < 2001 tais que d 9 − d 8 = 22 .

137. (Albânia-2001) Mostre as igualdades:

3π 2π  6π 1 2π  = ⋅ sen − sen 2 (i)  cos 2  ⋅ sen 11 11 11  11 4  3π 2π + 4 ⋅ sen = 11 (ii) tg 11 11 138. (Bielorússia-2001) Em um triângulo isósceles ABC , no qual AB = AC e

∠BAC = 30º , marcam-se os pontos Q e P sobre o lado AB e sobre a mediana AD respectivamente, de modo que PC = PQ ( Q ≠ B ). Determine a medida do ângulo ∠PQC .

139. (Bielorússia-2001) Eduardo escreveu todos os produtos, todas as somas e todos

os valores absolutos das diferenças dos inteiros positivos a1 , a 2 ,..., a100 tomados dois a dois. Qual o maior número de inteiros ímpares obtidos por Eduardo? 140. (Bielorússia-2001). Um inteiro positivo k é chamado bom se existe um inteiro

positivo N com k algarismos em sua representação decimal e tal que o i − ésimo algarismo da esquerda para a direita do número 3N é igual ao EUREKA! N°12, 2001

42


Sociedade Brasileira de Matemática

(k − i + 1) − ésimo

algarismo da direita para a esquerda do número 2N, 1 ≤ i ≤ k . Determine a soma de todos os números bons existentes entre 1 e 100. 141. (Bielorússia-2001) Mostre que

1 1   − 2 ≥ n2  a + − 2 n a a   para todo inteiro positivo n e todo real positivo a . an +

142. (Finlândia-2001) Determine n ∈ N tais que n + 2 divida 2 + 2001n . 2

143. (Itália-2001) Em um hexágono equiângulo, as medidas de quatro lados

consecutivos são, nesta ordem, 5, 3, 6 e 7. Determine as medidas dos outros dois lados. = yx , (i) determine todos os pares de soluções (x, y) tais que x seja um número primo e y um inteiro positivo. (ii) determine todos os pares de soluções (x, y) tais que x e y são inteiros positivos.

144. (Itália-2001) Dada a equação x

2001

145. (Israel-2001) As medidas dos lados de um triângulo ABC são 4, 5 e 6. Por um

ponto D qualquer de um de seus lados, traçamos as perpendiculares DP e DQ sobre os outros lados. Determine o valor mínimo de PQ. 146. (Israel-2001) Dados 2001 números reais x1 , x 2 ,..., x 2001 tais que 0 ≤ x n ≤ 1

para cada n = 1,2,...,2001 determine o valor máximo de  1 2001 2   1 2001   x n  −  x n   2001 2001 n =1 n =1     e onde este máximo é atingido.

147. (Grécia-2000) Seja

2

f : 1 → 5 uma função tal que f (1) = 3 e

f (2m ) + f (2n ) 2 para todos os inteiros não negativos m e n com m ≥ n . Determine a expressão de f(m). f (m + n ) + f (m − n ) − m + n − 1 =

EUREKA! N°12, 2001

43


Sociedade Brasileira de Matemática

148. (Grécia-2001) Fatore a expressão

A = x 4 + y 4 + z 4 − 2x 2 y 2 − 2 y 2 z 2 − 2z 2 x 2 e mostre que a equação A = 2000 não possui solução no conjunto dos números inteiros. 149. (Hungria-2000) O produto de 2001 inteiros positivos distintos possui

exatamente 2000 divisores primos distintos. Mostre que podemos escolher alguns destes 2001 números de modo que seu produto seja um quadrado perfeito. 150. (Hungria-2000) Seja S o número de subconjuntos com 77 elementos de H = {1,

2, 3,...2000, 2001} para os quais a soma dos elementos de cada subconjunto seja par e seja N o número de subconjuntos com 77 elementos de H para os quais a soma dos elementos de cada subconjunto seja ímpar. Determine qual dos dois S ou N é maior e o quanto?

ÞÞÞ

Agora vamos aos comentários e soluções dos leitores para alguns dos problemas apresentados em nossa seção nos números anteriores de EUREKA!. 9.

(Irlanda-1998) Um triângulo ABC possui medidas dos lados expressas por números inteiros, ∠A = 2 ⋅ ∠B e ∠C > 90º . Determine o valor mínimo do perímetro deste triângulo.

SOLUÇÃO DE EINSTEIN DO NASCIMENTO JÚNIOR (FORTALEZA – CE) (com adaptações):

A resposta é 77. De fato, fazendo-se ∠B = x temos ∠A = 2 x , sejam a, b e c as medidas dos lados opostos aos ângulos ∠A , ∠B e ∠C respectivamente. Prolongando-se o lado CA no sentido de C para A até o ponto D de um comprimento igual a c, vemos que o triângulo BCD é semelhante ao triângulo ABC logo, b a ou a 2 = b(b + c ) = a b+c Como estamos procurando um triângulo com o menor perímetro possível, podemos supor que a, b e c não possuem fatores primos comuns pois, de outra forma um outro exemplo menor haveria. De fato, b e c devem ser primos entre si porque qualquer fator primo comum a b e c seria também um fator de a. Alem disso, como o produto b(b + c) é um quadrado perfeito que é igual ao produto de dois números primos entre si tanto b quanto b + c devem eles mesmos serem EUREKA! N°12, 2001

44


Sociedade Brasileira de Matemática

quadrados perfeitos. Deste modo, para m e n inteiros positivos quaisquer primos entre si temos b = m2, b + c = n2, a = mn e daí n a = = 2 cos ∠B (pela lei dos senos) m b π o que implica em O ângulo ∠C = π − 3 ⋅ ∠B é obtuso assim, 0 < ∠B < 6 n 3 < cos ∠B < 1 e portanto, 3 < < 2 . É facil ver que esta desigualdade não 2 m possui soluções inteiras com m = 1, 2 ou 3 daí m ≥ 4 , n ≥ 7 e a + b + c = mn + n 2 ≥ 4 ⋅ 7 + 7 2 = 77 De fato, o par (m, n) = (4, 7) gera o triângulo (a, b, c) = (28, 16, 33) que satisfaz todas as condições do problema. 10.

Em um triângulo ABC tem-se que ∠BAC = 40º e ∠ABC = 60º . Sejam D e E pontos sobre os lados AC e AB respectivamente tais que ∠CBD = 40º e ∠BCE = 70º . Seja F o ponto de interseção de BD e CE. Mostre que a reta que que contém AF é perpendicular à que contém BC. (Canadá-1998)

SOLUÇÃO DE LUCAS DE MELO PONTES E SILVA (FORTALEZA – CE):

Sejam X o ponto de interseção de AF com BC e H o pé da altura que parte de A. Então pela lei dos senos temos nos triângulos CBD e BDA : CD BD DA BD = = e sen40º sen80º sen20º sen40º CD sen40º⋅sen40º = ou . DA sen80º⋅sen20º EA sen60º⋅sen10º = . Pelo Teorema de Ceva, se AX, BD e CE Analogamente EB sen70º⋅sen40º são cevianas concorrentes no triângulo ABC tem-se que: CD EA BX sen40º⋅sen40º⋅sen60º⋅sen10º BX ⋅ ⋅ =1⇒ ⋅ = 1 ou DA EB XC sen20º⋅sen80º⋅sen70º⋅sen40º XC XC 2 sen 20º⋅ cos 20º⋅sen60º⋅ cos 80º sen60º tg 60º = = = 1 ⋅ tg 80º tg 80º BX sen 20º⋅sen80º⋅ cos 20º 2 Como AH temos nos triângulos AHB e AHC respectivamente:

EUREKA! N°12, 2001

45


Sociedade Brasileira de Matemática

tg 60º =

AH AH tg 60º HC e tg 80º = logo = HB HC tg 80º HB

HC XC HC + HB XC + XB = ⇒ = e como ∠BAC é agudo então H está HB XB HB XB entre B e C e portanto, BC BC = ⇒ XB = HB ⇒ X coincide com H. HB XB Logo

13.

(Irlanda-1999). Uma função

f : 1 → 1 satisfaz às seguintes condições:

(i) f (ab ) = f (a ) ⋅ f (b ) se o máximo divisor comum de a e b é 1.

(ii) f ( p + q ) = f ( p ) + f (q ) para todos os números primos p e q.

Mostre que f (2 ) = 2, f (3) = 3 e f (1999 ) = 1999 .

SOLUÇÃO DE MARCÍLIO MIRANDA DE CARVALHO (TERESINA – PI):

Seja p um número primo ímpar. Então, f (2 p ) = f ( p + p ) = f ( p ) + f ( p ) = 2 ⋅ f ( p ) enquanto que f (2 p ) = f (2) ⋅ f ( p ) e daí, f (2 ) = 2.

Agora, Por f (4 ) = f (2) + f (2 ) = 4 ⇒ f (12 ) = 4 ⋅ f (3) . f (12 ) = f (7 ) + f (5) ⇒ f (12 ) = 2 ⋅ f (2) + f (3) + f (2 ) + f (3)

outro

⇒ f (3) = 3.

Finalmente,

f (5) = f (2 ) + f (3) = 5 ⇒ f (15) = 15 ⇒ f (13) = 13

⇒ f (26) = 26 ⇒ f (23) = 23.

Por outro lado,

f (13) = 13 ⇒ f (11) = 11 ⇒ f (33) = 33 ⇒ f (31) = 31

⇒ f (29) = 29.

Logo,

f (2001) = f (3)⋅ f (23) ⋅ f (29 ) = 2001 ⇒ f (1999 ) = 1999

EUREKA! N°12, 2001

46

lado,


Sociedade Brasileira de Matemática

16.

(Estônia-1999) Mostre que o segmento que une o ortocentro e o baricentro de um triângulo acutângulo ABC é paralelo ao lado AB se, e somente se, tg∠A ⋅ tg∠B = 3 .

SOLUÇÃO DE GERALDO PERLINO JÚNIOR (SÃO PAULO – SP) (com adaptações): Sejam D e E os pés das alturas traçadas dos vértices A e C respectivamente. Se H é o ortocentro do triângulo, observe que H pertence à uma reta paralela a AB e CE = 3 . Deste modo, basta mostrar que passando pelo baricentro se, e só se EH CE = tg∠A ⋅ tg∠B . ∠AHE = ∠CHD Como temos também que EH ∠EAH = ∠BCE , isto é os triângulos retângulos CEB e AEH são semelhantes e CE AE CE = . Observando que tg∠A = , obtemos a relação necessária. tg∠B = EB EH AE 23.

(Rússia-1999) A soma dos algarismos de um inteiro positivo n escrito no sistema de numeração decimal é igual a 100 e a soma dos algarismos do número 44n é 800. Determine a soma dos algarismos do número 3n.

SOLUÇÃO DE MARCÍLIO MIRANDA DE CARVALHO (TERESINA – PI):

Sejam a1 , a2 , a3 ,..., ak os algarismos de n e S (n ) a soma de seus algarismos. Então, 44n = 40(a1 a 2 ...a k ) + 4(a1 a 2 ...a k ) = 4(a1 a 2 ...a k ) ⋅ (10 + 1) Logo, se os algarismos de 44 n forem : 4a1 , (4 a2 + 4 a1 ) , (4a 3 + 4a 2 ) , ... , (4a k + 4a k −1 ) e 4a k teríamos S (44n ) = 8 ⋅ (a1 + a 2 + ⋅ ⋅ ⋅ + a k ) = 800 . Assim, se houvesse algum “vaium”, a soma dos algarismos de 44n cairia . Deste modo, todos os algarismos de n são menores ou iguais a 2 donde os algarismos de 3n são 3a1 ,3a 2 ,3a3 ,...,3a k . Assim, S (3n) = 3a1 + 3a 2 + 3a3 + ... + 3a k = 3.S (n) = 300 .

EUREKA! N°12, 2001

47


Sociedade Brasileira de MatemĂĄtica

Acusamos o recebimento de soluçþes de problemas anteriores dos seguintes leitores de EUREKA!: Anderson Torres Geraldo Perlino Ioziel Matos Corrêa Júnior Jorge Silva Júnior Marcelo R. De Souza Marcelo Rufino de Oliveira

São Paulo – SP Itapec. da Serra – SP Rio de Janeiro – RJ Cachoeiro do Itapemirim – ES Rio de Janeiro – RJ BelÊm – PA

Mauro FĂŠlix de Souza Osvaldo Mello Sponquiado

Cordovil – RJ Olímpia – SP

Paulo Alexandre AraĂşjo Sousa Renato Francisco Lopes Mello Wallace Rodrigues de Holanda Miranda

Teresina – PI Jaboatão dos Guararapes – PE Teresina – PI

Wilson Carlos de Silva Ramos

BelÊm – PA

Prob.16, 52, 67, 69, 70, 76, 102, 105, 118, 120. Prob. 115. Prob. 52, 62, 71, 74, 76, 78. Prob. 96, 98, 99, 101, 103, 110. Prob. 95, 115. Prob. 91, 92, 93, 95, 97, 98, 99, 101, 102, 104, 107, 109, 110, 111, 112, 114, 115, 118. Prob. 98. Prob. 29, 30, 38, 56, 57, 59, 63, 66, 67, 69, 74, 98, 107, 114, 118. Prob. 62, 78, 82, 88, 76, 70. Prob. 98, 99, 101, 109, 117. Prob. 97, 98, 99, 101, 102, 104, 106, 109, 112, 114, 116, 117, 120. Prob. 43, 68, 74, 82.

9RFr VDELDÂŤ

4XH IRL GHVFREHUWR XP QRYR SULPR GH 0HUVHQQH HP TXH WHP GtJLWRV H " e R Q~PHUR

p DR ODGR GH

XP GRV GRLV SULPRV FRQKHFLGRV FRP

PDLV GH XP PLOKmR GH GtJLWRV 2 GHVFREULGRU 0LFKDHO &DPHURQ GH

DQRV

p

XP

SDUWLFLSDQWH

GR

*,036

FRRSHUDWLYR SDUD SURFXUDU SULPRV GH 0HUVHQQH &RQVXOWH QD LQWHUQHW D SiJLQD KWWS ZZZ PHUVHQQH RUJ SULPH KWP

EUREKA! N°12, 2001

48

XP

SURMHWR


Sociedade Brasileira de Matemática

SOLUÇÕES DE PROBLEMAS PROPOSTOS

Publicamos aqui algumas das respostas enviadas por nossos leitores.

56) Para cada número n, seja f(n) a quantidade de maneiras que se pode expressar n como a soma de números iguais a 1, 3 ou 4. Por exemplo, f(4) = 4, pois todas as maneiras possíveis são 4 = 1 + 1 + 1 + 1, 4 = 1 + 3, 4 = 3 + 1, 4 = 4. Demonstrar que se n é par, f(n) é um quadrado perfeito. SOLUÇÃO DE MÁRCIO ASSAD COHEN (RIO DE JANEIRO – RJ):

O coeficiente de xn em (x + x3 + x4)k conta exatamente uma vez cada modo de representar o número n na forma α + 3β + 4γ com α + β + γ = k , ou seja, conta os modos de representar n utilizando 1’s, 3’s e 4’s com um total de k números. Variando k, segue que f(n) é o coeficiente de xn na série formal g ( x) =

∑ (x + x

3

+ x4 )k .

k =0

Somando a PG, fatorando, e decompondo em frações parciais tem-se: 1 g ( x) = 1 − x − x3 − x4 g ( x) =

1 1 x+2 1 x+3 + ⋅ = ⋅ 2 2 5 1− x − x2 (1 + x )(1 − x − x ) 5 1 + x 2

Utilizando respectivamente soma de PG e o método desenvolvido para séries formais no artigo da Revista Eureka! No. 11 tem-se: 1 = 1 − x 2 + x 4 − x 6 + ... 1+ x2 1 = F0 + F1 x + F2 x 2 + F3 x 3 + ... 2 1− x − x onde Fn é o n-ésimo número de Fibonacci, com F0 = F1 = 1, Fn+2 = Fn+1 + Fn. Se n é par, n = 2t, o coeficiente de xn no desenvolvimento de g(x) será então:

[x ]g ( x ) = 2 ⋅ ( − 1) n

EUREKA! N°12, 2001

49

t

+ F2 t −1 + 3 F2 t 5


Sociedade Brasileira de Matemática

Utilizando que Fn =

1 5

n +1

)

− β n +1 , onde α =

1+ 5 1− 5 ,β = : 2 2

2 ⋅ (−1) + F2t −1 + 3F2t = t

2 ⋅ (−1) t + [( F2t −1 + F2t ) + F2t ] + F2t = 2 ⋅ (−1) t + F2t + 2 + F2t = 1

2 ⋅ (−1) t +

5 1

2 ⋅ (−1) t +

5

2t + 3

)

1

− β 2t +3 +

[(α + α )α −1

2t + 2

5

(

2 t +1

)

− β 2t +1 =

)

− β + β −1 β 2t + 2

]

Notando que α ⋅ β = −1 : 1+ 5 1− 5 − = 5 2 2 = −α −1 − α = − 5

α + α −1 = β + β −1 Juntando tudo:

[x ]g ( x ) = 15 [2 ⋅ ( − 1) n

t

]

+ α 2t + 2 + β 2t + 2 =

α 2 t + 2 + β 2 t + 2 − 2 ⋅ (α ⋅ β ) t +1  α t +1 − β t +1 =  5 5 

2

  = Ft 2  

E, em particular, f(n) é quadrado perfeito sempre que n é par.

58) Determine todos os primos p para os quais o número de um inteiro. SOLUÇÃO DE RODRIGO VILLARD MILET (RIO DE JANEIRO – RJ):

Resposta: p = 3 ou p = 7. EUREKA! N°12, 2001

50

2 p −1 − 1 é o quadrado p


Sociedade Brasileira de Matemática

Primeiramente, note que

2 p −1 − 1 é inteiro, para p > 2 (pelo pequeno Teorema de p

Fermat), e p = 2 não serve. Daí, como p é ímpar, 2 p −1 − 1 ≡ 0(mod 3). Se p = 3, 2 3−1 − 1 = 1 que é quadrado. Se p = 3, como 3

2p−1 −1 é quadrado, p

9 2 p −1 − 1 ⇔ 2 p −1 ≡ 1(mod 9) ⇔ ord 9 2 = 6 p − 1 ⇒ p ≡ 1(mod 6).

2 p −1 − 1 2 6 k − 1 ( 2 3 k − 1)( 2 3 k + 1) ∴ ∃ k ∈ ; p = 1 + 6k ⇒ = = p p p mdc{2 3 k − 1, 2 3 k + 1} = mdc{2, 2 3 k + 1} = 1 como p é primo ( p > 3), p 2 3 k − 1

ou p 2 3 k + 1. (i) p 2 3 k − 1. Daí, como 2 3 k − 1 e 2 3 k + 1 não tem fatores em comum, 2 3 k + 1 é quadrado ∴ ∃ q ∈ N ;2 3 k + 1 = q 2 ⇔ 2 3 k = ( q − 1)( q + 1) ⇒ q = 3 e k = 1 (de fato, q-1 e q+1 devem ser potências de 2,o que só é possível se forem 2 e 4). Nesse caso, 2 7 −1 − 1 = 9 que é quadrado. 7 (ii) p 2 3 k + 1. (Assuma k > 1, pois k = 1 já é solução)

(

)(

)

Daí, 2 3 k − 1 é quadrado ∴ ∃ m ∈ N ;2 3 k − 1 = m 2 ⇔ 2 k − 1 2 2 k + 2 k + 1 = m 2 . Seja d = mdc{2 k − 1, 2 2 k + 2 k + 1} . d 2 k − 1 ⇒ d ( 2 k − 1) = ( 2 2

d 2 2 k + 2 k + 1,

mas

logo

2 k ⇒ d | 3 ⇒ d = 1 ou 3 . • d = 1 ⇒ 2 2k + 2 k + 1 (2 ) < 2 quadrado. k

2

2k

(

é

)

2

+ 2 +1< 2 +1 , k

k

d 3.2 k .

No

então não é possível

EUREKA! N°12, 2001

51

− 2 ⋅ 2 k + 1),

d 2 k − 1, d não

Como

quadrado.

2k

entanto, 2

2k

divide

temos

+ 2 +1 k

ser


Sociedade Brasileira de Matemática

2k −1 é quadrado. •d =3⇒ 3 Como

2k − 1 é ímpar, 3

2k −1 ≡ 1(mod 8) ⇒ 2 k ≡ 4 mod 8 ⇒ k = 2 ⇒ p = 13 , que não é solução. 3 59) Um pedestal de altura a sustenta uma coluna de altura b (b > a). A que distância do monumento se deve colocar um observador para ver o pedestal e a coluna sob ângulos iguais? SOLUÇÃO DE CARLOS ALBERTO DA SILVA VÍCTOR (NILÓPOLIS – RJ):

Supondo a altura do observador desprezível, teremos:

b

(b > a)

a

θ

θ

x

a a+b tgθ = ; tg 2θ = x x 2tgθ 2.a/x a + b a+b = ∴ = ⇒ 2 x x 1 − tg θ a2 1− 2 x 2 2a.x a+b ⇒ = ⇒ 2ax 2 = (a + b )x 2 − a 2 (a + b) ⇒ (b − a )x 2 = a 2 (a + b ), 2 2 x x(a − a ) logo x = a.

a+b . b−a

EUREKA! N°12, 2001

52


Sociedade Brasileira de Matemática

60) Se num triângulo ABC , A = 2B, provar que a 2 = b(b + c). Obs.: a, b e c são, respectivamente, os lados opostos aos ângulos A, B e C. SOLUÇÃO DE MARCELO RIBEIRO DE SOUZA (RIO DE JANEIRO – RJ):

A 2x c

b

x B

a

Pela lei dos senos, temos: senx sen2 x = b a senx 2 senx ⋅ cos x = , obviamente x ≠ kπ , k ∈ = b a a = 2b ⋅ cos x (1)

I)

II) Usando lei dos cossenos: b 2 = a 2 + c 2 − 2ac ⋅ cos x 2ac ⋅ cos x = a 2 + c 2 − b 2 a2 + c2 − b2 (2) cos x = 2ac  a2 + c2 − b2   III) Substituindo (2) em (1): a = 2b ⋅   ac 2  

⇔ a 2 c = a 2 b + bc 2 − b 3 ⇔ a 2 (c − b ) = b ( c 2 − b 2 ) ⇔ a 2 (c − b) = b(c − b)(c + b). Se b ≠ c, temos a 2 = b(b + c). Se b=c, temos b=c=x, donde 4 x = π ⇒ x = π / 4 e

a = b 2 ⇒ a 2 = 2b 2 = b(b + c).

EUREKA! N°12, 2001

53

C


Sociedade Brasileira de Matemática

61) Na figura abaixo um quadrado EFGH foi colocado no interior do quadrado ABCD, determinando 4 quadriláteros. Se a, b, c, e d denotam os medidas das áreas dos quadriláteros, mostre que a + b = c + d . a c d b

SOLUÇÃO DE MÁRCIO ASSAD COHEN (RIO DE JANEIRO – RJ): Lema: A área de um triângulo cujos vértices no plano complexo são z1, z2, z3

{

}

1 Im ( z 2 − z1 )( z 3 − z1 ) . 2 Prova: Se z 2 − z1 = a ⋅ cisθ 1 ; z 3 − z1 = b ⋅ cisθ 2 : (sentido horário) é dada por S =

{

}

1 1 Im ( z 2 − z 1 ) ( z 3 − z 1 ) = Im {a ⋅ cis θ 1 b ⋅ cis ( −θ 2 )}= 2 2 ab absen θ Im {cis (θ 1 − θ 2 )}= =S 2 2 Problema:

i

1 w+iv w+v w w–v w–iv

-1

-i

EUREKA! N°12, 2001

54


Sociedade Brasileira de Matemática

Coloque os eixos de modo que os vértices do quadrado maior sejam 1, i, –1, – i (basta adotar uma unidade de medida tal que 2u.m = diagonal). Seja w o centro do quadrado menor, e w + v um de seus vértices. Então, os outros vértices do quadrado menor serão w + iv, w – i, w – iv (pois rodar 90o é multiplicar por i). Dividindo cada quadrilátero em triângulos como mostra a figura temos: 1 a = Im ( w + iv − 1)(i − 1) + (1 − w − iv)v(1 − i ) 2 1 c = Im ( w − iv − 1)(−i + 1) + (−1 − w + iv)v(i − 1) 2 1 a + c = Im (1 + i) (− w − iv + 1) + v(1 − w − iv) + ( w − iv + 1) + v(1 + w − iv) 2

{

}

{

{

}

[

]}

Note que w é cancelado e a + c independe do centro do quadrado menor:

{

}

a + c = Im (1 + i)(1 − iv)(1 + v) Da mesma forma, obtemos:

{

}

1 b + d = Im (w + v + i)(1 + i) + (−i − w − v)v(−i − 1) + (w − v − i)(−1 − i) + (1 − w + v)v(1 + i) = 2 1 Im (1 − i) (w + v + i) + (i + w + v)v + (−w + v + i) + (−w + v + i)v = 2 Im (1 − i)(i + v)(v + 1)

{

{

[

]}

}

Fatorando, obtemos

{

}

{

}

b + d = Im − i(i + 1)(i + v)(v + 1) = Im (i + 1)(1 − iv)(v + 1) = a + c . 62) Se ABCD é um quadrilátero convexo tal que os lados AB, BC , CD e DA medem respectivamente a, b, c e d e que α, β , σ e γ são as medidas dos seus ângulos internos, mostre que a medida da área desse quadrilátero, denotada por (ABCD), é dada por:

EUREKA! N°12, 2001

55


Sociedade Brasileira de Matemática D

γ

d

A α

c

d1

σ

a

C

b

β B

(ABCD) =

( p − a )( p − b)( p − c)( p − d ) − abcd cos 2 δ onde:

a+b+c+d 2 β +γ α +σ δ= (a fórmula também vale se fizermos δ = ). 2 2 p=

SOLUÇÃO DE WALLACE RODRIGUES DE HOLANDA MIRANDA (TERESINA – PI):

Antes da resolução do problema precisa-se ressaltar algumas noções fundamentais: 1) cos(α + σ ) = cos α ⋅ cos σ − senα ⋅ senσ cos(2δ ) = cos α ⋅ cos σ − senα ⋅ senσ

cos 2 δ − sen 2δ = cos α ⋅ cos σ − senα ⋅ senσ

(

)

cos 2 δ − 1 − cos 2 δ = cos α ⋅ cos σ − senα ⋅ senσ senα ⋅ senσ = cos α ⋅ cos σ − 2 cos 2 δ + 1

2)

(ABD ) = 1 ⋅ a ⋅ d ⋅ senα

(BCD ) = 1 ⋅ b ⋅ c ⋅ senσ

2 2 3) Pela lei dos cossenos nos triângulos ABD e BCD:

a 2 + d 2 − 2ad ⋅ cos α = d 12 = b 2 + c 2 − 2bc ⋅ cos σ a 2 + d 2 − b 2 − c 2 = 2ad ⋅ cos α − 2bc ⋅ cos σ

Agora vamos à resolução do problema: (ABCD) = (ABD) + (BCD) EUREKA! N°12, 2001

56


Sociedade Brasileira de Matemática

(ABCD ) = 1 ⋅ a ⋅ d ⋅ senα + 1 ⋅ b ⋅ c ⋅ senσ 2

2

2(ABCD ) = a ⋅ d ⋅ senα + b ⋅ c ⋅ senσ

(2(ABCD ))2 = (a ⋅ d ⋅ senα + b ⋅ c ⋅ senσ )2 2 4(ABCD ) = a 2 ⋅ d 2 ⋅ sen 2α + 2abcd ⋅ senα ⋅ senσ + b 2 ⋅ c 2 ⋅ sen 2σ Substituindo (sen 2 ) por (1 − cos 2 ) e (senα ⋅ senσ ) por

(cos α ⋅ cos σ − 2 cos

2

)

δ + 1 de 1) fica:

(

)

(

)

4(ABCD) = a ⋅ d ⋅ 1 − cos 2 α + 2abcd ⋅ 1 + cos α ⋅ cos σ − 2 cos 2 δ + 2

2

2

(

+ b 2 ⋅ c 2 ⋅ 1 − cos 2 σ

)

(

)

4(ABCD ) = a ⋅ d 2 ⋅ 1 − cos 2 α + abcd ⋅ (2 + 2 cos α ⋅ cos σ ) + 2

2

(

)

+ b ⋅ c 1 − cos σ − 4abcd ⋅ cos 2 δ 2

2

2

(

)

4(ABCD ) = a ⋅ d ⋅ 1 − cos 2 α + abcd (1 − cos α − cos σ + cos α ⋅ cos σ + 1 + 2

2

2

(

)

+ cos α + cos σ + cos α ⋅ cos σ ) + b 2 c 2 1 − cos 2 σ − 4 abcd ⋅ cos 2 δ

(

)

4( ABCD) = a ⋅ d 1 − cos α + abcd [(1 − cos α )(1 − cos σ ) + (1 + cos α )(1 + cos σ )] + 2

(

2

2

)

2

b 2 c 2 1 − cos 2 σ − 4abcd ⋅ cos 2 δ 4( ABCD) 2 = ad (1 + cos α )ad (1 − cos α ) + ad (1 − cos α )bc(1 − cos σ ) + + ad (1 + cos α )bc(1 + cos σ ) + bc(1 + cos σ )bc(1 − cos σ ) − 4abcd ⋅ cos 2 δ

4( ABCD) 2 = ad (1 + cosα )[ad (1 − cosα ) + bc(1 + cosσ )] + bc(1 − cosσ )[ad (1 − cosα ) + + bc(1 + cosσ )] − 4abcd ⋅ cos2 δ

4( ABCD) 2 = [ad (1 − cosα ) + bc(1 + cosσ )][ad (1 + cosα ) + bc(1 − cosσ )] − 4abcd ⋅ cos2 δ 16( ABCD) 2 = 2(ad − ad ⋅ cosα + bc + bc ⋅ cos σ ) ⋅ 2 ⋅ (ad + ad ⋅ cosα + bc − bc ⋅ cos σ ) − 16abcd ⋅ cos 2 δ 16( ABCD) 2 = −(2ad ⋅ cos α − 2bc ⋅ cos σ − 2ad − 2bc)(2ad ⋅ cos α − 2bc ⋅ cos σ +

+ 2ad + 2bc) − 16abcd ⋅ cos 2 δ Substituindo (2ad ⋅ cos α − 2bc ⋅ cos σ ) por (a 2 + d 2 − b 2 − c 2 ) de 3) fica: 16( ABCD) 2 = −(a 2 − 2ad + d 2 − b 2 − 2bc − c 2 )(a 2 + 2ad + d 2 − b 2 + 2bc − c 2 ) − 16abcd ⋅ cos 2 δ 16 ( ABCD ) 2 = − ( a − d ) 2 − (b + c ) 2 ( a + d ) − (b − c ) 2 − 16 abcd ⋅ cos 2 δ

[

][

EUREKA! N°12, 2001

57

]


Sociedade Brasileira de Matemática

16( ABCD) 2 = −(a − d + b + c)(a − d − b − c)(a + d − b + c)(a + d + b − c) − 16abcd ⋅ cos2 δ 16( ABCD) 2 = (a + b + c − d )(−a + b + c + d )(a − b + c + d )(a + b − c + d ) −16abcd⋅ cos2 δ 16( ABCD) 2 = (−a + b + c + d )(a − b + c + d )(a + b − c + d )(a + b + c − d ) − 16abcd ⋅ cos2 δ (−a + b + c + d) (a − b + c + d) (a + b − c + d ) (a + b + c − d) ⋅ ⋅ ⋅ − abcd⋅ cos2 δ 2 2 2 2 ( ABCD) 2 = ( p − a )( p − b)( p − c)( p − d ) − abcd ⋅ cos 2 δ

( ABCD) 2 =

( ABCD) = ( p − a )( p − b)( p − c)( p − d ) − abcd ⋅ cos 2 δ . Agradecemos também o envio das soluções e a colaboração de: Bruno de Souza Ramos Everaldo de Melo Bonotto Fernando Carvalho Ramos Geraldo Perlino Jorge Silva Junior Marcelo Ribeiro Marcelo Rufino de Oliveira Marcílio Miranda de Carvalho Mateus Queiroz Guilherme de Oliveira Oswaldo Melo Sponquiado

Realengo – RJ Guararapes – SP Santa María – RS Itapecerica da Serra – SP Cachoeiro de Itapemerim – ES Rio de Janeiro – RJ Belém – PA Teresina – PI Fortaleza – CE Olímpia – SP

Seguimos aguardando o envio de soluções do problema proposto No. 57 publicado na revista Eureka! No. 11.

EUREKA! N°12, 2001

58


Sociedade Brasileira de Matemática

PROBLEMAS PROPOSTOS

Convidamos o leitor a enviar soluções dos problemas propostos e sugestões de novos problemas para os próximos números.

63) Prove que existem infinitos números naturais múltiplos de 51000 sem nenhum 0 na representação decimal. 64) Iniciando de um certo inteiro positivo, é permitido fazer apenas uma operação: o dígito das unidades é separado e multiplicado por 4, e então este valor é somado ao restante do número. Por exemplo, o número 1997 é transformado para 7.4 + 199 = 227. A operação é feita repetidamente. Prove que se a seqüência de números obtida contém 1001, então nenhum dos números na seqüência pode ser um número primo. 65) Determine todos os inteiros N tais que, em base 10, os dígitos de 9N são os mesmos dígitos de N na ordem inversa, e N possui no máximo um dígito igual a 0. 66) Prove que, dados um inteiro n ≥ 1 e um conjunto A ⊂ = / n = com n B ⊂ = / n 2 = com n elementos tal que elementos existe A + B = x + y x ∈ A, y ∈ B ⊂ = / n 2 = tem mais de n 2 / 2 elementos. 2

{

}

67) Seja ABCD um quadrilátero tal que os círculos circunscritos aos triângulos ABC e BCD são ortogonais.Prove que os círculos circunscritos aos triângulos BCD e DAB também são ortogonais.

Problema 63 proposto por Wallace Rodrigues de Holanda Miranda (Teresina – PI); Problemas 64 e 65 propostos por Marcelo Rufino de Oliveira (Belém – PA); Problema 67 proposto por Luciano Castro (Rio de Janeiro – RJ).

EUREKA! N°12, 2001

59


Sociedade Brasileira de Matemática

AGENDA OLÍMPICA XXIV OLIMPÍADA BRASILEIRA DE MATEMÁTICA NÍVEIS 1, 2 e 3 Primeira Fase – Sábado, 8 de junho de 2002 Segunda Fase – Sábado, 14 de setembro de 2002 Terceira Fase – Sábado, 19 de outubro de 2002 (níveis 1, 2 e 3) Domingo, 20 de outubro de 2002 (níveis 2 e 3 - segundo dia de prova). NÍVEL UNIVERSITÁRIO Primeira Fase – Sábado, 14 de setembro de 2002 Segunda Fase – Sábado, 19 e Domingo, 20 de outubro de 2002 ♦

VIII OLIMPÍADA DE MAIO maio de 2002 ♦

XIII OLIMPÍADA DE MATEMÁTICA DO CONE SUL junho de 2002 Fortaleza – CE, Brasil ♦

XLIII OLIMPÍADA INTERNACIONAL DE MATEMÁTICA julho de 2002 Glasgow, Reino Unido ♦

XVII OLIMPÍADA IBEROAMERICANA DE MATEMÁTICA setembro de 2002 El Salvador ♦

V OLIMPÍADA IBEROAMERICANA DE MATEMÁTICA UNIVERSITÁRIA outubro de 2002 ♦♦♦

EUREKA! N°12, 2001

60


Sociedade Brasileira de Matemática

COORDENADORES REGIONAIS Amarísio da Silva Araújo Alberto Hassen Raad

(UFV) (UFJF)

Viçosa – MG Juiz de Fora – MG

Angela Camargo Benedito Tadeu Vasconcelos Freire

(Centro de Educação de Adultos – CEA) (UFRN)

Blumenau – SC Natal – RN

Carlos Frederico Borges Palmeira Claudio Arconcher

(PUC-Rio) (Colégio Leonardo da Vinci)

Rio de Janeiro – RJ Jundiaí – SP

Claus Haetinger

(UNIVATES)

Lajeado – RS

Cleonor Crescêncio das Neves Élio Mega

(UTAM) (Colégio Etapa)

Manaus – AM São Paulo – SP

Rosângela Souza Florêncio Ferreira Guimarães Filho

(Colégio Singular) (UFES)

Santo André – SP Vitória – ES

Gisele de Araújo Prateado Gusmão Ivanilde Fernandes Saad

(UFGO) (UC. Dom Bosco)

Goiânia – GO Campo Grande– MS

Jacqueline Fabiola Rojas Arancibia

(UFPB)

João Pessoa – PB

João Benício de Melo Neto João Francisco Melo Libonati

(UFPI) (Grupo Educacional Ideal)

Teresina – PI Belém – PA

Irene Nakaoka José Carlos Pinto Leivas

(UEM) (UFRG)

Maringá – PR Rio Grande – RS

José Cloves Saraiva

(UFMA)

São Luis – MA

José Gaspar Ruas Filho José Luiz Rosas Pinho

(ICMC-USP) (UFSC)

São Carlos – SP Florianópolis – SC

José Vieira Alves Marcelo Rufino de Oliveira

(UFPB) (Sistema Titular de Ensino)

Campina Grande – PB Belém – PA

Licio Hernandes Bezerra Luzinalva Miranda de Amorim

(UFSC) (UFBA)

Florianópolis – SC Salvador – BA

Marcondes Cavalcante França

(UFC)

Fortaleza – CE

Pablo Rodrigo Ganassim Paulo Henrique Cruz Neiva de Lima Jr.

(Liceu Terras do Engenho) (Escola Técnica Everardo Passos)

Piracicaba – SP SJ dos Campos – SP

Reinaldo Gen Ichiro Arakaki Ricardo Amorim

(INPE) (Centro Educacional Logos)

SJ dos Campos – SP Nova Iguaçu – RJ

Roberto Vizeu Barros

(Colégio Acae)

Volta Redonda – RJ

Sérgio Cláudio Ramos Silvio de Barros Melo

(IM-UFRGS) (UFPE)

Porto Alegre – RS Recife – PE

Tadeu Ferreira Gomes Tomás Menéndez Rodrigues

(UEBA) (U. Federal de Rondônia)

Juazeiro – BA Porto Velho – RO

Valdenberg Araújo da Silva Wagner Pereira Lopes

(U. Federal de Sergipe) (Escola Técnica Federal de Goiás)

São Cristovão – SE Jataí – GO

EUREKA! N°12, 2001

61


Turn static files into dynamic content formats.

Create a flipbook
Issuu converts static files into: digital portfolios, online yearbooks, online catalogs, digital photo albums and more. Sign up and create your flipbook.